Você está na página 1de 282

GRAMMAR IN

USE,
READING AND
LISTENING

DOSSIER 2-1: 1 DISEO


Units 1 to 60 and Readings

Sant Jaume Apstol


Prof. Victor esteve
GRAMMAR IN USE,
READING AND LISTENING

READTHEORY
Name________________
Date________________

Reading Comprehension 1

Level 5

Directions: Read the passage. Then answer the questions below.


Marcia says that all of her friends have a cell phone, but Marcias mom doesn't want to buy her
one. Marcia's mom doesn't want Marcia to play video games either. What is more, the Internet scares her.
Marcia's mom says, If Marcia has a cell phone, how do we know whom she is talking to? Video games
are bad for you. The Internet is dangerous and uncontrolled. Its like having a gun in the house. We
should just ban her from using the computer, and I'm not buying her a cell phone until she is eighteen.
This is the only way we can be sure that Marcia is safe."
Marcias dad disagrees with Marcia's mom. Although he agrees that there are some dangers to it,
he likes the Internet, and finds it to be very useful. The trouble is, he says, We just cant stop Marcia
from using the Internet, as this would put her at a disadvantage. What is more, I like video games. I think
that, when played in moderation, they are fun. Obviously, it is not good to play them without restraint or
self-control. Finally, I think Marcia needs a cell phone. We cant take these things away.

Questions

1) Which of the following best describes the difference between Marcia's mom and Marcia's dad?
A.
B.
C.
D.

Mom wants to ban Marcia from using the computer, while dad likes to play video games.
Mom thinks technology is dangerous, while dad thinks it can be useful.
Mom cares little about Marcia's future, while dad is very supportive.
Mom is very strict while Dad is open minded.

2) Which of the following best describes the similarity between Marcia's mom and Marcia's dad?
A.
B.
C.
D.

Mom and dad both like technology.


Mom and dad both think video games are bad.
Mom and dad both think the internet is dangerous.
Mom and dad both care about Marcia's wellbeing.

3) In paragraph 1, Marcia's mom says, "It's like having a gun in the house." She says this in order to
A.
B.
C.
D.

support the idea that the Internet is dangerous


reject the claim that guns can be safe if used responsibly
encourage Marcia's dad to purchase a gun
explain why the Internet is uncontrolled

4) In paragraph 2, Marcia's dad says, "We just cant stop Marcia from using the Internet, as this would
put her at a disadvantage." What does Marcia's dad mean by this?
A.
B.
C.
D.

Marcia needs to learn how to use the internet if she wants to have friends in the future.
Marcia should not stop using the internet because this will seriously slow her learning.
If a person's ability to use the internet becomes important in the future, Marcia will be at a loss.
If Marcia does not learn to use the internet on her own, then she will never learn to recognize its
dangers.
Copyright Read Theory LLC, 2012. All rights reserved.

5) In paragraph 2, Marcia's dad says, "Finally, I think Marcia needs a cell phone." Given what you know
about Marcias moms concerns, what is the best reason Marcia's dad can provide to convince
Marcia's mom that Marcia needs a cell phone?
A.
B.
C.
D.

Marcia can use her cell phone to talk to her friends, instead of needing to borrow one of ours.
Having a cell phone will teach Marcia how to use new technology.
Because all of her friends have one, it would be unfair to disallow Marcia to have a cell phone.
If Marcia's is in trouble she can use her cell phone to call for help.

6) Which of the following describes a level of game play that Marcia' dad would disapprove of?
A. On Saturday, Marcia plays video games all evening. The next day, she wakes up early and goes
for a walk through the woods near her house. When her friend calls Marcia, they talk about the
what they are going to wear to school on Monday.
B. Marcia plays video games for an hour or two. Then she eats lunch and meets her friends at the
skating rink. That night, Marcia and her friends go to see a movie.
C. Marcia plays video games all morning. When her friends ask her to come play outside, Marcia
tells them that she is too busy and continues to play.
D. Marcia plays video games for an hour every day for five days straight. During these days, she
does not watch any TV.
7) Marcia's mom can best be described as
A.
B.
C.
D.

ridiculous
careful
cruel
rude

8) Marcia's dad can best be described as


A.
B.
C.
D.

shameless
foolish
wild
reasonable

9) Which of the following would be the best way for Marcia to change the way her mom thinks about
technology?
A.
B.
C.
D.

Read her a newspaper article that talks about the importance of technology.
Provide her an instruction manual detailing how the latest cell phone functions.
Take her to the library and show her the top five most popular internet websites.
Spend the weekend playing video games with her.

Copyright Read Theory LLC, 2012. All rights reserved.

Answers and Explanations


1) B
In paragraph 1, Marcia's mom says, "The Internet is dangerous and uncontrolled," and "Video games are bad for you." This lets us
know that mom thinks cell phones and the Internet are dangerous and that video games are bad for you. In paragraph 2 we learn
that although he agrees with mom that there are some dangers involved with using the internet, he "likes the Internet, and finds it to
be very useful." Moreover, dad thinks Dad continues to say, "I like video games." At the end of paragraph 2 dad says, "I think Marcia
needs a cell phone." This let us know that dad thinks the Internet is useful, video games are fun (when played in moderation), and
that Marcia needs a cell phone. Cell phones, video games and the Internet are all examples of technology. Mom thinks these things
are dangerous and Dad thinks these things are useful. Therefore (B) is correct. Mom does want to ban Marcia from the computer
and Dad does like playing video games. However, (A) is not the best choice because these are not the only differences between
Marcias Mom and Dad. In addition, Mom doesnt want Marcia using certain technologies because she is worried that they are
dangerous or bad for Marcia. Marcias dad thinks that Marcia should use the technology because it is useful and fun. This means
that both parents want the best for Marcia. Since both parents support Marcia and care about her future, (C) is incorrect. Finally, just
because Mom is concerned about the use of certain technologies doesnt mean that she is strict in general. Just because Dad is
pro-technology does not mean that he is open minded about everything. Since choice (D) makes generalizations not supported by
the passage, it is incorrect.
2) D
In paragraph 1, Marcia's mom describes why she thinks technology is bad. She says that the internet, cell phones, and video games
are dangerous. At the end of the paragraph she says, "This is the only way we can be sure that Marcia is safe." This lets us know
that the reason why she dislikes technology is because it will put Marcia's safety at risk. This lets us know that Marcia's mom cares
about Marcia's wellbeing. In paragraph 2, we learn about why Marcia's dad thinks technology is good for Marcia. According to him,
technology is "very useful." Moreover, he does not want to take technology away from Marcia, as this will "put her at a
disadvantage." This lets us know that Marcia's dad also cares about Marcia's wellbeing. Therefore (D) is correct. The passage does
not provide information to support choices (A), (B), and (C). Therefore they are incorrect.
3) A
Feedback: To answer this question, it helps to use context. In paragraph 1, Mom says The Internet is dangerous and uncontrolled.
Its like having a gun in the house. Since Mom says that the Internet is dangerous immediately before she compares it to having a
gun in the house, we can understand that Mom draws this comparison to illustrate how dangerous she thinks the Internet is. This
makes (A) the correct choice. Mom is not arguing that guns can never be safe. Rather, she compares the Internet to guns to
express how dangerous she finds the Internet. This makes (B) incorrect. Mom does not encourage Dad to buy a gun, so (C) is not
the best choice. Mom compares having the Internet to having a gun in the house. She does this to explain how dangerous the
internet is, not explain why the Internet is uncontrolled. This rules out (D).
4) C
disadvantage (noun): an unfavorable or poor condition.
In paragraph 2, we learn about why Marcia's dad thinks technology is good for Marcia. According to him, technology is "very useful."
Moreover, he does not want to take technology away from Marcia, as this will "put her at a disadvantage." This lets us know that
Marcia's dad also cares about Marcia's wellbeing. So, it makes sense to think that if Marcia does not learn to use the internet, and
the internet becomes important in the future, Marcia will be at a loss. Choice (C) is correct. The passage does not provide
information to support choices (A), (B), and (D). Therefore they are incorrect.
5) D
In paragraph 1, we learn that Marcias mom does not want Marcia to have a cell phone. This is because if Marcia had a cell phone,
Marcia's mom would not know who Marcia was talking to. In addition, Marcia's mom wants to ban Marcia from using the computer
because she thinks the Internet is dangerous. We can understand from this that Mom is very concerned about Marcias safety.
Since Marcia could use her cell phone to call for help, the cell phone may actually make Marcia safer. Since Marcias mother is very
concerned about safety, this reason would be most likely to change her mind about Marcia having a cell phone. This makes (D) the
best choice. The passage does not provide any information to make us think that Marcias mom is concerned about Marcia
borrowing her cell phone. Since this reason does not address any of Moms concerns, (A) is not a good choice. Moreover, Marcias
mom does not appreciate any of the technology discussed in the passage, so it is very unlikely that she thinks Marcia needs to learn
how to use new technology. This makes (B) incorrect. As explained above, Marcias mom is very concerned with safety and what is
best for her daughter. The fact that all of Marcias friends have cell phones does not address any these concerns. Moreover, in the
opening line of the passage we learn that, "Marcia says that all of her friends have a cell phone, but Marcias mom doesn't want to
buy her one." This lets us know that Marcia's mom does not care about the fact that all of Marcia's friends have cell phones. This
means (C) is incorrect.
6) C
In paragraph 2, Marcia's dad says, "What is more, I like video games. I think that, when played in moderation, they are fun.
Obviously, it is not good to play them without restraint or self-control." From this, we can understand that Marcia's dad thinks it is
okay to play video games in moderation. When you do something in moderation, you exercise restraint, and do not do it too much.
Using this information, we can understand that if Marcia plays games all morning and then declines an invitation to play outside with
her friends, then she is not playing in moderation. This means (C) is correct. (A) is incorrect because although Marcia plays video
games all evening, she goes for a walk the next day. When her friend calls, they talk about school. This shows that Marcia has other
interests in addition to playing video games. What is more, she plays the video games on a Saturday, which is a weekend.
Weekends are good times to play games. (B) is incorrect because Marcia only plays video games for an hour or two. Then she goes
to the skating rink with her friends. This is a social activity in which Marcia can get some exercise. Later, they go to a movie. This

Copyright Read Theory LLC, 2012. All rights reserved.

shows that Marcia has other interests in addition to playing video games. (D) is incorrect because although Marcia plays video
games for five days straight, she only plays them for one hour each day. This is a moderate level of play. What is more, she does
not watch TV. This is good because it means she is spending her time doing something other than looking at a screen. For example,
she might be doing homework, playing outside, or doing arts and crafts.
7) B
In paragraph 1, we learn about how Marcia's mom thinks the internet is "dangerous and uncontrolled." For this reason, she does not
want Marcia to use technological devices. She is very concerned about her daughter's wellbeing, and does not want her to take any
risks. Using this information, we can understand that Marcia's mom can best be described as careful. Although Marcia's mom
seems to have an extreme view of technology, she does this only because she cares about her child, Marcia. Caring about one's
child is not ridiculous. Choice (B) is correct. Although Marcia's mom seems to have an extreme view of technology, she does this
only because she cares about her child, Marcia. Caring about one's child is not ridiculous. Therefore (A) is incorrect. (C) is incorrect
because it is wrong to think that Marcia's mom is cruel, or mean, simply because she has an extreme view of technology. She does
this only because she cares about her child, Marcia. (D) is incorrect because Marcia's mom is never rude, or impolite.
8) D
In paragraph 2, we learn that Marcia's dad thinks that although "there are some dangers to it, he likes the Internet, and finds it to be
very useful." Marcia's dad continues to say, "We just cant stop Marcia from using the Internet, as this would put her at a
disadvantage. What is more, I like video games. I think that, when played in moderation, they are fun. Obviously, it is not good to
play them without restraint or self-control." This lets us know that Marcia's dad cares about Marcia's future. He realizes that there
are dangers involved in using technology, such as the risk of playing video games too much, but that the world is changing. If Marcia
does not change with it, she will be left behind. This is a reasonable way to think. This makes (D) correct. Although Marcia's dad is
more willing to take the risks involved in allowing Marcia to use technology, this does not mean that he is shameless, foolish, or wild.
These words are too strongly negative. In life, risks must be taken. One must face these risks, and think about ways to overcome
them. This means choices (A), (B), and (C) are incorrect.
9) A
In paragraph 1, we learn that Marcia's mom does not like the internet and other forms of technology because she thinks they are
"dangerous and uncontrolled." In order to change the way she thinks about technology, it makes sense to read her a newspaper
article that talks about the importance of technology. This is a good idea because it involves a form of communication that is not
technological. Also, a newspaper article is typically viewed as a reputable source. Marcia's mom will be most likely to consider the
opinion of a newspaper article author. Therefore (A) is correct. (B) is incorrect because an instruction manual will not provide
arguments in favor of the idea that a child should be able to use technology. Although it may be a good idea to take Marcia's mom to
the library to show her the internet, it is not a good idea to show her the top five most popular websites. The most popular websites
may not demonstrate the true value to be gained by using the internet. This means (C) is incorrect. Although it may be a good idea
for Marcia to play a video game with her mom to show her what they are like, it is not a good idea to play them all weekend with her.
This would be bad because you should not play video games for such a long time. Marcia's mom would not like this. Even Marcia's
dad, who is in favor of using technology, would consider this level of game play to be excessive, or immoderate. Choice (D) is
incorrect.

Copyright Read Theory LLC, 2012. All rights reserved.

READTHEORY
Name________________
Date________________

Reading Comprehension 2

Level 5

Directions: Read the passage. Then answer the questions below.

Have you ever heard someone use the phrase once in a blue moon? People use this
expression to describe something that they do not do very often. For example, someone might say that
he tries to avoid eating sweets because they are unhealthy, but will eat chocolate once in a blue moon.
Or someone who does not usually like to go to the beach might say I visit the shore once in a blue
moon. While many people use this phrase, not everyone knows the meaning behind it.
The first thing to know is that the moon itself is never actually blue. This is just an expression. The
phrase blue moon actually has to do with the shape of the moon, not the color.
As the moon travels around the earth, it appears to change shape. We associate certain names
with certain shapes of the moon. For example, when we can see a small part of the moon, it is called a
crescent moon. A crescent is a shape that looks like the tip of a fingernail. When we cannot see the moon
at all, it is called a new moon. When we can see the entire moon, it is called a full moon. Usually, there is
only one full moon every month. Sometimes, however, there will be two full moons in one month. When
this happens, the second full moon is called a blue moon.
Over the next 20 years, there will only be 15 blue moons. As you can see, a blue moon is a very
rare event. This fact has led people to use the expression once in a blue moon to describe other very
rare events in their lives.

Questions
1) Which of the following would be a good example of someone doing something once in a blue
moon?
A. Mary likes to go to the mountains every weekend. Mary goes to the mountains once in a blue
moon.
B. Tom rarely remembers to take out the trash. Tom takes out the trash once in a blue moon.
C. Cindy hates to wash the dishes. Nevertheless, she does it every day. Cindy washes the dishes
once in a blue moon.
D. Ming sometimes forgets to do his homework. Ming forgets to do his homework once in a blue
moon.
2) When does a blue moon happen in nature?
A.
B.
C.
D.

when there are two full moons in one month


when the moon has a blue color
when we cannot see the moon at all
when we can only see a small part of the moon

Copyright Read Theory LLC, 2012. All rights reserved.

3) Using the passage as a guide, it can be understood that which of the following sentences does not
contain an expression?
A.
B.
C.
D.

Thomas has lost his mind.


An apple a day keeps the doctor away.
I'll mow the grass after I finish my homework.
It's never a bad time to start something new.

4) As described in paragraph 3, what is another example of something that has a crescent shape?
A.
B.
C.
D.

your thumb
a distant star
the letter C
the letter "H"

5) In the final paragraph, the author states: Over the next 20 years, there will only be 15 blue moons.
This means that over the next 20 years, a blue moon will happen
A.
B.
C.
D.

once a year
less than once a year
more than once a year
not enough information is provided

6) As used in the final paragraph, which is the best antonym for rare?
A.
B.
C.
D.

common
strange
colorful
infrequent

7) In the final paragraph the author writes, "As you can see, a blue moon is a very rare event." The
purpose of this statement is to
A.
B.
C.
D.

answer an earlier question


provide an example
support an upcoming conclusion
challenge a previous statement

Copyright Read Theory LLC, 2012. All rights reserved.

Answers and Explanations


1) B
In the first and last paragraphs, we learn that the phrase once in a blue moon describes very rare events. If Tom rarely remembers
to take out the trash, then it makes sense to say he takes out the trash once in a blue moon. Therefore (B) is correct. (A) is
incorrect because Mary goes to the mountains every weekend. This happens more often than the blue moon. (C) is incorrect
because Cindy washes the dishes every day. This happens more often than the blue moon. (D) is incorrect because Ming
sometimes forgets to do his homework. This happens more often than the blue moon.
2) A
In the third paragraph, the author tells us that when there are two full moons in one month, the second moon will be called a blue
moon. Choice (A) is correct. The passage does not provide information to support choices (B), (C), and (D). Therefore they are
incorrect.
3) C
In paragraph 1 the author introduces the phrase "once in a blue moon." Then the author tells us that "People use this expression to
describe something that they do not do very often." This lets us know that the phrase "once in a blue moon" is an expression. In
paragraph 2 the author writes, "The first thing to know is that the moon itself is never actually blue. This is just an expression." This
lets us know that an expression contains information or facts that are not actually true. The sentence in (C) does not contain any
facts that are not actually true. Therefore it is the correct choice. (A) is incorrect because a person cannot actually lose his or her
mind. (B) is incorrect because eating an apple a day does not necessarily keep someone from having to visit the doctor. (D) is
incorrect because there are some times in which it is indeed a bad idea to start something new. For example, it's a bad idea to learn
how to fly a kite in a lightning storm.
4) C
crescent (noun): the figure of the moon in its first or last quarter, resembling the segment of a ring thinning to points at the ends.
In paragraph 3, the author says that a crescent is a shape that looks like the tip of a fingernail. Of all the choices, the letter C is
the one most similar to this shape. Therefore (C) is correct. The passage does not provide information to support choices (A), (B),
and (D). Therefore they are incorrect.
5) B
This question involves some math. If a blue moon will happen 15 times over the next 20 years, this is the same as saying it will
happen 15 out of 20 times, or 15/20. Since 15/20 is less than one, we know that the blue moon will happen less than one time per
year. This means (B) is correct. The passage does not provide information to support choices (A), (C), and (D). Therefore they are
incorrect.
6) A
rare (adjective): something that does not happen very often.
The question asks us to find the best antonym. Antonyms are words that have opposite meanings. In the last paragraph, the author
describes a blue moon as a very rare event. In this paragraph and the rest of the passage, blue moons are described as events
that do not happen very often. Therefore, common, a word that means happening very often, is the best opposite of rare. Choice (A)
is correct. The passage does not provide information to support choices (B), (C), and (D). Therefore they are incorrect.
7) C
To answer this question correctly, it helps to use context. In the final paragraph the author writes, "As you can see, a blue moon is a
very rare event. This fact has led people to use the expression once in a blue moon to describe other very rare events in their
lives." The key phrase here is "This fact has led people to." This is meant to show that the fact that a blue moon is a very rare event
lends support to the conclusion that the expression once in a blue moon to describe other very rare events in their lives. This
means choice (C) is correct. The passage does not provide information to support choices (A), (B), and (D). Therefore they are
incorrect.

Copyright Read Theory LLC, 2012. All rights reserved.

READTHEORY
Name________________
Date________________

Reading Comprehension 3

Level 5

Directions: Read the passage. Then answer the questions below.


Many people who are looking to get a pet dog get a puppy. There are many reasons why people
get puppies. After all, puppies are cute, friendly, and playful. But even though puppies make good pets,
there are good reasons why you should consider getting an adult dog instead.
When you get a puppy, you have to teach it how to behave. You have to make sure that the
puppy is housebroken so that it does not go to the bathroom inside the house. You have to teach the
puppy not to jump up on your guests or chew on your shoes. You have to train the puppy to walk on a
leash. This is a lot of work.
On the other hand, when you get an adult dog, there is a good chance that it will already know
how to do all of the previously mentioned things. Many adult dogs have already been housebroken. Many
adult dogs will not jump on or chew things that you do not want them to jump on or chew. Many adult
dogs will be able to walk on a leash without pulling you to the other side of the street.
Puppies also have a lot of energy and want to play all of the time. This can be fun, but you might
not want to play as much as your puppy does. Puppies will not always sleep through the night or let you
relax as you watch television.
On the other hand, most adult dogs will wait on you to play. What is more, they will sleep when
you are sleeping and are happy to watch television on the couch right beside you.
There is one last reason why you should get an adult dog instead of a puppy. When most people
go to the pound to get a dog, they get a puppy. This means that many adult dogs spend a lot of time in
the pound, and some never find good homes. So if you are looking to get a dog for a pet, you should think
about getting an adult dog. They are good pets who need good homes.

Questions

1) The author apparently thinks that puppies are


A.
B.
C.
D.

bad pets because they take too much work to own


friendly, playful, and a lot of work
not as cute as adult dogs
not as playful as adult dogs

2) As used in paragraph 2, which is the best synonym for behave?


A.
B.
C.
D.

listen
understand
train
act

3) The main purpose of paragraph 2 is to explain how puppies


A.
B.
C.
D.

are very immature


do not make good pets
can be very destructive
are a lot of work

Copyright Read Theory LLC, 2012. All rights reserved.

4) As used in paragraph 2, which is the best example of a dog that is housebroken?


A.
B.
C.
D.

Spot goes outside to use the bathroom.


Rex always breaks things inside of the house.
Rover never jumps on guests.
Muffin chews on peoples shoes.

5) According to the passage, why are adult dogs easier to take care of than puppies?
I. Puppies need to learn how to walk nicely on a leash.
II. Adult dogs have less energy than puppies do.
III. It is harder for adult dogs to find a home than it is for puppies.
A.
B.
C.
D.

l only
I and II only
II and III only
I, II, and III

6) Based on information in the passage, which of the following statements is false?


A.
B.
C.
D.

Puppies have a lot of energy.


Puppies need a lot of attention.
Adult dogs do not like to play.
Adult dogs do not need eat very much.

7) As used in paragraph 4, which is the best synonym for relax?


A.
B.
C.
D.

work
leave
play
rest

8) The author begins paragraphs 3 and 5 with the phrase, "On the other hand." This phrase is used to
A.
B.
C.
D.

highlight an example
contrast previous information
contradict a later statement
support the upcoming paragraph

9) In the final paragraph, the author says, "many adult dogs spend a lot of time in the pound, and some
never find good homes." Based on what you have read in the passage, why is this most likely the
case?
A. People see adult dogs as unhappy and dangerous, while they see puppies as cute and friendly.
B. People understand that most adult dogs still need to a lot of training before they understand how
to behave properly.
C. People think that puppies are cute and playful and do not always think about how much work it
will take to train them.
D. People do not want to get a dog that does not have much time left to live.
10) Based on information in the passage, it can be understood that someone who owns a puppy must be
A.
B.
C.
D.

strict
serious
careful
responsible
Copyright Read Theory LLC, 2012. All rights reserved.

Answers and Explanations


1) B
In paragraph 1, the author says that "puppies are cute, friendly, and playful." In paragraph 2, the author talks about all of the things
you have to do to care for a puppy. At the end of the paragraph the author says, "This is a lot of work." These ideas are presented in
(B), making it the correct choice. The passage does not provide information to support choices (A), (C), and (D). Therefore they are
incorrect.
2) D
behave (verb): to act.
The question asks us to find the best synonym. Synonyms are words that have nearly the same meanings. In paragraph 2, the
author says, "When you get a puppy, you have to teach the puppy how to behave." The author continues to talk about how the
puppy goes to the bathroom, jumps on guests, chews your shoes, and walks on a leash. These are all ways in which the puppy
acts. This means act is a good synonym for behave. Therefore (D) is correct. The passage does not contain information to support
choices (A), (B), and (C). Therefore they are incorrect.
3) D
In paragraph 2, the author describes the things you will have to teach your puppy in order to make it behave properly. At the end of
this paragraph, the author says, "This is a lot of work." Using this information, we can understand that the main idea of paragraph 2
is to explain how puppies are a lot of work. Choice (D) is correct. Paragraph 2 does describe ways in which puppies are immature
and destructive, but this is not the main purpose. These are details used to support the idea that puppies are a lot of work. This
means (A) and (C) are incorrect. (B) is incorrect because the author does not mean to say that puppies do not make good pets. We
know this because the author says "puppies are cute, friendly, and very playful" in the first paragraph.
4) A
housebroken (adjective): trained to avoid excreting inside the house or in improper places.
In paragraph 2 the author says that you have to make sure that the puppy is housebroken so that it does not use the bathroom
inside of your house. Using this information, we can understand that if a dog is housebroken, then it will go outside to use the
bathroom. Choice (A) is correct. The passage does not provide information to support choices (B), (C), and (D). Therefore they are
incorrect.
5) B
In paragraph 2, the author says you need to teach puppies how to do things that adult dogs already know how to do. The author
tells us that you "have to train the puppy to walk on a leash." This supports option (I). In paragraph 4, the author says, "Puppies
also have a lot of energy and want to play all of the time." On the other hand, in paragraph 5 the author says that adult dogs "sleep
when you are sleeping and are happy to watch television on the couch right beside you." This lets us know that the author thinks
that adult dogs have less energy than puppies do. This supports option (II). In the final paragraph, the author tells us that "many
adult dogs spend a lot of time in the pound, and some never find good homes." This lets us know that it is harder for adult dogs to
find a home than it is for puppies However, this is not meant to be a reason why adult dogs are easier to take care of than puppies.
This eliminates option (III). Therefore (B) is correct.
6) C
In paragraph 5, the author says, "most adult dogs will wait on you to play." This is not meant to say that adult dogs do not like to
play. The author says this simply to contrast adult dogs with puppies, as described in paragraph 4. This makes (C) false, and
therefore the correct choice. In paragraph 4, the author says, "Puppies also have a lot of energy and want to play all of the time."
This means (A) is incorrect. Also in paragraph 4, the author says, "Puppies will not always sleep through the night or let you relax as
you watch television." Since puppies don't sleep through the night and don't let you relax, it can be understood that they need a lot
of attention. So (B) is incorrect. The passage does not provide information to support choice (D). Therefore it is incorrect.
7) D
relax (verb): to rest, especially after doing work.
The question asks us to find the best synonym. Synonyms are words that have nearly the same meanings. In paragraph 3 the
author says that puppies have a lot of energy and always want to play. They will not let you relax as you watch television. Given
that people typically want to rest while watching television, we can understand that rest is a good synonym for relax. Choice (D) is
correct. Work and play are not synonyms. Rather, they are antonyms (words that have the opposite meaning). Therefore (A) and
(C) are incorrect. The passage does not provide information to support choice (B). Therefore it is incorrect.
8) B
To answer this question correctly, it helps to use context. In paragraphs 2 and 4, the author gives reasons why puppies are more
work to teach than adult dogs. In paragraphs 3 and 5, the author gives reasons why adult dogs are easier to teach than puppies.
Given that paragraphs 2 and 4 come previous to, or before, paragraphs 3 and 5, and that they supply contrary information, we can
understand that this phrase is used to contrast previous information. Choice (B) is correct. The passage does not provide
information to support choices (A), (C), and (D). Therefore they are incorrect.
9) C
In paragraph 1, the author talks about how many people want to get a pet puppy because they "are cute, friendly, and playful." The
author spends the rest of the passage describing how in reality, puppies take a lot of work before they learn to behave properly.
Using this information, we can understand that the author believes people think that puppies are cute and playful and do not always
think about how much work it will take to train them. This means (C) is correct. The author does not talk about how people see adult
dogs as unhappy and dangerous. This means choice (A) is incorrect. (B) is incorrect because the author describes how adult dogs

Copyright Read Theory LLC, 2012. All rights reserved.

do not need much training. Rather, the author describes how puppies need a lot of training. While it may be true that adult dogs do
not have much time left to live, this is not a reason the author gives for why puppies are more likely to find good homes. Therefore
(D) is incorrect.
10) D
In paragraph 1, the author writes, "But even though puppies make good pets, there are good reasons why you should consider
getting an adult dog instead." The author spends the rest of the passage telling us how much care a puppy needs. According to the
author, it takes "a lot of work" to teach a puppy how to behave. Using this information, we can understand that someone who owns a
puppy must be very responsible. Choice (D) is correct. Although in paragraph 2 the author says that "When you get a puppy, you
have to teach it how to behave," this does not necessarily mean that you have to be strict, or severe, with it. Therefore (A) is
incorrect. The author lets us know that someone who owns a puppy must be responsible. This does not mean that someone must
also be serious, however. You can be humorous or happy and still be responsible. (B) is incorrect. Although you have to "do a lot of
work" to teach a puppy how to behave properly, you do not necessarily have to be careful to do this. This makes (C) incorrect.

Copyright Read Theory LLC, 2012. All rights reserved.

READTHEORY
Name________________
Date________________

Reading Comprehension 4

Level 5

Directions: Read the passage. Then answer the questions below.

Many people like to eat pizza, but not everyone knows how to make it. Making the perfect pizza
can be complicated, but there are lots of ways for you to make a more basic version at home.
When you make pizza, you must begin with the crust. The crust can be hard to make. If you want
to make the crust yourself, you will have to make dough using flour, water, and yeast. You will have to
knead the dough with your hands. If you do not have enough time to do this, you can use a prepared
crust that you buy from the store.
After you have chosen your crust, you must then add the sauce. Making your own sauce from
scratch can take a long time. You have to buy tomatoes, peel them, and then cook them with spices. If
this sounds like too much work, you can also purchase jarred sauce from the store. Many jarred sauces
taste almost as good as the kind you make at home.
Now that you have your crust and your sauce, you need to add the cheese. Cheese comes from
milk, which comes from cows. Do you have a cow in your backyard? Do you know how to milk the cow?
Do you know how to turn that milk into cheese? If not, you might want to buy cheese from the grocery
store instead of making it yourself.
When you have the crust, sauce, and cheese ready, you can add other toppings. Some people
like to put meat on their pizza, while other people like to add vegetables. Some people even like to add
pineapple! The best part of making a pizza at home is that you can customize it by adding your own
favorite ingredients.

Questions

1) The author's main purpose in writing this passage is to


A.
B.
C.
D.

describe the history of pizza


teach a healthier way to make pizza
outline steps to make a basic pizza at home
provide tips about how to make your pizza especially delicious

2) As used in paragraph 1, which word means the opposite of complicated?


A.
B.
C.
D.

difficult
simple
easy
manageable

3) As used in paragraph 3, which is the best synonym for purchase?


A.
B.
C.
D.

forget
buy
ask
cook

Copyright Read Theory LLC, 2012. All rights reserved.

4) In paragraph 3, the author writes, "Many jarred sauces taste almost as good as the kind you make at
home." The purpose of this statement is to
A.
B.
C.
D.

clarify a later statement


provide an example
clarify an earlier statement
support the previous paragraph

5) In paragraph 4, the author asks a series of questions in order to


A.
B.
C.
D.

support the idea that most people cannot make homemade cheese
reinforce the idea that most people probably live on farms
prove that store-bought cheese tastes better than homemade cheese
emphasize the superiority of homemade cheese over store bought cheese

6) As used in paragraph 5, which is the best definition for customize?


A.
B.
C.
D.

to make personal
to prepare for more than one
to eat while hot
to desire

7) According to the author, which of the following ingredients do you need to have ready before you can
add the toppings?
I. crust
II. sauce
III. cheese
A.
B.
C.
D.

I only
I and II only
II and III only
I, II, and III

8) Which of the following words best describes how the author feels about making a pizza from scratch?
A.
B.
C.
D.

helpful
understanding
enthusiastic
negative

9) Which of the following conclusions would work best at the end of this passage.
A. Although the crust, sauce, and toppings are all important ingredients in pizza, it is clear that the
cheese is most important. Therefore, be sure your cheese is homemade.
B. It can be understood that making your pizza from scratch should be avoided at all costs. Use
store bought ingredients and save yourself a heap of trouble.
C. As you can see, cooking a pizza can be fun, but it can also be very expensive. But, as you can
see, the best things are worth paying for.
D. Once you have prepared the crust, sauce, cheese, and toppings, you are ready to bake your
pizza. I think you will see that making pizza at home can be a good alternative to purchasing it
from the store.

Copyright Read Theory LLC, 2012. All rights reserved.

Answers and Explanations


1) C
In the first paragraph, the author introduces the main idea. The author says, "Making the perfect pizza can be complicated, but there
are lots of ways for you to make a more basic version at home." Here, the author tells us that he or she wants to give us some ways
to make a more basic pizza at home. The author spends the rest of the passage outlining the steps we need to take to make the
pizza. This means (C) is correct. The passage does not provide information to support choices (A), (B), and (D). Therefore they are
incorrect.
2) B
complicated (adjective): difficult to explain, understand, or analyze.
In paragraph 1, the author says, Making the perfect pizza can be complicated, but there are lots of ways for you to make a more
basic version at home. Given that making pizza at home can be complicated or difficult, the author wants to explain a way that is
more basic. So, the author wants to explain a way to make pizza that is basic, or not complicated. Simple is the only choice that
means something similar to basic, or not complicated. Therefore (B) is correct. The passage does not provide information to support
choices (A), (C), and (D). Therefore they are incorrect.
3) B
purchase (verb): to acquire through the payment of money or its equivalent; to buy.
The question asks us to find the best synonym. Synonyms are words that have nearly the same meanings. The author tells us that if
making homemade tomato sauce sounds like too much work, we can purchase jarred sauce from the store. Using context, we can
understand that another word for purchase is buy. If you are unfamiliar with the word purchase, you might still be able to guess
that people usually go to a store in order to buy things. Choice (B) is correct. The passage does not provide information to support
choices (A), (C), and (D). Therefore they are incorrect.
4) C
To answer this question correctly, it helps to use context. At the end of paragraph 3 the author writes, "If this sounds like too much
work, you can also purchase jarred sauce from the store. Many jarred sauces taste almost as good as the kind you make at home."
The author tells us that as a substitute for something that takes too much work, we can purchase jarred sauce. This may seem like a
poor substitute, but the author explains that this is not necessarily the case; many jarred sauces taste almost as good as the kind
you make at home. Using this information, we can understand that the purpose of the statement in question is to clarify an earlier
statement. Choice (C) is correct. The passage does not provide information to support choices (A), (B), and (D). Therefore they are
incorrect.
5) A
In paragraph 4, the author asks the reader, "Do you have a cow in your backyard? Do you know how to milk the cow? Do you know
how to turn that milk into cheese?" The author thinks that he or she knows the answer to these questions. The author thinks that you
probably do not have a cow in your backyard, do not know how to milk a cow, and do not know how to turn milk in cheese. After all,
not many people would answer these questions positively. Given that the author expects that most people would not answer
positively to these questions, then it is fair to assume that most people are unable to make home-made cheese. The authors
question is intended to be rhetorical, or perhaps even sarcastic. This means choice (A) is correct. The passage does not provide
information to support choices (B), (C), and (D). Therefore they are incorrect.
6) A
customize (verb): to build according to personal preferences or tastes.
In order to answer this question correctly, it helps to use context. In paragraph 5, the author says, The best part of making a pizza
at home is that you can customize it by adding your favorite ingredients. After the author says this, he or she continues to give
examples: some people like to add vegetables, meat, or even pineapple. Since the author tells us that different people like different
things on their pizza, we can understand that the choice of toppings is personal, or special for each person. Therefore, you can
customize things to make them personal. This lets us know that (A) is correct. The passage does not provide information to support
choices (B), (C), and (D). Therefore they are incorrect.
7) D
In paragraph 2, the author says, "When you make pizza, you must begin with the crust." This comes before the author instructs you
to add the toppings. This supports option (I). In paragraph 3, the author says, "After you have chosen your crust, you must then add
the sauce." This comes before the author instructs you to add the toppings. This supports option (II). In paragraph 4, the author
says, " Now that you have your crust and your sauce, you need to add the cheese." This comes before the author instructs you to
add the toppings. This supports option (III). Therefore (D) is correct.
8) D
In paragraph 2 the author writes, "If you want to make the crust yourself, you will have to make dough using flour, water, and yeast.
You will have to knead the dough with your hands.." In paragraph 3 the author writes, "Making your own sauce from scratch can
take a long time. You have to buy tomatoes, peel them, and then cook them with spices." In paragraph 4 the author writes, "Do you
have a cow in your backyard? Do you know how to milk the cow? Do you know how to turn that milk into cheese?" In all of these
quotations, the author talks about how hard it is to make a pizza from scratch. The author uses negative language to highlight this
difficulty. This lets us know that the author is feels negative about making pizza from scratch. Choice (D) is correct. Choices (A), (B),
and (C) all contain positive words. However, the author is negative about making a pizza from scratch. Therefore these choices are
incorrect.

Copyright Read Theory LLC, 2012. All rights reserved.

9) D
A good conclusion restates the main idea and the basic points (or thesis) of the passage. In paragraph 1 the author introduces the
idea of making a pizza at home. In paragraph 2 the author talks about the crust. In paragraph 3 the author talks about the sauce. In
paragraph 4 the author talks about the cheese. In paragraph 5 the author talks about the toppings. Using this information, we can
understand that (D) is correct. (A) is incorrect because the passage does not provide information to support the idea that cheese is
the most important ingredient in pizza. Although the author does advise against making your own ingredients, this is not the main
idea of the passage. Therefore (B) is incorrect. The author does not talk about the cost of making pizza in this passage. Therefore
(C) is incorrect.

Copyright Read Theory LLC, 2012. All rights reserved.

READTHEORY
Name________________
Date________________

Reading Comprehension 5

Level 5

Directions: Read the passage. Then answer the questions below.


Dan, Sarah, Ted, and Rebecca have been playing a friendly game of poker for almost an hour.
So far, Ted has the biggest pile of winnings, with a total of $5.45. Sarah has won a small sum, and
Rebecca has lost more than Dan.

Questions
1) At this stage of the game, who has lost the most money?
A.
B.
C.
D.

Ted
Sarah
Dan
Rebecca

2) How much money might each player have started with, if each player started with the same amount?
A.
B.
C.
D.

$4
$6
$8
Not enough information is provided

3) If the game started at 7:00, what time might it be at this stage of the game?
A.
B.
C.
D.

7:30
7:50
8:00
8:45

4) At this stage of the game, who has/have won money?


I. Ted
II. Sarah
III. Dan
A.
B.
C.
D.

I only
I and II only
II and III only
I, II, and III

Copyright Read Theory LLC, 2012. All rights reserved.

Answers and Explanations


1) D
The passage tells us that "Rebecca has lost more than Dan." This lets us know that Rebecca and Dan have lost money. Since
Rebecca has lost more than Dan, we know that she has lost the most money. This means choice (D) is correct. The passage tells
us that Ted "has the biggest pile of winnings," and that "Sarah has won a small sum." This lets us know that Ted and Sarah have
won money. This eliminates choices (A) and (B). The passage also tells us that "Rebecca has lost more than Dan." Choice (C) is
incorrect.
2) A
The passage says that, "Ted has the biggest pile of winnings, with a total of $5.45." Since Ted has the biggest pile of winnings, and
each player started with the same amount, he must have started with less than $5.45. Answer (A) is correct. The values in choice
(B), (C), and (D) are all higher than $5.45. Therefore they are incorrect.
3) B
The passage says that "Dan, Sarah, Ted, and Rebecca have been playing a friendly game of poker for almost an hour." If the game
started at 7:00, and has been going on for almost an hour, then the only time that makes sense is 7:50. Therefore (B) is correct. The
passage does not provide information to support choice (A), (C), or (D). Therefore they are incorrect.
4) B
We know that Ted has won money, because the passage tells us that he "has the biggest pile of winnings." This supports option (I).
We also know that Sarah has won money, because the passage says that Sarah has "won a small sum." This supports option (II).
Finally, we know that Dan has not won money. This is because the passage says that "Rebecca has lost more than Dan." If
Rebecca has lost more than Dan, this implies that Dan has also lost money in addition to Rebecca. This eliminates option (III).
Therefore (B) is correct.

Copyright Read Theory LLC, 2012. All rights reserved.

ReadTheory.Org 2010

Name________________
Date________________

EnglishForEveryone.Org 2008

A Cold Day
Reading Comprehension Short Stories

Directions: Read the story. Then answer the questions below.

It is a frigid January day in York, Pennsylvania.


The temperature is below freezing. Snow is starting to fall.
Dr. James turns on the television to check the weather.
He must leave for his work at the hospital soon.
Today is going to be very cold, says the TV weatherman. Be sure to wear very
warm clothes when you go outside. Also, be careful driving on the roads.
Snowfall will make them slippery. In fact, if you can stay home today, do it!
Dr. James cannot stay home. Very sick people are waiting to see him at the
hospital.
He goes to his closet. He takes out the warmest clothes he has.
He puts on a sweater, jacket, gloves, socks, boots, and a hat.
He opens his front door to go to work. A gust of cold air blows inside.
Wow, it is very cold outside, Dr. James says. He is from Miami and is not used
to the cold. The weatherman was right!
Before he can drive to work, Dr. James must clear the snow off his car.
He does this very fast. He hops in the car. He shivers. His neck feels especially
cold.
Dr. James drives slowly to work. Everyone else is driving slowly, too. There is a
lot of traffic on the road. There are cars in front of and behind him.
Suddenly, the cars in front of Dr. James come to a stop. There has been
an accident!
Dr. James hurries from his car to check on the driver of the car that has swerved
off the road. Is everyone okay? Dr. James asks.
Yes, yes, we are fine. We slipped on a patch of ice, the driver says. This would
have been a good day to stay home in bed.

Questions:
1) If the weather is frigid, it is very
A.
B.
C.
D.

cold
rainy
windy
slippery

3) Dr. James doesnt stay home


because
A. There are sick people waiting for
him.
B. He listens to the weatherman.
C. He has to clear snow off his car.
D. He must help the people in the
accident.

5) What does Dr. James do before he


leaves for work?
I. He dresses warmly.
II. He checks the weather on TV.
III. He clears snow from his car.
A.
B.
C.
D.

I only
I and II
II and III
I, II, and III

7) How are people driving today?


A.
B.
C.
D.

badly
slowly
very fast
like they do not care

2) In what city does this story take


place?
A.
B.
C.
D.

Florida
Miami
Pennsylvania
York

4) What could Dr. James have put on


his neck to keep it warm?
A.
B.
C.
D.

gloves
a scarf
another hat
a jacket

6) Given what is said in the story, what


is probably true about Miami?
A.
B.
C.
D.

It is a warm place.
It is a rainy place.
It is a windy place.
People drive slowly there.

8) As described in the beginning of the


story, what does it mean if the
roads are slippery?
A. The roads are full of cars.
B. The roads are easy to slide on.
C. The roads are very long and
curvy.
D. The roads lead to a place with
lots of snow.

Questions (continued):
9) Why does Dr. James clear the snow 10) There is a lot of traffic on the
off his car quickly?
road. How can we rewrite this
sentence?
A. because he is very cold
B. because he is late for work
A. There is a lot of snow on the
C. because he knows his patients
road.
are waiting
B. There are a lot of cars on the
D. because he knows he will have
road.
to drive slowly
C. There are a lot of accidents on
the road.
D. There are a lot of people walking
on the road.

11) Why did the car have an accident?


A.
B.
C.
D.

because it was snowing outside


because Dr. James hit the car
because it was so cold outside
because the car slipped on a
patch of ice

13) What is the weatherman right


about?
I. It is a windy day.
II. It is very cold outside.
III. The roads are slippery.
A.
B.
C.
D.

I only
I and II
II and III
I, II, and III

12) What is an accident?


A. something that hurts people
B. something that happens only in
the snow
C. something that happens that has
not been planned
D. something that happens
because other people want it to

14) Why does Dr. James hurry from his


car after the accident?
A. because he is very cold
B. because he wants to get to the
hospital as soon as possible
C. because he is worried about his
patients waiting at the hospital
D. because he wants to make sure
the people in the car are okay

Do you like the weather where you live? Why or why not?
________________________________________________________________________
________________________________________________________________________
________________________________________________________________________
________________________________________________________________________

Answers and Explanations


1) A
frigid (adjective): extremely cold.
The first line of the story tells the reader that it is a frigid day: The
temperature is below freezing. Snow is starting to fall. We can understand
from this that frigid means very cold. Therefore (A) is correct.
Since snow is starting to fall, we know that it is not rainy. Therefore (B) is
incorrect. A windy day may or may not be freezing and snowy, so frigid
cannot mean windy. Therefore (C) is incorrect. It may or may not be slippery
out when it is below freezing, so frigid cannot mean slippery. Therefore (D) is
incorrect.
2) D
The first line of the story says, It is a frigid January day in York,
Pennsylvania. Therefore (D) is correct. Florida is a state, not a city.
Therefore (A) is incorrect. Although we learn later in the story that Dr. James
is from Miami, that is not where the story takes place. Therefore (B) is
incorrect. Pennsylvania is a state, not a city. Therefore (C) is incorrect.
3) A
In the middle, the story says, Dr. James cannot stay home. Very sick people
are waiting to see him at the hospital. Dr. James cannot stay at home
because there are sick people waiting for him. Therefore (A) is correct.
Dr. James does listen to the weatherman, but that is not the reason why he
cannot stay home. Therefore (B) is incorrect. He also clears snow off his car,
but that is not the reason why he cannot stay home. Therefore (C) is
incorrect. The accident is not the reason why Dr. James cannot stay home.
Therefore (D) is incorrect.
4) B
A scarf is the item of clothing used to keep the neck warm. It is also the only
item of clothing Dr. James is not already wearing. Wearing a scarf would keep
his neck warm. Therefore (B) is correct.
Gloves go on hands, and Dr. James is already wearing gloves. Therefore (A)
is incorrect. Hats keep heads warm, and another hat will not keep his neck
warm. He is already wearing a hat. Therefore (C) is incorrect. Jackets keep
the torso and arms warm, and Dr. James is already wearing a jacket.
Therefore (D) is incorrect.
5) D
In the middle of the story, Dr. James takes out the warmest clothes he has.
Dr. James puts on a sweater, jacket, gloves, socks, boots, and a hat. This
supports option (I). At the beginning, the story says, Dr. James turns on the
television to check the weather. This supports option (II). In the middle, the

story says, Dr. James must clear the snow off his car. Dr. James clears the
snow off his car very fast. This supports option (III). Therefore (D) is
correct.
6) A
In the middle, the story says Dr. James is from Miami and is not used to the
cold. We can understand from this that Miami is not cold. Since Miami is not
cold, it is probably warm. Therefore (A) is correct.
The story says that Dr. James is not used to the cold because he is from
Miami. This gives us an idea of how warm it is in Miami, but does not give us
any idea whether Miami is rainy. Therefore (B) is incorrect. It also does not
tell us whether Miami is windy. Therefore (C) is incorrect. The story describes
what the traffic is like in York on one day, but that does not give us any idea
what the traffic is like in Miami. Therefore (D) is incorrect.
7) B
The story says that Dr. James drives slowly, and everyone else is driving
slowly, too. Therefore (B) is correct. The story does not provide information
to support answer choices (A), (C), and (D). Therefore they are incorrect.
8) B
slippery (adjective): causing or tending to cause items to slide or slip.
In the middle of the story, the TV weatherman says, Be careful driving on the
roads. Snowfall will make them slippery. At the end of the story, a driver who
swerved off the road tells Dr. James, We slipped on a patch of ice. Taking
these statements together, we can understand from this that snowfall makes
the roads dangerous because they became easy to slip on. Slippery means
making it easy to slide or slip. Therefore (B) is correct.
The snowfall will not make the roads full of cars. Therefore (A) is incorrect.
The snowfall will not make the roads long or curvy. Therefore (C) is incorrect.
The snowfall does not affect where roads lead. Therefore (D) is incorrect.
9) A
In the middle of the story, Dr. James clears off his car. When Dr. James is
finished clearing off the car, he hops in the car and shivers. Since Dr.
James gets in the car quickly and shivers after clearing off the car, we can
understand that Dr. James cleared the car off quickly because he felt cold.
Therefore (A) is correct.
Dr. James goes to work though the snow has made driving difficult because
he knows that he has patients waiting for him, but that is not why he clears
the snow quickly. Therefore (C) is incorrect. The story does not provide
information to support answer choices (B) and (D). Therefore they are
incorrect.
10) B
traffic (noun): the vehicles coming and going on a street, in a town, etc.

Near the end, the story says, There is a lot of traffic on the road. There are
cars in front of and behind him. We can understand from this that traffic
means there are a lot of cars on the road. Therefore (B) is correct. The story
does not give us information to support answer choices (A), (C), and (D).
Therefore they are incorrect.
11) D
At the end of the story, Dr. James goes up to a car that has swerved off the
road and asks the driver if everyone is okay. The driver says, Yes, yes, we
are fine. We slipped on a patch of ice. We can understand from this that the
car had an accident because it slipped on the ice. Therefore (D) is correct.
While the snow may have made the roads icy, the direct cause of the
accident was the ice itself. Therefore (A) is incorrect. The story does not say
Dr. James hit the car. Therefore (B) is incorrect. Therefore it is incorrect.
Although the cold weather has caused the ice to freeze, the direct cause of
the accident was the ice itself. Therefore (C) is incorrect.
12) C
accident (noun): an unexpected event, especially one that causes harm.
The weather is snowy, which has made the roads slippery. At the end of the
story, the slippery roads cause an accident. The accident is that one of the
cars swerved off the road. The driver swerved off the road because of the ice.
The driver did not plan to swerve off the road. Therefore (C) is correct.
Although people can be hurt in accidents, the people in this car accident were
fine. They were not hurt. Therefore (A) is incorrect. The snow may make
accidents more likely to happen, but accidents can happen without snow.
Therefore (B) is incorrect. Accidents are not planned, as it would not make
sense for the driver to plan to swerve off the road. Therefore (D) is incorrect.
13) C
The weatherman does not say that the day will be windy. This eliminates
option (I). Near the beginning of the story, the weatherman says, Today is
going to be very cold. This supports option (II). The weatherman also says,
Be careful driving on the roads. Snowfall will make them slippery. This
supports option (III). Therefore (C) is correct.
14) D
At the end, the story says, Dr. James hurries from his car to check on the
driver of the car that has swerved off the road. Therefore (D) is correct.
We know that Dr. James is cold. This is why he cleans his car off quickly. But
this is not why Dr. James gets out of his car quickly after the accident.
Therefore (A) is incorrect. We also know that Dr. James wants to get to the
hospital to take care of his patients. This is why Dr. James leaves his house
against the weathermans advice. But this is not the reason that Dr. James
hurries from his car after the accident. This makes (B) incorrect. It is possible
that Dr. James is worried about his patients at the hospital, but the story does
not tell us this is so. Therefore (C) is incorrect.

englishforeveryone.org

10

15

Name________________
Date________________

Beginning Critical Reading Soaps and Detergents

Soaps and detergents are used for washing. Soaps and detergents are used for cleaning.
People usually use detergent to wash clothes. People usually use detergent to wash dishes.
People usually use soap to wash their bodies.
Soap has a long history. There was soap in Ancient Babylon. There was soap in Ancient
Egypt. Some people think there was soap before people wrote history.
Soap and detergent are similar, but soap and detergent are not exactly the same. Soaps are
made of natural products. Detergents are made of man-made products.
In some cases, soap is better than detergent. For example, soap is milder on the skin.
Soap is milder on the environment. Soap is biodegradable natures processes clean soap up.
Soap does not build up in rivers. Soap does not cause pollution in rivers. Soap does not build up
in streams. Soap does not cause pollution in streams.
In some cases, detergent is better than soap. For example, soap builds up in clothes after
many washings. Detergent does not build up in clothes after many washings. Soap loses its
cleaning power in clothes over time. Detergent does not lose its cleaning power in clothes over
time.

Questions
1. According to the passage, which of the following statements is/are true?
I)
Soap is made of natural or man-made products.
II) Detergents are less mild than soaps.
III) Detergent washes out of clothing.
IV)
A)
B)
C)
D)
E)

I only
II only
III only
I and II only
II and III only

2. What is the main idea of the fifth paragraph?


A)
B)
C)
D)
E)

How soaps and detergents are made


What soap and detergent are used for
Which product causes less pollution
How soap loses its cleaning power
When detergent is better than soap

3. In line 9, biodegradable most closely means


A)
B)
C)
D)
E)

man-made
dangerous
permanent
easy to clean up
hard to clean up

Answers and Explanations


1. The correct answer is E.
I)

Incorrect. Line 7 states that Soaps are made of natural products. Detergents are
made of man-made products.
II) Correct. Lines 8-9 state that For example, soap is milder on the skin. Soap is
milder on the environment. Since soap is milder than detergent, detergent is less
mild than soap.
III) Correct. Line 14 states that Detergent does not build up in clothes after many
washings. Since detergent does not build up, or stay, in clothing, it must wash out.
A)
B)
C)
D)
E)

I only
II only
III only
I and II only
II and III only

2. The correct answer is E.


A) Incorrect. Line 7 in the third paragraph explains how soap and detergent are made of
different products, when it states that Soaps are made of natural products. Detergents are
made of man-made products, the fifth paragraph gives examples of ways in which
detergent is sometimes better than soap. For example, the fifth paragraph states that
Soap loses its cleaning power in clothes over time. Detergent does not lose its cleaning
power in clothes over time.
B) Incorrect. While lines 1-2 explain the uses of soaps and detergents when they state that
Soaps and detergents are used for washing. Soaps and detergents are used for cleaning,
the fifth paragraph gives examples of ways in which detergent is sometimes better than
soap. For example, the fifth paragraph states that Soap loses its cleaning power in
clothes over time. Detergent does not lose its cleaning power in clothes over time.
C) Incorrect. While lines 9-11 explain that soap causes less pollution when they state that
Soap is milder on the environment. Soap is biodegradable natures processes clean
soap up. Soap does not build up in rivers. Soap does not cause pollution in rivers. Soap
does not build up in streams. Soap does not cause pollution in streams, the fifth
paragraph gives examples of ways in which detergent is sometimes better than soap. For
example, the fifth paragraph states that Soap loses its cleaning power in clothes over
time. Detergent does not lose its cleaning power in clothes over time.
D) Incorrect. While line 14 states that Soap loses its cleaning power in clothes over time, it
does not tell how soap loses its cleaning power.
E) Correct. The fifth paragraph gives examples of ways in which detergent is sometimes
better than soap. For example, the fifth paragraph states that Soap loses its cleaning
power in clothes over time. Detergent does not lose its cleaning power in clothes over
time.

3. The correct answer is D.

A) Incorrect. Since line 7 states that Soaps are made of natural products, and line 9 states
that Soap is biodegradable, the word biodegradable cannot mean man-made.
B) Incorrect. Since lines 8-9 state that soap is milder on the skin. Soap is milder on the
environment, and line 9 states that Soap is biodegradable, the word biodegradable
cannot mean dangerous.
C) Incorrect. Since lines 9-10 state that natures processes clean soap up, the word
biodegradable cannot mean permanent.
D) Correct. Since lines 9-10 state that natures processes clean soap up, the word
biodegradable probably means easy to clean up.
E) Incorrect. Since lines 9-10 state that natures processes clean soap up, the word
biodegradable probably doesnt mean hard to clean up.

ReadTheory.Org 2010

Name________________
Date________________

EnglishForEveryone.Org 2008

Bullied
Reading Comprehension Short Stories

Directions: Read the story. Then answer the questions below.

Bryan doesn't like going to school anymore.


He is tired of being bullied.
Some of the bigger boys at school are mean to him.
They push him to the ground.
They hide his books.
They call him names.
They are bullies.
When things get really bad, Bryan tells his mom he is sick.
You should stay home and rest, Mom says. You'll feel better tomorrow.
Then Bryan gets to stay home.
He reads books. He watches television. He eats what he wants. He is not scared.
You should tell someone you are getting bullied at school, says Bryans best
friend, Link.
But Bryan is scared. He doesn't want to tell an adult.
That will make things worse, Link, says Bryan. You dont go to my school. You
have no idea how mean these guys can be.
You cant keep missing school, says Link. Your grades will fall. You will be in
trouble at home. And besides, you don't want to be in the seventh grade forever,
do you?"
Bryan thinks Link is probably right. He will tell his mom about the bullies when
she gets home from work tonight.
He will see what happens.

Questions:
1) Why doesn't Bryan like going to
school anymore?
A. The work is hard.
B. The kids are mean.
C. His best friend does not go
there.
D. He wants to read books at
home.

3) Who are the bullies in this story?


A. Bryan and Link
B. the gang members at Bryan's
school
C. the teachers
D. the bigger boys at Bryan's
school

5) What do the mean kids do to Bryan?


I. call him names
II. steal his money
III. hide his backpack
A.
B.
C.
D.

I only
I and II
II and III
I, II, and III

7) Why doesnt Bryan tell his mom he


is being bullied?
A. She might hurt the mean kids.
B. He is scared of what the bullies
will do if he tells an adult.
C. She might talk to the teachers at
the school.
D. She will make Bryan go to
school.

2) How might Bryan feel when the kids


at school are mean to him?
I. angry
II. scared
III. strong
A.
B.
C.
D.

I only
I and II
II and III
I, II, and III

4) What grade is Bryan in?


A.
B.
C.
D.

fourth grade
sixth grade
seventh grade
tenth grade

6) Why hasn't Bryan's mom helped


him?
A.
B.
C.
D.

She is busy at work.


She does not care.
She does not know.
She does not want to.

8) What does Bryan do when he stays


home from school?
A.
B.
C.
D.

does his homework


plays video games
gets scared
reads books

Questions (continued):
9) Who tells Bryan that he should tell
someone about being bullied?
A.
B.
C.
D.

Link
Bryans mom
the school counselor
the school principal

11) When does Bryan plan to tell his


mom about the bullies?
A.
B.
C.
D.

today
tonight
tomorrow
tomorrow night

10) What is likely to happen if Bryan


keeps missing school?
A.
B.
C.
D.

He will get into a fight.


His grades will fall.
Link will stop being his friend.
The bullies will come to his
house.

12) Which of the following adjectives


accurately describe Link?
I. shy
II. caring
III. helpful
A.
B.
C.
D.

I only
I and II
II and III
I, II, and III

13) What does it mean to be bullied?


A. to not have friends at school
B. to have a lot of work
C. to have people say and do mean
things to you
D. to not like school

Have you ever been bullied? Do you know someone who has? Explain.

________________________________________________________________________
________________________________________________________________________
________________________________________________________________________
________________________________________________________________________
________________________________________________________________________
________________________________________________________________________

Answers and Explanations


1) B
At the beginning of the story it says, Bryan doesn't like going to school
anymore. After that, we learn that the reason Bryan doesnt like going to
school is that he is being bullied and that some of the bigger boys at school
are mean to him. Bryan does not like going to school because some of the
kids are mean. This makes (B) correct.
The story does not tell us whether Bryans schoolwork is hard, so (A) is
incorrect. Bryan tells his best friend, Link, You dont go to my school. You
have no idea how mean these guys can be. We know that Bryans best
friend does not go to his school, but this is not the reason why Bryan does not
want to go to school. Therefore (C) is incorrect. In the story, we learn that
when Bryan stays home, he reads books, but that is not the reason why he
stays home. This means (D) is incorrect.
2) B
At the beginning of the story, we learn that some bigger boys at school bully
Bryan. In the middle of the story, Link tells Bryan to tell someone about the
bullying, but Bryan is scared. He doesnt want to tell an adult because Bryan
thinks that telling an adult will make things worse. We can understand from
this that Bryan is scared of the bullies. This supports option (I). The bullies do
many mean things to Bryan: They push him to the ground. They hide his
books. They call him names. When people are pushed around and made fun
of, it is natural for them to feel angry. Since the bullies were mean to Bryan,
Bryan probably feels angry. This supports option (II). We know that Bryan
stays home from school because he doesnt want to be bullied. We also know
that Bryan does not want to tell an adult because he is afraid that the bullying
will get worse. Feeling strong is feeling forceful and brave. Someone who
feels forceful and brave would not hide from what he is scared of. This
eliminates option (III). Therefore (B) is correct.
3) D
Near the beginning of the story it says, Some of the bigger boys at school
are mean to him. Therefore (D) is correct.
At the beginning of the story, we learn that Bryan is the name of the boy who
is being bullied. In the middle of the story, we learn that Link is Bryans best
friend. Therefore, Bryan and Link are not the bullies. This means (A) is
incorrect. The story does not talk about any gang members, so (B) is
incorrect. The story does not mention teachers at all, so (C) is incorrect.
4) C
At the end of the story, Link tells Bryan that if Bryan keeps missing school,
Bryans grades will fall. Link then asks Bryan, You dont want to be in the
seventh grade forever, do you? We can understand from this that if Bryans
grades fall, he wont be able to move up to the next grade and will have to

stay in the grade he is in now. This means that Bryan is in the seventh grade
now. This means (C) is correct.
The story does not contain information to support answer choices (A), (B),
and (D). Therefore they are incorrect.
5) A
At the beginning of the story, the author lists the things that the older boys do
to bully Bryan. One of the things they do is call him names. This supports
option (I). The story does not say that the bullies steal Bryans money. This
eliminates option (II). The story does say that the bullies hide Bryans
books. However, the story does not say that the bullies hide Bryans
backpack. Hiding a backpack, which holds books, is not the same as hiding
the books themselves. This eliminates option (III). Therefore (A) is correct.
6) C
Bryan tells his mom he is sick so that he can stay home and not be bullied.
Bryans friend Link tells him you should tell someone you are being bullied at
school. From these statements, we can understand that Bryan has not told
his mother about the bullying. At the end of the story, we learn that Bryan will
tell his mom about the bullies when she gets home from work. Since Bryan is
planning to tell his mother about the bullying, we can infer that he has not
already told her and so she does not know that Bryan is being bullied. This
makes (C) correct.
The story does not contain information to support answer choices (A), (B),
and (D). Therefore they are incorrect.
7) B
In the middle of the story, Link tells Bryan that he should tell someone about
the bullying. Right after that we learn that Bryan is scared. He doesnt want
to tell an adult. Bryan thinks that telling an adult will make things worse. We
can understand from this that Bryan doesnt want to tell his mother, who is an
adult, about the bullying because he is scared that telling an adult will make
the bullying will get worse. Therefore (B) is correct.
There is no information in the story to make us think that Bryans mother will
hurt the mean kids. This makes (A) incorrect. Although Bryans mother might
talk to the teachers at school, there is nothing in the story that leads us to
believe that Bryan doesnt tell his mother about the bullying because he is
afraid that she will talk to the teachers. This means (C) is incorrect. Bryan tells
his mother that he is sick so that he can stay home and avoid the bullies.
There is no information in the story to make us think that Bryans mother will
definitely make him go to school if he tells her about the bullying. This means
(D) is incorrect.
8) D
In the middle of the story, we learn that when Bryan stays home, Bryan reads
books. This means (D) is correct. The story does not contain information to
support answer choices (A) and (B). Therefore they are incorrect. The story

says that when Bryan stays home, Bryan is not scared. This makes (C)
incorrect.
9) A
In the middle of the story, Bryans best friend, Link, says, You should tell
someone you are getting bullied at school. This means (A) is correct. The
story does not contain information to support answer choices (B), (C), and
(D). Therefore they are incorrect.
10) B
At the end of the story, Bryans friend Link says that Bryan cant keep
missing school, because Bryans grades will fall. Therefore (B) is correct.
The story does not contain information to support answer choices (A), (C),
and (D). Therefore they are incorrect.
11) B
At the end of the story Bryan decides to tell his mom about the bullies when
she gets home from work tonight, so (B) is correct. The story does not
contain information to support answer choices (A), (C), and (D). Therefore
they are incorrect.
12) C
An adjective is a word that is used to describe a thing. Being shy is being
nervous and quiet around other people. Link does not appear nervous or
quiet around other people. This eliminates option (I). In the middle of the
story, Link tells Bryan to tell someone about the bullying. Link also tells Bryan
that Bryan cant keep missing school, or else Bryans grades will fall and
Bryan will be in trouble at home. The story says that Link is Bryans best
friend. Friends try and help each other out because they care about each
other. We can understand from this information that Link is giving Bryan
advice because he cares about what happens to Bryan. He is caring. This
supports option (II). Since Link is trying to help Bryan, he is being helpful.
This supports option (III). Therefore (C) is correct.
13) C
bully (verb): to use strength or influence to harm or intimidate a person
At the beginning of the story, we learn that Bryan is being bullied. The story
says that bigger boys at school are mean to Bryan. The story then lists a
few things that the bigger boys do, like pushing Bryan to the ground, hiding
Bryans books and calling him names. Using this information, we can
understand that bullying is doing and saying mean things to other kids.
Therefore (C) is correct.
Just because someone is bullied does not mean that they do not have a lot of
friends at school. This means (A) is incorrect. There is no information in the
story to make us think that being bullied means having a lot of work, so (B) is
incorrect. Someone who is bullied might not like going school because they
get bullied while they are there. But other kids may not like school for different

reasons. This is different from being bullied, which is when some kids say and
do mean things to you. Therefore (D) is incorrect.

ReadTheory.Org 2010

Name________________
Date________________

EnglishForEveryone.Org 2008

By the Water
Reading Comprehension Short Stories

Directions: Read the story. Then answer the questions below.

I live in a house by the water.


I sit by the water each day.
I take my bag there with me. In it, I pack a book and a blanket. I also carry a chair
and a basket of food.
I walk down my back steps and sit in the same spot along the grass.
When I go out to the water in the morning, I am alone.
I hear the water.
I see the boats.
I feel calm.
It is the part of the day I like best.
Later, some children come to play by the water.
It is afternoon.
I hear them laughing.
I see them play ball.
Nestor, Nestor! the children yell when they see me on the beach.
I wave and smile.
Play ball with us, Nestor! the children shout.
No, thanks, I say. "I am too old to play ball. I walk with a cane and my hands
are no longer good at catching."
I try to read my book, but it is hard with all the noise.
I watch mothers and fathers fish along the shore. I am happy.
I hear the water.
I see the boats.
I eat my lunch.
Later, the sky gets dark.
I gather all of my things and go back to the house.
I get in bed. I hear the water through my open window.
The sound puts me to sleep.

Questions:
1) Where does Nestor live?
A.
B.
C.
D.

on a farm
on a mountain top
in a boat on the water
in a house by the water

3) Which of the following things does


Nestor bring with him to the water?
I. food
II. a blanket
III. a fishing pole
A.
B.
C.
D.

I only
I and II
II and III
I, II, and III

5) What do the children do by the


water?
I. fish
II. laugh
III. play ball
A.
B.
C.
D.

I only
I and II
II and III
I, II, and III

7) What kind of person does Nestor


seem to be?
A.
B.
C.
D.

calm
sad
loud
young

2) What does Nestor do each day?


A.
B.
C.
D.

He sits by the water.


He plays by the water.
He runs by the water.
He swims in the water.

4) Nestor lives in a house by the


water. What is another way to write
this?
A.
B.
C.
D.

in the water
over the water
close to the water
far away from the water

6) When do the children come to the


water?
A.
B.
C.
D.

in the morning
in the afternoon
in the evening
at night

8) What does the children's noise


make it hard for Nestor to do?
A.
B.
C.
D.

eat
catch fish
read
sleep

Questions (continued):
9) Who fishes?
A.
B.
C.
D.

Nestor
the children
mothers and fathers
all of the above

10) Why doesn't Nestor play with the


children?
I. He is too tired.
II. He cannot catch.
III. He feels he is too old.
A.
B.
C.
D.

11) How do Nestor's feelings change


during the story?
A.
B.
C.
D.

A.
B.
C.
D.

12) What does Nestor mean when he


says that the sky gets dark?

from calm to sleepy to happy


from sleepy to calm to happy
from calm to happy to sleepy
from happy to sleepy to calm

13) What is Nestor's favorite part of the


day?

I only
I and II
II and III
I, II, and III

A.
B.
C.
D.

that it is night
that it is raining
that the wind is blowing
that the sky is angry

14) When does Nestor hear the water?

the morning
when he is watching the children
the afternoon
the night

A.
B.
C.
D.

only in the morning


only in the afternoon
only in the night
all the time

15) Nestor says, It is the part of the day I like best. What is another way to write
this sentence, while keeping its original meaning?
A.
B.
C.
D.

I do not like this part of the day.


I love this part of the day.
It is my favorite part of the day.
I like all parts of the day.

What is your favorite part of the day? Why?


________________________________________________________________________
________________________________________________________________________

Answers and Explanations


1) D
In paragraph 1, the speaker says, I live in a house by the water. In
paragraph 8, the speaker of the story describes some children calling to him.
They yell, Nestor, Nestor. This lets us know the speakers name is Nestor.
From this information, we know that Nestor lives in a house by the water, so
(D) is correct.
The story does not provide information to support answer choices (A) and
(B). Therefore they are incorrect. Nestor says, I see the boats, but he does
not live on a boat. This means (C) is incorrect.
2) A
In paragraph 1, Nestor says, I sit by the water each day. Therefore (A) is
correct. In the middle of the story, it says that the children play by the water,
but Nestor does not play with them. This means (B) is incorrect. The story
does not provide information to support answer choices (C) and (D).
Therefore they are incorrect.
3) B
In paragraph 2, we learn that Nestor packs a basket of food. This supports
option (I). In paragraph 2, we learn that Nestor packs a blanket. This
supports option (II). Even though Nestor lives beside the water and he
watches other people fish along the shore, Nestor does not bring a fishing
pole. This eliminates option (III). Therefore (B) is correct.
4) C
In paragraph 1, we learn that Nestor lives by the water. He walks to the
beach. This means he is close to the water. If something is by something, it
is close to it. The following words are all part of the same family, and have
almost the same meaning: close to, by, beside, next to, near. This means (C)
is correct.
Nestors house is not in the waters, so (A) is incorrect. Nestors house is not
over the water, which would mean on top of the water. Therefore (B) is
incorrect. We know Nestors house is not far from the water because Nestor
walks to the water every morning. This makes (D) incorrect.
5) C
In paragraph 10, Nestor watches the mothers and fathers fish. The children
do not fish. This eliminates option (I). In paragraph 7, Nestor hears children
laughing. This supports option (II). Also, Nestor sees the children play
ball. This supports option (III). Therefore (C) is correct.
6) B
The first sentence of paragraph 7 says, Later, some children come to play by
the water. The second sentence states, It is afternoon. This lets us know
that the children come to the water in the afternoon, so (B) is correct. The

story does not provide information to support answer choices (A), (C), and
(D). Therefore they are incorrect.
7) A
In the last sentence of paragraph 5, Nestor says, I feel calm. Therefore (A)
is correct.
We know Nestor is not sad because in paragraph 10 he says, I am happy.
This makes (B) incorrect. He is not loud because he does not make noise in
the story. This means (C) is incorrect. In paragraph 8, Nestor says, I walk
with a cane and my hands are no longer good at catching. This lets us know
that he is not young, so (D) is incorrect.
8) C
In paragraph 9, Nestor says, I try to read my book, but it is hard with all the
noise. This means (C) is correct. The story does not provide information to
support answer choices (A), (B), and (D). Therefore they are incorrect.
9) C
In paragraph 10, Nestor says, I watch the mothers and fathers fish along the
shore. This makes (C) correct. The story does not provide information to
support answer choices (A), (B), and (D). Therefore they are incorrect.
10) C
The story does not say that Nestor is tired. This eliminates option (I). In
paragraph 8, we learn that the children ask Nestor to play ball. Nestor says, I
am too old to play ball. I walk with a cane and my hands are no longer good
at catching. This information tells us he cannot catch. This supports option
(II). These sentences also tell us that he feels he is too old to play. This
supports option (III). Therefore (C) is correct.
11) C
In paragraph 5, Nestor says, I feel calm. Later, in paragraph 10, Nestor
says, I am happy. Finally, in paragraph 11, Nestor goes to bed. He says that
the sound of the water puts him to sleep. We can understand that this
means that he is sleepy. So Nestors feelings change from calm to happy to
sleepy. This means (C) is correct.
The story does not provide information to support answer choices (A), (B),
and (D). Therefore they are incorrect.
12) A
In paragraph 11, Nestor says, The sky gets dark. We know that the time has
passed from morning to afternoon, and now it is night. The sky gets dark at
night, so he means that it is night. This makes (A) correct.
The sky can get dark when it is raining, but the story does not say anything
about rain. Therefore (B) is incorrect. The story does not provide information
to support answer choices (C) and (D). Therefore they are incorrect.

13) A
In paragraph 4, Nestor says he goes out to the water in the morning. In
paragraph 6, he tells us, It is the part of the day I like best. We can
understand that morning is the time of day that Nestor likes best. Another way
to say that you like something best is to say that it is your favorite. Morning is
Nestors favorite time of day, so (A) is correct.
The story does not provide information to support answer choices (B), (C),
and (D). Therefore they are incorrect.
14) D
In the first part of the story, which takes place in the morning, Nestor says, I
hear the water. The second part of the story takes place in the afternoon,
and then Nestor also says, I hear the water. At the end of the story, Nestor
is in bed and can hear the water through his open window. Therefore, Nestor
hears the water in the morning, afternoon, and night. This means (D) is
correct.
Since Nestor hears the water throughout the entire day, it is incorrect to say
he only hears it in the morning, or in the afternoon, or in the night. Therefore
(A), (B), and (C) are incorrect.
15) C
In the middle of the story, Nestor says, It is the part of the day I like best.
Another way to say that you like something best is to say it is your favorite.
Therefore (C) is correct.
Since Nestor likes that part of the day best, we can understand that he does
like it, so (A) is incorrect. Sometimes we love the things that we like the best,
but not always. Nestor does not say he loves this part of the day. He says he
likes it best. This means (B) incorrect. Nestor chooses one part of the day to
like best. Liking something best is different than liking everything. We do not
know if Nestor likes all parts of the day, so (D) is incorrect.

ReadTheory.Org 2010

Name________________
Date________________

EnglishForEveryone.Org 2008

Late
Reading Comprehension Short Stories

Directions: Read the story. Then answer the questions below.

Martin is in a hurry.
He is late to work again.
Martin's boss doesn't like it when he is late. Martin was late last week. His boss
told him not to be late again. He really wasnt joking either. He was serious. I
mean it, Martin remembers him saying.
Martin thinks he might lose his job if he is late again.
The time is now 7:15 am. Martin needs to be at work by 7:30. It takes him 22
minutes to drive to work.
Things dont look good, he says to himself.
Martin runs out of the house. He jumps in his car. He puts the car in reverse. He
backs up without looking.
BOOM!
There is a sound like someone hitting a drum. Martins car jerks
to a stop. He has hit the car parked behind him.
Oh, no! Martin exclaims. He is angry now.
Martin looks at his watch. It is 7:18. He needs to get to work.
He looks around. There is no one on the street. There is no one nearby.
He looks in the parked car. It is empty.
Martin drives off quickly.
He gets to work 10 minutes late.
Martins boss is not around. Thank goodness, he says to himself.
He stops worrying. He sits at his desk to work.
During lunch, Martin goes out to the parking lot. He looks at his car. There is a
big dent in the back.
Then he thinks about the other car the car he hit this morning. "I know that car
is damaged too," he thinks.
He feels guilty.
That was not right, Martin says to himself.
He will see if the car is still outside his house when he gets off work.

Questions:
1) At the beginning of the story, Martin
is in a hurry. What does this mean?
A.
B.
C.
D.

He is running.
He is angry.
He is late.
He is moving fast.

3) Martin remembers when his boss


told him, I mean it. What does this
mean?
A.
B.
C.
D.

The boss is serious.


The boss is angry.
The boss is emotional.
The boss is interested.

5) What time does Martin need to be at


work?
A.
B.
C.
D.

6:30
7:15
7:30
8:15

7) What is the loud noise?


A.
B.
C.
D.

Martin yelling
police sirens sounding
Martin driving off the road
Martin hitting someones car

9) Why doesn't Martin get in trouble


with his boss today?
A.
B.
C.
D.

His boss is not around.


He gets to work on time.
His boss feels sorry for him.
Martin says he is sorry for being
late.

2) Why is Martin in a hurry?


A.
B.
C.
D.

because he is late
because he is running
because he is moving fast
because he had a car accident

4) What does Martin think will happen


if he is late to work again?
A.
B.
C.
D.

He will get a pay cut.


He will need to buy a watch.
He will get fired from his job.
He will need to adjust his
schedule.

6) If Martin leaves the house at 7:15,


what time does he get to work?
A.
B.
C.
D.

7:32
7:37
7:40
7:52

8) Martin checks if there is anyone


nearby. What does nearby mean?
A.
B.
C.
D.

in
next to
close by
far away

10) Who does Martin talk to in the


story?
I. himself
II. his boss
III. his friends
A.
B.
C.
D.

I only
I and II
II and III
I, II, and III

Questions (continued):
11) How do Martin's feelings change
during the story?
A.
B.
C.
D.

from hurried to guilty to angry


from angry to guilty to hurried
from hurried to angry to guilty
from angry to hurried to angry

13) As used at the end of the story,


what does it mean to feel guilty?
A. to feel bad about doing
something
B. to feel worried about doing
something
C. to feel unsure about doing
something
D. to feel like you want to change
something

12) Why did Martin look around after


he hit the car?
A.
B.
C.
D.

He needed help.
He wanted a ride to work.
He needed to check the time.
He wanted to check if anyone
saw him.

14) What might Martin do if the car he


hit is still there after work?
I. He might try to hide the dent.
II. He might try to find the owner.
III. He might leave a note on the
windshield with his phone
number on it.
A.
B.
C.
D.

I only
I and II
II and III
I, II, and III

Do you think Martin is a good employee? Why or why not?

________________________________________________________________
________________________________________________________________
________________________________________________________________
________________________________________________________________
________________________________________________________________
________________________________________________________________
________________________________________________________________
________________________________________________________________
________________________________________________________________

Answers and Explanations


1) D
At the beginning of the story, Martin is in a hurry. In paragraph 7, Martin
runs out of the house. He jumps in the car. People run and jump in order to
move fast. Being in a hurry means moving fast. Therefore (D) is correct.
Martin runs because he is in a hurry. But being in a hurry does not just mean
running. People can be in a hurry and move fast in other ways, such as
driving fast or walking fast. This makes (A) incorrect. In the middle of the
story, it says, He is angry now. He is angry because he hit another car. But
he was in a hurry before he hit the car. So in a hurry does not mean angry.
Therefore (B) is incorrect. In paragraph 2, the story says, Martin is late.
People are sometimes in a hurry because they are late. But in a hurry does
not always mean late. People can be in a hurry for other reasons besides for
being late. For example: Johns favorite team is playing baseball at the park.
He is in a hurry to go see them. In this example, John is in a hurry because
he is excited, not late. This means (C) is incorrect.
2) A
In paragraph 1, it says, Martin is in a hurry. The next paragraph explains,
He is late to work again. Martin is in a hurry because he is late. Therefore
(A) is correct.
Martin is running because he is in a hurry. He is not in a hurry because he is
running. This means (B) is incorrect. Martin is moving fast because he is in a
hurry. He is not in a hurry because he is moving fast. This means (C) is
incorrect. Martin gets in a car accident because he is in a hurry, not the other
way around. This means (D) is incorrect.
3) A
In paragraph 3, Martins boss tells him not to be late again. We also learn
that Martins boss was serious. Martins boss says, I mean it. Because we
already know that Martins boss is serious, we can understand that I mean
it means that the boss is serious. Therefore (A) is correct.
We know that Martins boss does not like it when he is late, but the story
does not say he is angry. Instead, the story says that he was serious and
not joking when he told Martin, I mean it. This makes (B) incorrect. There
is no information in the story to make us think Martins boss is emotional.
This makes (C) incorrect. There is no information in the story to make us
think Martins boss is interested. This makes (D) incorrect.
4) C
In paragraph 4, it says, Martin thinks he might lose his job if he is late again.
Being fired means losing your job. If you get fired, the boss says you cant
work there anymore. Therefore (C) is correct. The story does not contain

information to support answer choices (A), (B), and (D). Therefore they are
incorrect.
5) C
In paragraph 4, the author says, Martin needs to be at work by 7:30 am.
Therefore (C) is correct. The story does not mention the time 6:30. This
makes (A) incorrect. Although the story says the current time is 7:15, this is
not the time Martin needs to be at work. This means (B) is incorrect. The
story does not mention the time 8:15. This makes (D) incorrect.
6) B
In paragraph 5, we learn that it takes Martin 22 minutes to drive to work. The
question tells us that Martin leaves the house at 7:15. In order to find out how
long it takes Martin to drive to work, we need to add the time it takes Martin
to drive to work (22 minutes) to the time he leaves (7:15). So, 7:15 + 22 =
7:37. Martin gets to work at 7:37 if he leaves at 7:15. Therefore (B) is correct.
The story does not contain information to support answer choices (A), (C),
and (D). Therefore they are incorrect.
7) D
Both paragraph 8 and paragraph 9 give examples of loud noises. Paragraph
8 simply says, BOOM! In paragraph 9, Martin hears a sound like someone
hitting a drum. In paragraph 9, the author says that Martin has hit the car
parked behind him. It is clear that hitting someones car caused the loud
noise Martin hears. Therefore (D) is correct.
The story does not contain information to support answer choices (A), (B),
and (C). Therefore they are incorrect.
8) C
In paragraph 12, Martin looks around. He sees that there is no one on the
street. There is no one nearby. Martin is looking to see if anyone is close by
him. Therefore (C) is correct.
It would not make sense to say there is no one in. In is a preposition which
requires an object, such as in the car or in the box. This means (A) is
incorrect. Martin is looking on the entire street, not just right next to him. This
makes (B) incorrect. Martin would not be able to see that there is no one far
away just by looking. Also, he is worried about people near him who might
have seen the accident. He is not worried about people who are far away and
did not see it. This makes (D) incorrect.
9) A
In paragraph 16 the story says, Martins boss is not around. Martin does not
get in trouble with his boss because his boss is not around. Therefore (A)
is correct. The story does not contain information to support answer choices
(B), (C), and (D). Therefore they are incorrect.

10) A
In paragraph 16, Martin says, Thank goodness. He says this to himself. In
paragraph 21, the author writes, That was not right, Martin says to himself.
Both these paragraphs show Martin taking to himself. This supports option
(I). Martin does not talk to his boss. This eliminates option (II). He also does
not talk to his friends. This eliminates option (III). Therefore (A) is correct.
11) C
At the beginning of the story, in paragraph 1, Martin is in a hurry. In the
middle of the story, in paragraph 10, Martin is angry after he hits the car.
By the end of the story, in paragraph 20, Martin feels guilty. Martin goes
from hurried to angry to guilty. Therefore (C) is correct. The story does not
contain information to support answer choices (A), (B), and (D). Therefore
they are incorrect.
12) D
Paragraph 12 says that Martin looks around but there is no one on the
street and no one nearby. He is trying to find out whether or not anyone
saw him hit the other car. When he realizes that no one is around, he drives
off without reporting the accident. Therefore (D) is correct.
Martin does not need help, so (A) is incorrect. He has a car, so he does not
need a ride to work. This means (B) is incorrect. He has a watch, so he does
not need to ask anyone the time. This means (C) is incorrect.
13) A
guilty (adjective): responsible for doing something wrong.
In paragraph 19, Martin thinks about the damage he did when he hit the other
car. In paragraph 20, it says, He feels guilty. In paragraph 21, Martin thinks,
That was not right. He thinks what he did was wrong, and he feels bad
about it. Using this information we can understand that feeling guilty means
feeling bad about doing something wrong. Therefore (A) is correct.
Martin was worried about being late to work, but once he gets there, he
stops worrying. This means (B) is incorrect. Martin does not feel unsure at
the end of the story. He knows what he did was not right. This makes (C)
incorrect. Martin might wish to change what he did, but the story does not tell
us that information. Wanting to change something is feeling regretful, not
guilty. This means (D) is incorrect.
14) C
In paragraph 20, Martin feels guilty because he damaged the other car. He
does not want to hide the damage. He wants to take responsibility for what
he did. This eliminates option (I). Because Martin feels guilty, he wants to tell
the owner that he hit his or her car. Martin could do this by trying to find the
owner. This supports option (II). He could also do this by leaving a note on
the windshield with his phone number on it. This supports option (III).
Therefore (C) is correct.

ReadTheory.Org 2010

Name________________
Date________________

EnglishForEveryone.Org 2008

The Brenners
Reading Comprehension Short Stories

Directions: Read the story. Then answer the questions below.

The Brenner family has a mom, a dad, three boys, two dogs, a hamster, and four
frogs.
They are very happy.
Their house is very small.
Mom and Dad share one bedroom.
The three boys share one bedroom. Their names are Billy, Bobby, and Brad.
All the pets sleep in the living room.
Mr. and Mrs. Brenner came home today from the doctor with news. It is very big
news.
There are going to be two more of us, Mrs. Brenner says with a smile.
I am pregnant.
Billy, Bobby, and Brad cheer. We will have a basketball team, Billy says.
And do you know what Dad and I have decided we need? asks Mrs. Brenner.
A coach? asks Bobby.
No, Mr. Brenner says with a laugh. A bigger house.
Over the next week, the Brenner family starts looking for a new house.
On Monday, they see a brick house with a big front porch. It has three bedrooms.
Mr. Brenner likes this one.
On Thursday, they see a wooden house with a big backyard. It has four
bedrooms. Mrs. Brenner thinks this house is great.
On Saturday, they see a house with four bedrooms and a pool. The boys like this
house a lot. They dream of having their friends over for a swim.
But its smaller than the house we live in now! Mrs. Brenner says.
All of the houses cost a lot of money.
I think we need to save more money before we buy a bigger house, says Mr.
Brenner. Mrs. Brenner agrees.
Soon, two new babies arrive in their very small house. Their names are Brandi
and Brooke.
The two baby girls sleep in a crib by their parents bed.
Mr. Brenner buys a basketball hoop and puts it in the driveway.

Questions:
1) What is the Brenner house like?
I. crowded
II. small
III. dirty
A.
B.
C.
D.

I only
I and II
II and III
I, II, and III

3) What pets do the Brenners have?


I. two dogs
II. a hamster
III. four cats
A.
B.
C.
D.

I only
I and II
II and III
I, II, and III

5) What can be said about all the kids'


names?
A.
B.
C.
D.

They are all girls names.


They all start with the letter B.
They are all long names.
They all end in the letter Y.

7) Why does Bobby say the Brenners


need a coach?
A. because Mr. Brenner is too busy
to coach
B. because they need help buying
a house
C. because they will soon have
enough kids for a basketball
team
D. because Mrs. Brenner will not
be able to coach while she is
pregnant

2) Who shares a bedroom?


I. Billy and Mom
II. Mom and Dad
III. Billy, Bobby, and Brad
A.
B.
C.
D.

I only
I and II
II and III
I, II, and III

4) How many dogs do the Brenners


have?
A.
B.
C.
D.

one
two
four
five

6) As used in the beginning of the


story, what is the big news?
A. Mrs. Brenner is pregnant with
twins.
B. The Brenners are moving to a
new house.
C. The Brenners are starting a
basketball team.
D. The Brenners are getting some
new pets.

8) Which house does Mr. Brenner


like?
A.
B.
C.
D.

the one with a pool


the one with a big backyard
the one with a big front porch
the one with four bedrooms

Questions (continued):
9) Which house does Mrs. Brenner
like?
I. the one with a big front porch
II. the one with a big backyard
III. the one with four bedrooms
A.
B.
C.
D.

I only
I and II
II and III
I, II, and III

11) How are all the houses they see


alike?
A.
B.
C.
D.

They are all very pretty.


They are all very big.
They each have four bedrooms.
They all cost a lot of money.

13) What does Mr. Brenner put in the


driveway?
A.
B.
C.
D.

a car
a dog crate
their new van
a basketball hoop

10) Why do the boys like the house


with the pool?
A.
B.
C.
D.

It has the biggest yard.


It is the biggest house.
They are on the swim team.
They want to have their friends
over to swim.

12) What must Mr. and Mrs. Brenner


do before buying a new house?
A.
B.
C.
D.

get new jobs


have the babies
save more money
move to a new city

14) What do the Brenners newborn


babies have in common?
I. They have the same gender.
II. They sleep in the same place.
III. Both their first and last names
start with the same letter.
A.
B.
C.
D.

I only
I and II
II and III
I, II, and III

How big is your family? Do you like this size? Why or why not?
________________________________________________________________
________________________________________________________________
________________________________________________________________
________________________________________________________________
________________________________________________________________

Answers and Explanations


1) B
The story says that three children and two parents live in the house, sharing
just two bedrooms. The family is looking for a bigger house because there
are going to be two more of us. We can guess from this that the house is
crowded. This supports option (I). At the beginning of the story it says that
the house is very small. This supports option (II). There is nothing in the
story that tells us that the house is dirty. This eliminates option (III).
Therefore (B) is the correct answer.
2) C
Because Mom and Dad share one bedroom and the three boys share the
other, we know that Billy and Mom do not share a bedroom. This eliminates
option (I). At the beginning of the story it says, Mom and Dad share one
bedroom. This supports option (II). It also says, The three boys share one
bedroom. This supports option (III). Therefore (C) is the correct answer.
3) B
The first sentence says that the Brenner family has two dogs, a hamster, and
four frogs. This supports option (I) and option (II). The story does not say
that the Brenners have any cats. This eliminates option (III). Therefore (B) is
correct.
4) B
The first sentence says that the Brenner family has two dogs. This means
(B) is correct. The story does not contain information to support answer
choices (A), (C), and (D). Therefore they are incorrect.
5) B
Billy, Bobby, and Brad all start with the letter B. Brandi and Brooke also start
with the letter B. Therefore (B) is the correct answer. We know that the three
oldest children are boys, so (A) is incorrect. Brad is only four letters, which is
not long. The other names are either five or six letters. This means they are
not long, so (C) is incorrect. Only Billy and Bobby end with the letter Y, so (D)
is incorrect.
6) A
In the beginning of the story, right after we learn that Mr. and Mrs. Brenner
have big news, Mrs. Brenner says, I am pregnant. Later we learn there will
be two babies, which means she is having twins. Therefore (A) is the correct
answer.
Although the Brenners start to look for a new house after they announce the
news, they only want a new house because they are expecting two new
babies, so (B) is incorrect. While the kids are excited that they will soon have
enough kids for a basketball team, they are not starting a basketball team, so
(C) is also incorrect. The story does not say the Brenners are getting some
new pets. Therefore (D) is incorrect.

7) C
Basketball teams have five people. The Brenner family has three children, so
two more children in the family would make fiveenough children for a
basketball team. That is why Billy says We will have a basketball team, right
after Mrs. Brenner tells the boys that there will be two more children in the
family. Basketball teams usually have coaches. That is why Bobby says the
Brenners need a coach. Therefore (C) is the correct answer.
The story does not contain information to support answer choices (A), (B),
and (D). Therefore they are incorrect.
8) C
In the middle of the story, we learn that the family sees a brick house with a
big front porch. The house has three bedrooms. Mr. Brenner likes this one.
This means (C) is the correct answer.
The boys like the house with the pool, not Mr. Brenner, so (A) is incorrect.
Mrs. Brenner likes the house with the big backyard, not Mr. Brenner. This
means (B) is incorrect. Two of the houses have four bedrooms, but the one
Mr. Brenner likes has three. Therefore (D) is incorrect.
9) C
In the middle of the story, we learn that the house with a big front porch is
the one that Mr. Brenner likes, not Mrs. Brenner. This eliminates option (I). In
the middle of the story, it says that the family sees a wooden house with a
big backyard. It has four bedrooms. Mrs. Brenner thinks this house is great.
We can understand from this information that the house Mrs. Brenner likes
has a big backyard. This supports option (II). We can also understand from
this information that the house Mrs. Brenner likes has four bedrooms. This
supports option (III). Therefore (C) is the correct answer.
10) D
In the middle of the story, it says that the boys like the house with the pool a
lot and that the boys dream of having their friends over for a swim.
Therefore (D) is the correct answer. The story does not contain information to
support answer choices (A), (B), and (C). Therefore they are incorrect.
11) D
At the end of the story it says, All of the houses cost a lot of money.
Therefore (D) is correct. We do not know if any of the houses are pretty, so
(A) is incorrect. It says that the house with the pool is smaller than the house
they live in now, so (B) is incorrect. It says that the house with the big front
porch has three bedrooms, so (C) is incorrect.
12) C
At the end of the story, Mr. Brenner says, I think we need to save more
money before we buy a bigger house. Therefore (C) is the correct answer.
The story does not contain information to support answer choices (A), (B),
and (D). Therefore they are incorrect.

13) D
The last sentence in the story says, Mr. Brenner buys a basketball hoop and
puts it in the driveway. This means (D) is the correct answer. The story does
not contain information to support answer choices (A), (B), and (C). Therefore
they are incorrect.
14) D
At the end of the story, we learn that the two baby girls sleep in a crib by their
parents bed. We can understand from this that both the babies are girls.
This supports option (I). We can also understand from this sentence that they
both sleep in the same crib. This supports option (II). The story also says that
the babies are named Brandi and Brooke. We also know that the babies last
name is Brenner. This lets us know that their first and last names start with
the same letter. This supports option (III). Therefore (D) is correct.

ReadTheory.Org 2010

Name________________
Date________________

EnglishForEveryone.Org 2008

The New School


Reading Comprehension Short Stories

Directions: Read the story. Then answer the questions below.

Tracey is starting a new school today.


She is very sad. She is very scared.
I dont want to go to school today, Tracey tells her dad.
I understand, sweetheart, Dad says. Starting a new school can be very scary.
Tracey has moved to a new town. She has moved to a new house. She is
starting a new school today. She has done all of this in a week!
I feel sick, Tracey says. My stomach hurts. I cant eat breakfast.
I think that is because you are nervous, Dad says. He pats Traceys hair. He
gives her a little hug. Try drinking just a little juice. Then I will walk you to
school.
Tracey and her dad walk to school.
Tracey thinks about many things.
Will I make friends?
Will I like my teacher?
What if I dont know the answer to a question?
Will kids laugh at me?
What if no one likes me?
Were here, says Dad.
Tracey looks up at the big building. Her other school was small.
Tracey wishes she could run away.
She knows she cannot.
She takes a deep breath.
She walks up the steps to school.
She walks into her third grade classroom.
That must be Tracey, she hears a boy say.
Hello, Tracey!
Welcome, Tracey!
Let me show you around.

Everyone seems kind. Tracey feels a little better.


Bu she is still not happy. She is still a little scared. She cannot eat her lunch.
Dad picks Tracey up after school.
How was your day? he asks.
Okay, she says.
It will get better, Dad says. Big changes are hard.
I know, says Tracey. She reaches for her dads hand to hold as they walk
home.

Questions:
1) Why is Tracey sad and
scared?
A. She is moving to a new
house today.
B. She is moving to a new
town today.
C. She is starting a new
school today.
D. She is walking to school
alone today.

3) What grade is Tracey in?


A.
B.
C.
D.

second grade
third grade
fourth grade
fifth grade

5) Why does Tracey's dad think


she feels sick?
A.
B.
C.
D.

because she is sleepy


because she is nervous
because she has a cold
because she ate too much

2) What has Tracey done during the week?


I. made a new friend
II. moved to a new town
III. moved to a new house
A.
B.
C.
D.

I only
I and II
II and III
I, II, and III

4) Why can't Tracey eat breakfast?


I. She is nervous.
II. Her stomach hurts.
III. She is late for school.
A.
B.
C.
D.

I only
I and II
II and III
I, II, and III

6) As used at the beginning of the story,


what does nervous mean?
A.
B.
C.
D.

to be happy
to be hungry
to be mad
to be scared

Questions (continued):
7) What does Dad tell Tracey to do
before school?
A.
B.
C.
D.

stop crying
get her bag
drink some juice
put on her shoes

9) What does Tracey think about most


as she walks?
A.
B.
C.
D.

her math test


answering questions
having friends
her teachers

11) What is Tracey's new school like?


A.
B.
C.
D.

big
dirty
loud
small

8) How do Tracey and Dad get to


school?
A.
B.
C.
D.

They drive.
They walk.
They run.
They take the bus.

10) When does Tracey seem to be


brave?
A.
B.
C.
D.

when she cannot eat


when she feels better
when she holds her dad's hand
when she walks up the steps to
the school

12) How do the children in Tracey's


class seem?
A.
B.
C.
D.

kind
smart
funny
different

Have you ever felt nervous about doing something new? Explain.
________________________________________________________________________
________________________________________________________________________
________________________________________________________________________
________________________________________________________________________
________________________________________________________________________
________________________________________________________________________
________________________________________________________________________

Answers and Explanations


1) C
In paragraph 1, the story says, Tracey is starting a new school today. In
paragraph 2, the reader learns that she is very sad. She is very scared. In
paragraph 3, Tracey tells her dad, I dont want to go to school today. We
can understand that Tracey is sad and scared because she is starting a new
school today. Therefore (C) is correct.
At the beginning of the story we learn, Tracey has moved to a new house. It
has already happened, so she is not moving to a new house today. Therefore
(A) is incorrect. At the beginning of the story we learn, Tracey has moved to
a new town. It has also already happened, so she is not moving to a new
town today. Therefore (B) is incorrect. Near the middle of the story we learn
that Tracey and her dad walk to school. Since Tracey walks with her dad,
she is not alone. Therefore (D) is incorrect.
2) C
The story does not say that Tracey has made a new friend. This eliminates
option (I). In paragraph 5, the reader learns that Tracey has moved to a new
town. This supports option (II). Later in paragraph 5, the reader finds out
that Tracey has moved to a new house. This supports option (III). Therefore
(C) is correct.
3) B
In the middle of the story it says, She walked into her third grade classroom.
This lets us know that Tracey is in third grade. Therefore (B) is correct. The
story does not contain information to support answer choices (A), (C), and
(D). Therefore they are incorrect.
4) B
nervous (adjective): highly excitable; unnaturally or acutely uneasy or
apprehensive.
Tracey cannot eat breakfast. In the middle of the story, Traceys dad says, I
think that is because you are nervous. As Tracy walks to school, she worries
about many things, most of which involve life at her new school. We can
understand that she is nervous. This supports option (I). Near the beginning
of the story, Tracey says, My stomach hurts. I cant eat breakfast. This
supports option (II). The story does not say that Tracey is late for school.
This eliminates option (III). Therefore (B) is correct.
5) B
Near the beginning of the story, Tracey tells her dad, I feel sick. Next,
Traceys dad tells her, I think that is because you are nervous. Therefore (B)
is the correct answer. The story does not contain information to support
answer choices (A), (C), and (D). Therefore they are incorrect.

6) D
At the beginning, the story tells us that Tracey is scared of starting a new
school. Then, Tracey tells her dad that she feels sick, her stomach hurts,
and she cant eat breakfast. Later, Traceys dad says, I think that is
because you are nervous. Traceys dad is telling her that she feels this way
because she is nervous. Using this information we can understand that
nervous can mean scared. Therefore (D) is correct. The story does not
contain information to support answer choices (A), (B), and (C). Therefore
they are incorrect.
7) C
In the middle of the story, Traceys dad tells her, Try drinking just a little
juice. Therefore (C) is correct. The story does not contain information to
support answer choices (A), (B), and (D). Therefore they are incorrect.
8) B
In the middle of the story, the author writes, Tracey and her dad walk to
school. Therefore (B) is correct. Tracey wishes she could run away, but she
does not run. Therefore (C) is incorrect. The story does not contain
information to support answer choices (A) and (D). Therefore they are
incorrect.
9) C
In the middle of the story, it says, Tracey thinks about many things. Tracey
does not think about her math test. Therefore (A) is incorrect. She thinks
about answering questions. She also thinks about her teachers. But she
thinks most about having friends. She thinks, Will I make friends? She also
thinks, What if kids laugh at me? and What if no one likes me? Both of
these ideas are related to having friends. Your friends do not laugh at you.
Your friends like you. Tracey thinks about having friends more than she thinks
about any other subject. Therefore (B) and (D) are incorrect, and (C) is the
correct answer.
10) D
At the beginning of the story, the reader learns that Tracey is scared
because she is starting a new school. Near the end of the story, Tracey
walks up the steps to school even though she is scared. Doing something
even when you are scared is brave. Therefore (D) is correct.
Tracey cannot eat because she is scared. It is not brave to not eat. Therefore
(A) is incorrect. Tracey feels better because she was brave, but feeling better
is not the action that made her seem most brave. Therefore (B) is incorrect.
Tracey holds her dads hand for comfort. This does not make her seem brave.
Therefore (C) is incorrect.
11) A

Near the end of the story, the author writes, Tracey looked up at the big
building. Because we know Tracey is walking to school, we can understand
that the big building is where she will go to school. This lets us know that
Traceys new school is big. Later in the same paragraph we learn that her
other school was small. Traceys other school was small, but her new school
is big. Therefore (A) is the correct answer.
The story does not contain information to support answer choices (B) and (C).
Therefore they are incorrect. Traceys other school was small, but not her
new school. Therefore (D) is incorrect.
12) A
Near the end of the story, the author says, Everyone seems kind. At school
other children have said, Hello, Tracey; Welcome, Tracey; and Let me
show you around. This behavior tells us that at the new school everyone
seems kind. Therefore (A) is correct.
The story does not contain information to support answer choices (B), (C),
and (D). Therefore they are incorrect.

ReadTheory.Org 2010

Name________________
Date________________

EnglishForEveryone.Org 2008

The Park
Reading Comprehension Short Stories

Directions: Read the story. Then answer the questions below.

Reem likes to go to the park with her mom.


She likes to play at the park with her friends.
Today when Reem goes to the park, she looks around and becomes very sad.
What is wrong, Reem? Mom asks.
There is so much trash on the ground, Reem says. It seems like each day, I see
more trash here.
Reem and her mom look around.
There are old boxes on the ground.
There are popped balloons on the ground.
There are old cans, too.
You are right, Mom says. The park is very dirty.
What can we do? Reem asks.
I am sure you will think of something, Mom says.
Reem and her mom go home. Reem paints a picture of a park. She uses blue,
green, brown, and yellow paint.
PLEASE KEEP OUR PARK CLEAN, she writes in big letters on top of her picture.
Lets go back to the park, Mom, Reem says.
Okay, Mom says.
Reem grabs a bunch of trash bags to take with her. She also brings along some
tape.
When Reem gets to the park, she tapes her picture on a big trash can.
Here you go! she says to her mom. She hands her mom a bag. Will you help
me pick up the trash?

I sure will, Mom says.


The children at the park run over to see what is going on.
Reem hands them bags. Lets clean this place up, she says.
The children pick up lots of trash. They talk and laugh as they work. Soon all the
bags are full.
We need to come back another day. There is still some trash on the ground,
Reem says. But the park looks much better.
It does, Mom says. I knew you would think of some way to help.

Questions:
1) Why is Reem sad?
A. Her friends are not at the park.
B. She cannot find her mom.
C. There is a lot of trash at the
park.
D. She hurts herself on the slide.

3) What does Reem do when she gets


home?
A.
B.
C.
D.

She makes a sign.


She eats dinner.
She paints her face.
She cleans her room.

5) Why does Reem tape her picture to


the trash can?
A.
B.
C.
D.

so people can see it


so it falls to the ground
so people get mad
she is tired of holding it

2) What is on the ground?


I. old boxes
II. paper
III. plastic wrappers
A.
B.
C.
D.

I only
I and II
II and III
I, II, and III

4) What is the purpose of Reems


picture?
A. to encourage people to clean up
the park
B. to make the park more beautiful
C. to improve her skills as an artist
D. to show people how beautiful
the park is

6) Who helps Reem pick up trash?


I. her dad
II. her mom
III. other children
A.
B.
C.
D.

I only
I and II
II and III
I, II, and III

Questions (continued):
7) What does Reem give the children?
A.
B.
C.
D.

brooms
gifts
mops
trash bags

9) How might Reem and her friends


feel after cleaning up?
A.
B.
C.
D.

cold
good
mad
sad

8) What does Reem learn at the park?


A. that she can help make things
better
B. that people do not care about
the park
C. that her mom does not like to
pick up trash
D. that the park has no trash cans

10) How does the park look after


everyone cleans it up?
A.
B.
C.
D.

all clean
better
very dirty
dirtier

11) At the end of the story, why does


Reem say that she needs to come
back another day?
A.
B.
C.
D.

to play at the park


to clean up more trash
to empty the trash bags
to show her dad her work

What are some ways we can fight pollution and preserve the environment?

________________________________________________________________________
________________________________________________________________________
________________________________________________________________________
________________________________________________________________________
________________________________________________________________________

Answers and Explanations


1) C
At the beginning, the story says, Today when Reem goes to the park, she
looks around and becomes very sad. When Reems mom asks Reem what is
wrong, Reem says, There is so much trash on the ground. Reem also says,
It seems like each day, I see more trash here. We can understand from this
that Reem is sad because there is a lot of trash at the park. Therefore (C) is
correct.
The story does not provide information to support answer choices (A), (B),
and (D). Therefore they are incorrect.
2) A
In the middle of the story, Reem and her mom look around the playground
and see that there are old boxes on the ground. This supports option (I).
The story does not say that there is paper on the ground, so that eliminates
option (II). The story does not say that there are plastic wrappers on the
ground. That eliminates option (III). Therefore (A) is correct.
3) A
In the middle of the story, Reem and her mom go home. Reem paints a
picture of a park. She uses blue, green, brown, and yellow paint. PLEASE
KEEP OUR PARK CLEAN, she writes in big letters on top of her picture. We
can understand from this that when Reem got home, she made a sign.
Therefore (A) is correct.
The story does not provide information to support answer choices (B), (C),
and (D). Therefore they are incorrect.
4) A
When Reem and her mother go to the park, Reem notices that it is very dirty.
Reem wonders what she can do about the dirty park. Reems mother tells
Reem, I am sure you will think of something. As soon as Reem gets home,
Reem makes a sign that says, PLEASE KEEP OUR PARK CLEAN. Reem
then goes back to the park and places the sign over a trashcan. We can
understand from this that Reem wanted to do something to help clean up the
park. Reem decided to make a sign that encourages people to keep the park
clean. Reem places the sign right above the garbage can to remind people to
throw trash into the garbage can. Therefore (A) is correct.
Reem wants to make the park more beautiful by getting rid of the garbage,
not by putting up pretty posters. Therefore (B) is incorrect. Reem is trying to
clean up the park, not become a better artist. Therefore (C) is incorrect. The
posters do not show how beautiful the park is. Therefore (D) is incorrect.
5) A
Reem is sad because the park is dirty. Reem wants to do something about
the dirty park, so Reem makes a sign encouraging people to keep the park

clean. Reem tapes the sign up to a garbage can. Since Reem wants to clean
up the park and has made a sign encouraging other people to keep the park
clean, Reem must want people to read the sign. It will help people remember
to keep the park clean. Therefore (A) is correct.
The purpose of a sign is to tell something to the person who reads it. A sign
that falls to the ground cannot tell people anything, so Reem would not tape
up a sign just so it could fall to the ground. Therefore (B) is incorrect. Reem
wants people to throw their trash away, not to be mad. Therefore (C) is
incorrect. The story does not say that Reem is tired of holding the sign.
Therefore (D) is incorrect.
6) C
Reems dad is not in the story, so this rules out option (I). Near the end of the
story, Reem givers her mom a bag and asks if her mom will help pick up the
trash. I sure will, Reems mom says. We can understand from this that
Reems mom helped pick up the trash. This supports option (II). Also near the
end of the story, the children at the park run over to see what is going on.
Reem hands them bags. Then, the children pick up lots of trash. We can
understand from this that the children help Reem pick up trash at the park.
This supports option (III). Therefore (C) is correct.
7) D
When the children at the park come up to Reem, she hands them bags.
Then the children help pick up trash. This means that Reem gave the children
trash bags. Therefore (D) is correct.
While brooms are helpful items for cleaning, the children were picking up
trash and putting it in trash bags, not sweeping. Therefore (A) is incorrect.
The story does not provide information to support (B), so it is incorrect. Mops
are helpful items for cleaning, but the students were not mopping the ground.
Therefore (C) is incorrect.
8) A
In this story, Reem first sees that the park is dirty. Reem wants to help
change the park. Reem decides to make a sign to help people remember to
keep the park clean. Reem also decides to pick up trash. After Reem does
these things, the park looks much better. We can understand from this that
Reem has learned that she can help to make things better. Therefore (A) is
correct.
The children in the park helped Reem, so the children must care about the
park. Therefore (B) is incorrect. Reems mom helped pick up trash. Therefore
(C) is incorrect. The park did have a trash can. Therefore (D) is incorrect.
9) B
While Reem and the children clean up the park, they talk and laugh. When
they are finished, the park looks much better. Since the children had fun

working together and accomplished their goal, they should feel good about
cleaning up. Therefore (B) is correct.
The story does not tell us what the weather is like at the park, so there is not
enough information to know whether they are cold. Therefore (A) is incorrect.
The children had fun working together and did something good, so they
should not feel mad. Therefore (C) is incorrect. For these reasons, they
should not feel sad, either. Therefore (D) is incorrect.
10) B
Reem and the children fill all the garbage bags. Reem says, We need
to come back another day. There is still some trash on the ground. Then
Reem says, But the park looks much better. We can understand from this
that there is still some trash at the park, but the park looks better. Therefore
(B) is correct.
Since there is still some trash on the ground, the park is not all clean.
Therefore (A) is incorrect. Since the park looks much better, it does not still
look very dirty. Therefore (C) is incorrect. The park looks better because there
is less trash, not because it is dirtier. Therefore (D) is incorrect.
11) B
At the end of the story, Reem says, We need to come back another day.
There is still some trash on the ground. Since we know that Reem wants the
park to be clean and there is still trash on the ground, we can understand that
Reem wants to come back another day to pick up the rest of the trash.
Therefore (B) is correct.
Reem says she wants to come back because there is still trash at the park,
not because she wants to play. Therefore (A) is incorrect. The story does not
provide information to support answer choices (C) and (D). Therefore they are
incorrect.

ReadTheory.Org 2010

Name________________
Date________________

EnglishForEveryone.Org 2008

Vet Emergency!
Reading Comprehension Short Stories

Directions: Read the story. Then answer the questions below.

It is 10:00 on a Saturday night. The doors to the animal emergency room


fly open.
Cyrus carries a tiny, black lab puppy into the building.
"Save him. Please!" cries Cyrus. "He ran out into the street and got hit by a car."
The puppy is whining. There is a lot of blood.
Dr. Williams comes from behind the desk. She reads the puppys name tag.
Here, let me take Lobo from you, Dr. Williams tells Cyrus gently. I will take him
in the back for x-rays and see what needs to be done.
When Dr. Williams gets to the back, she takes a closer look at Lobo. Her face
is grim. She can see the dog's leg is broken badly. She fears there is other
damage she cannot see.
The assistants in the room are silent. Everyone knows that Lobo may not survive.
If he does survive, his leg injury may be so severe that walking will be hard for
him to do.
The x-rays show that Lobos front leg has been broken in three places. He is also
bleeding inside.
Dr. Williams gets to work. She knows she must stop the bleeding quickly.
The surgery takes just under two hours. Dr. Williams fixes the damage to Lobo's
leg. She stops the bleeding inside his chest.
I will go tell Lobos owner that this little pup is going to make it, Dr. Williams
says. Well see in a few weeks how well he will walk again.

Questions:
1) The story takes place in a(n)
A.
B.
C.
D.

police station
waiting room
doctor's office
animal hospital

2) What does the mood in the room


seem to be?
A.
B.
C.
D.

angry
joyful
relaxed
tense

Questions (continued):
3) Dr. Williams knows just how badly
Lobo has been hurt when
A.
B.
C.
D.

Cyrus asks for help


she takes a closer look
she sees Lobo is a lab puppy
she looks at the x-rays

5) As used in the middle of the story,


which is the best synonym
for grim?
A.
B.
C.
D.

calm
serious
hopeful
pleasant

7) Dr. Williams is a(n)


A.
B.
C.
D.

pediatrician
dentist
internist
veterinarian

9) In how many places was Lobo's leg


broken?
A.
B.
C.
D.

1
2
3
4

4) Around what time did Lobo's


surgery end?
A.
B.
C.
D.

10:00 at night
10:00 in the morning
noon
midnight

6) For about how long is Lobo in


surgery?
A.
B.
C.
D.

45 minutes
1hour, 30 minutes
1 hour, 50 minutes
2 hours, 15 minutes

8) To have a severe injury means to


A.
B.
C.
D.

have a mild injury


have a lot of pain
not be able to walk
have a very bad injury

10) What does the reader know about


Lobo by the end of the story?
I. He will survive.
II. He is a black lab.
III. He will walk fine.
A.
B.
C.
D.

I only
I and II
II and III
I, II, and III

How do you think Cyrus will react when Dr. Williams tells him about Lobo? Why?
________________________________________________________________________
________________________________________________________________________
________________________________________________________________________
________________________________________________________________________

Answers and Explanations


1) D
At the beginning, the story says, It is 10:00 on a Saturday night. The doors to
the animal emergency room fly open. Animal emergency rooms are in animal
hospitals. Since the story starts in the animal emergency room, we can
understand that the story takes place in an animal hospital. Therefore (D) is
correct.
There is no police station mentioned in the story. Therefore (A) is incorrect.
Animal hospitals do have waiting rooms. However, the puppy has surgery in
the story, and surgery does not take place in the waiting room. This means
the story does not take place in the waiting room. Therefore (B) is incorrect.
Animals are not treated in doctors offices. Since Lobo is a puppy, Lobo is
treated in an animal hospital, not a doctors office. Therefore (C) is incorrect.
2) D
At the beginning of the story, Cyrus brings the puppy in and cries out for Dr.
Williams to save him. When Dr. Williams takes a look at Lobo, her face is
grim. The assistants are silent. Everyone is worried about Lobo. We can
understand from this information that everyone in the room is scared for Lobo
and no one knows what will happen to the puppy. Tense means in a state of
suspense or nervous tension. Since everyone in the story is in suspense
wondering what will happen to Lobo, the mood of the room is tense.
Therefore (D) is correct.
Everyone is worried about Lobo, but no one does anything that makes us
think he or she is angry. Therefore (A) is incorrect. To be joyful is to be happy
and feel good. No one feels happy, because Lobo is badly hurt. Therefore (B)
is incorrect. To feel relaxed is to feel at ease. Everyone is worried, so no one
feels at ease. Therefore (C) is incorrect.
3) D
In the middle of the story, Dr. Williams takes a look a Lobo. The story says,
She can see the dog's leg is broken badly. She fears there is other damage
she cannot see. Then Dr. Williams looks at the x-rays. The x-rays show that
Lobos front leg has been broken in three places. He is also bleeding inside.
We can understand from this information that Dr. Williams knew how badly
the puppys leg was broken and knew for sure that the puppy was bleeding
inside after she looked at the x-rays. Therefore (D) is correct.
When Cyrus first asks for help, Dr. Williams has not looked at the puppy at all.
She could not have known at that time how badly Lobo was hurt. Therefore
(A) is incorrect. When Dr. Williams takes a closer look, she knows the dogs
leg is broken, and she is afraid that there is more damage. However, she is
not sure that the dog is bleeding inside until after she looks at the x-rays.
Therefore (B) is incorrect. Dr. Williams sees that Lobo is a black lab before
she has a chance to take a look at his injuries. Therefore (C) is incorrect.

4) D
At the beginning, the story says, It is 10:00 on a Saturday night. Near the
end, the story says, The surgery takes just under two hours. Since the story
starts at 10:00 at night and the surgery takes just under two hours, the
surgery ends around midnight. Therefore (D) is correct.
The story begins at 10:00 at night, but the surgery isnt done until two hours
later. Therefore (A) is incorrect. The story happens at night, not in the
morning. Therefore (B) is incorrect. The surgery did not end at noon, because
the story happens at night. Therefore (C) is incorrect.
5) B
grim (adjective): dismal or gloomy; stern; resolute.
In the middle, the story says, When Dr. Williams gets to the back, she takes
a closer look at Lobo. Her face looks grim. She can see the dog's leg is
broken badly. She fears there is other damage she cannot see. This
information tells us that Dr. Williams knows that the dog is badly hurt and is
afraid for the dog. Since Dr. Williams knows that the dog is hurt, her face
must look stern and gloomy. We can understand from this information that
grim means stern and gloomy. Something serious is earnest or so important it
causes anxiety. Serious a good synonym for grim. Therefore (B) is correct.
Calm means still, serene or not excited. While Dr. Williams was not excited by
the condition of the dog, it is unlikely her face would look serene because she
felt anxious for the dog. Therefore (A) is incorrect. Something hopeful is
optimistic or promising. Since Dr. Williams is afraid that the dog is badly hurt,
her face would not look optimistic. Therefore (C) is incorrect. Pleasant means
giving a sense of happy satisfaction. Since Dr. Williams is so worried about
the puppy, the look on her face is probably not one of happy satisfaction.
Therefore (D) is incorrect.
6) C
Near the end, the story says, The surgery takes just under two hours. 1
hour, 50 minutes is only 10 minutes less than two hours. This is just a little
less than two hours. Therefore (C) is correct.
45 minutes is less than 1 hour, so it is not even close to 2 hours. Therefore
(A) is incorrect. 1 hour, 30 minutes is in between 1 and 2 hours, so it is not
just under two hours. Therefore (B) is not the best choice. 2 hours, 15
minutes is over 2 hours, not just under 2 hours. Therefore (D) is incorrect.
7) D
A veterinarian is a doctor that takes medical care of animals. Since Dr.
Williams treated Lobo, and Lobo is an animal, Dr. Williams is a veterinarian.
Therefore (D) is correct.
A pediatrician takes care of children. Dr. Williams takes care of animals, not
children. Therefore (A) is incorrect. A dentist takes care of peoples teeth. Dr.
Williams takes care of animals, not peoples teeth. Therefore (B) is incorrect.

An internist is a doctor that specializes in human internal medicine. Since Dr.


Williams treats animals, she is not an internist. Therefore (C) is incorrect.
8) D
severe (adjective): very serious; harsh or strict.
Lobo is badly hurt. No one knows if he will even survive. In the middle, the
story says, If he does survive, his leg injury may be so severe that walking
will be hard for him to do. Since the injury might make walking hard for Lobo
because the injury is severe, severe means very serious. To have a very
serious injury is to have a very bad injury. Therefore (D) is correct.
Mild means gentle or moderate. If Lobo had a mild injury, it wouldnt make
him unable to walk, or even possibly die. This means having a mild injury is
the opposite of having a severe injury. Therefore (A) is incorrect. Although a
severe injury might cause a lot of pain, severe means very serious, not very
painful. Other things besides a severe injury could cause a lot of pain, such
as an illness. Therefore (B) is incorrect. The word severe tells us how bad the
injury is. It does not tell us what type of injury it is. We know that the injury
might affect Lobos ability to walk because it is a leg injury. A severe injury
does not necessarily affect the ability to walk. Therefore (C) is incorrect.
9) C
In the middle, the story says, The x-rays show that Lobos front leg has been
broken in three places. Lobos leg was broken in 3 places. Therefore (C) is
correct. The story does not provide information to support answer choices
(A), (B) or (D). Therefore they are incorrect.
10) B
At the end of the story, Dr. Williams says, I will go tell Lobos owner that this
little pup is going to make it[.] We can understand from this sentence that
Lobo will survive. This supports option (I). Near the beginning, the story says,
Cyrus carries a tiny black lab puppy into the building. We can understand
from this information that Lobo is a black lab. This supports option (II). At the
end of the story, Dr. Williams says, Well see in a few weeks how well he will
walk again. We can understand from this sentence that Lobo may or may not
walk fine. This eliminates option (III). Therefore (B) is the correct answer.

englishforeveryone.org

10

15

20

Name________________
Date________________

Intermediate Critical Reading - Clocks

Clocks are used to measure time. Clocks are used to show the time. Clocks are used so
that people can agree on the time.
People use clocks to measure units of time that are shorter than natural units. Natural
units are the day, the month, and the year. The sun rises at the beginning of each day. The moon
is new at the beginning of each month. The sun appears at the same place in the sky at the
beginning of each year.
The clock is a very old invention. Sundials were used in ancient times. Sundials work by
measuring shadows that are cast by the sun. Candle clocks were used a very long time ago.
Candle clocks work by measuring the time it takes for the wax to melt. Hourglasses are similar to
candle clocks because they also measure the time it takes for something to happen. Hourglasses
work by measuring the time it takes for sand to pour through a small opening into a glass.
Mechanical clocks appeared in the 13th century in Europe. They work with a system of moving
gears. The gears always move at the same speed. Pendulum clocks work with a swinging weight.
The swinging weight is called a pendulum. The pendulum makes the clock parts move.
Now people typically use mechanical clocks or digital clocks. Digital clocks show
numbers on a screen. The numbers represent the time. Some people use auditory clocks.
Auditory clocks use language to tell the time aloud. Computers also use clocks. Computers use
internal clocks in order to work properly.
In todays world, clocks are everywhere in homes, schools, offices, and public places.
Many people live their lives according to the clock. People go to work and return home
according to the clock. School days start and end according to the clock. Airplanes take off and
land according to the clock. It is fun to imagine a world without clocks!

Questions
1. According to the passage, which of the following statements is/are true?
I)
The clock is peoples oldest invention.
II) Sundials were the first kind of clock that people invented.
III) Mechanical clocks contain gears.
A)
B)
C)
D)
E)

I only
II only
III only
I and II only
II and III only

2. What is the main idea of the third paragraph?


A)
B)
C)
D)
E)

Why clocks are used


How sundials work
How hourglasses are similar to candle clocks
What types of clocks have been invented
How mechanical clocks are different from digital clocks

3. In line 16, auditory most closely means


A)
B)
C)
D)
E)

related to machines
related to numbers
related to hearing
related to moving parts
related to the sun

Answers and Explanations


1. The correct answer is C.
Incorrect. While line 7 states that The clock is a very old invention, it does not
state that the clock is peoples oldest invention.
II) Incorrect. While line 7 states that Sundials were used in ancient times, there is no
mention of whether it was the oldest type of clock.
III) Correct. Lines 11-12 state that Mechanical clocks appeared in the 13th century in
Europe. They work with a system of moving gears.
I)

A)
B)
C)
D)
E)

I only
II only
III only
I and II only
II and III only

2. The correct answer is D.


A) Incorrect. Line 3 in paragraph 2 explains why clocks are used with the statement
People use clocks to measure units of time that are shorter than natural units.
B) Incorrect. While lines 7-8 explain how sundials work with the statement Sundials work
by measuring shadows that are cast by the sun, this is only one explanation of how a
type of clock works in a series of types of clocks and their workings. This is not the main
idea of the paragraph.
C) Incorrect. While lines 9-10 explain how hourglasses are similar to candle clocks, this is
only one explanation of how these clocks work in a series of types of clocks and how
they work. This is not the main idea of the paragraph.
D) Correct. The paragraph is a listing of various types of clocks and how they work.
E) Incorrect. While lines 12-13 explain what mechanical clocks are and how they work by
stating Mechanical clocks appeared in the 13th century in Europe. They work with a
system of moving gears. The gears always move at the same speed, there is no mention
of digital clocks in paragraph 3. More information is given about mechanical clocks and
digital clocks in paragraph 4: Now people usually use mechanical clocks or digital
clocks. Digital clocks show numbers on a screen. The numbers represent the time.
3. The correct answer is C.
A) Incorrect. Lines 15-17 tell about three kinds of clocks mechanical, digital, and
auditory. Since these words are related to three different types of clocks, auditory
cannot mean mechanical.

B) Incorrect. Lines 15-17 tell about three kinds of clocks mechanical, digital, and
auditory. Lines 1516 state that Digital clocks show numbers on a screen. Since
numbers on a screen are related to digital clocks, it is unlikely that the word auditory is
related to numbers.
C) Correct. Since line 17 states that Auditory clocks use language to tell the time aloud,
and hearing the sense which would be associated with time told aloud, it is very likely
that the word auditory is related to hearing.
D) Incorrect. Lines 15-17 tell about three types of clocks mechanical, digital, and
auditory. Since lines 12-13 state that Mechanical clocks appeared in the 13th century in
Europe. and They work with a system of moving gears, it seems that mechanical
clocks are related to moving parts, not primarily auditory clocks. So the word auditory is
most likely not related to moving parts.
E) Incorrect. Lines 15-17 tell about three types of clocks mechanical, digital, and auditory.
The sun is mentioned in lines 6-8 with the sentences Sundials were used in ancient
times. Sundials work by measuring shadows that are cast by the sun. Sundials were
invented long before other clocks mentioned in this passage. It is unlikely that the word
auditory is related to the sun.

englishforeveryone.org

10

15

20

25

30

Name________________
Date________________

Intermediate Critical Reading Hybrid Vehicles

A hybrid vehicle is a vehicle which uses two or more kinds of propulsion. Most hybrid
vehicles use a conventional gasoline engine as well as an electric motor to provide power to the
vehicle. These are usually called hybrid-electric-vehicles, or HEVs. Hybrids use two types of
propulsion in order to use gasoline more efficiently than conventional vehicles do. Most hybrid
vehicles use the gasoline engine as a generator which sends power to the electric motor. The
electric motor then powers the car. In conventional vehicles, the gasoline engine powers the
vehicle directly.
Since the main purpose of using a hybrid system is to efficiently use resources, most
hybrid vehicles also use other efficient systems. Most hybrid vehicles have regenerative braking
systems. In conventional vehicles, the gasoline engine powers the brakes, and the energy used in
braking is lost. In regenerative braking systems, the energy lost in braking is sent back into the
electrical battery for use in powering the vehicle. Some hybrid vehicles use periodic engine shutoff as a gassaving feature. When the vehicle is in idle, the engine temporarily turns off. When
the vehicle is put back in gear, the engine comes back on. Some hybrids use tires made of a stiff
material which rolls easily and prevents drag on the vehicle.
Hybrid vehicles save up to 30% of the fuel used in conventional vehicles. Since hybrid
vehicles use less gasoline, the cost of operating them is less than the cost of operating
conventional vehicles. Therefore, hybrid vehicles are gaining in popularity. According to a
recent study, over the five years it typically takes for a person to pay for a car, a typical hybrid
car driver would save over $6,000 in gasoline costs. Almost all the worlds major automakers are
planning and producing safe and comfortable hybrid vehicles to meet the demand for these
increasingly popular vehicles.
Although hybrid vehicles do represent a marked improvement in environmentally
conscious engineering, there still remains one significant potential drawback: battery disposal.
Batteries are difficult to dispose of in an environmentally safe manner. To properly dispose of
the battery in a hybrid car requires substantial effort. If the battery is not disposed of properly,
the environmental impact of a hybrid car can be equal, if not greater than, that of a regular gas
only car.
Since hybrid vehicles use less gasoline than conventional vehicles, they put fewer
emissions into the atmosphere than conventional vehicles do. As hybrids become more popular,
conventional vehicles are being used less, and the level of emissions being put into the air is
decreasing. Hybrid vehicles are an example of an energy-efficient technology that is good for
both consumers and the environment.

Questions
1. According to the passage, which of the following statements is/are true?
I)
Two braking systems are used in most hybrid vehicles.
II) Approximately 30% of vehicles on the road are hybrid vehicles.
III) Some HEVs have engines which turn off when the vehicle is not moving.
A)
B)
C)
D)
E)

I only
II only
III only
I and II only
II and III only

2. According to the passage, HEVs use two types of propulsion mainly in order to
A)
B)
C)
D)
E)

go faster.
use gasoline efficiently.
provide a comfortable ride.
provide a safe driving experience.
put fewer emissions into the atmosphere.

3. In line 9, regenerative most closely means


A)
B)
C)
D)
E)

electric
gasoline
powerful
restorative
secondgeneration

4. In the context of the passage, which of the following best articulates how the author regards
the topic?
A) Conventional vehicles may be more powerful than hybrid vehicles, but hybrid vehicles
are the more socially responsible vehicles to operate.
B) Since hybrid vehicles use less gasoline and put fewer emissions into the atmosphere than
conventional vehicles, they are better for drivers and for the environment.
C) Conventional vehicles are faster than hybrid vehicles, but hybrid vehicles are better for
the environment than conventional vehicles.
D) Since hybrid vehicles are much less expensive to purchase and operate than conventional
vehicles, they are a smarter buy than conventional vehicles.
E) Two sources of propulsion provide more power to a hybrid vehicle, making it more
powerful and faster than a conventional vehicle, so it is more socially responsible to buy
a conventional vehicle.

Answers and Explanations


1. The correct answer is C.
I)

Incorrect. According to line 1, A hybrid vehicle is a vehicle which uses two or


more kinds of propulsion, not braking systems. While line 9 states that Most
hybrid vehicles have regenerative braking systems, there is no mention that they
use any additional type of braking system.
II) Incorrect. While line 16 states that Hybrid vehicles save up to 30% of the fuel used
in conventional vehicles, there is no mention of the percentage of hybrid vehicles
on the road.
III) Correct. Some HEVs have engines which turn off when the vehicle is not moving.
According to line 13, When the vehicle is in idle, the engine temporarily turns
off.
A)
B)
C)
D)
E)

I only
II only
III only
I and II only
II and III only

2. The correct answer is B.


A) Incorrect. There is no mention of the vehicles speed anywhere in the passage.
B) Correct. According to line 8, the main purpose of using a hybrid system is to efficiently
use resources, and lines 1617 and 23 state that hybrid vehicles use less gasoline, and
since the fuel source, or resource, of conventional vehicles is gasoline, this is the correct
answer.
C) Incorrect. While line 21 states that automakers are planning and producing safe and
comfortable hybrid vehicles, line 8 states that the main purpose of using a hybrid
system is to efficiently use resources.
D) Incorrect. While line 21 states that automakers are planning and producing safe and
comfortable hybrid vehicles, line 8 states that the main purpose of using a hybrid
system is to efficiently use resources.
E) Incorrect. While lines 29-30 state that hybrid vehicles put fewer emissions into the
atmosphere than conventional vehicles do, line 8 states that the main purpose of using
a hybrid system is to efficiently use resources.
3. The correct answer is D.
A) Incorrect. According to lines 1112, In regenerative braking systems, the energy lost in
braking is sent back into the electrical battery for use in powering the vehicle. If the
braking systems were electric, the energy would come from the electric system; it would
not be sent to the electrical system. So regenerative cannot mean electric.
B) Incorrect. According to lines 1012, In conventional vehicles, the gasoline engine
powers the brakes, and the energy used in braking is lost. In regenerative braking
systems, the energy lost in braking is sent back into the electrical battery for use in

powering the vehicle. Since gasoline engines are compared to systems which use
regenerative braking, gasoline cannot mean the same as regenerative.
C) Incorrect. Lines 89 state that most hybrid vehicles also use other efficient systems.
Most hybrid vehicles have regenerative braking systems. Although it is possible that an
efficient system could be powerful, power is not its main attribute efficiency is. So it is
unlikely that regenerative means powerful.
D) Correct. According to lines 1112, In regenerative braking systems, the energy lost in
braking is sent back into the electrical battery for use in powering the vehicle. So the
energy puts back or restores the electrical batterys power, which means that the word
regenerative is likely to mean restorative.
E) Incorrect. Since the word regenerative contains the prefix re and the root word
generative, it seems logical that it could mean secondgeneration, especially coupled
with the idea that hybrid vehicles are a new type or new generation of vehicles. However,
the sentence in lines 1112, In regenerative braking systems, the energy lost in braking
is sent back into the electrical battery for use in powering the vehicle seems to indicate
that the word regenerative describes the action or function of the braking system rather
than its place in vehicular genealogy.
4. The correct answer is B.
A) Incorrect. While it may be true that conventional vehicles are more powerful than hybrid
vehicles, there is no mention in the passage that conventional vehicles are more
powerful than hybrid vehicles.
B) Correct. In the last paragraph, the author makes all of these points. Lines 16-17 state that
Since hybrid vehicles use less gasoline than conventional vehicles, they put fewer
emissions into the atmosphere than conventional vehicles do. Lines 32-33 state that
Hybrid vehicles are an example of an energyefficient technology that is good for both
consumers and the environment. The author does mention the potential drawbacks in
improper battery disposal, but this does not contradict their overall view regarding the
benefit of hybrid vehicle usage.
C) Incorrect. While it may be true that conventional vehicles are faster than hybrid vehicles,
there is no mention in the passage that conventional vehicles are faster than hybrid
vehicles.
D) Incorrect. While lines 16-17 state that Since hybrid vehicles use less gasoline, the cost of
operating them is less than the cost of operating conventional vehicles, there is no
mention of the cost of purchasing hybrid vehicles.
E) Incorrect. There is no mention in the passage that two sources of propulsion make the
hybrid vehicle either more powerful or faster.

englishforeveryone.org

10

15

20

Name________________
Date________________

Intermediate Critical Reading - Photography

In 1760, a man named Tiphaigne de la Roche made a bizarre prediction. In an imaginary


story called Giphantie, mirror images of scenes from nature could be captured permanently on a
canvas covered with a sticky material. After the material dried in darkness, the image would
remain on the canvas forever. At the time, the idea was unheard of. It was not until the following
century that the concept of photography was born, starting with some experiments by Nicphore
Nipce.
Nicphore Nipce, who was a French inventor, was interested in lithography, which is a
printmaking technique. He was experimenting with lithography when he found a way of copying
etchings onto glass and pewter plates using a chemical that changes when it is exposed to light.
He learned to burn images onto the plates and then print the images on paper. He shared his
findings with Louis Jacques Mand Daguerre, who improved the process and announced it to the
French Academy of Sciences in 1839. The Daguerreotype, the photography method named after
Daguerre, met with great success. It was so successful, in fact, that French newspapers said the
French public had an illness called Daguerreotypomania! Daguerreotypes were inexpensive and
were suitable for portraiture. People called the Daguerreotype a mirror with a memory. Some
portrait artists went out of business when Daguerreotypes came into vogue. Others became
Daguerreotypists, now known as photographers.
Photography took off and today includes You Tube fans, IReporters, and everybody
with a cell phone camera. Today photography captures life around the globe. The bizarre
prediction of Mr. de le Roche has come true, and then some.
But how was the prediction of mirror-image pictures made in the first place? Was de la
Roches prediction of mirror-image photography just a lucky guess? Or was the Daguerreotype a
picture-perfect case of life imitating art?

Questions
1. According to the passage, which of the following statements is/are true?
I)
Photographic images were shown in de la Roches book Giphantie.
II) Nicphore Nipce experimented with printmaking.
III) Louis Jacques Mand Daguerre invented photography.
A)
B)
C)
D)
E)

I only
II only
III only
I and II only
II and III only

2. In line 1, bizarre most closely means


A)
B)
C)
D)
E)

common
humorous
obvious
popular
strange

3. In the context of the passage, which of the following best articulates the authors opinion?
A) It is hard to know how de le Roche predicted a photographic-type process a full century
before photography was invented.
B) It is hard to draw a comparison between early Daguerreotypists and modern-day
photographers.
C) It was inaccurate to say that the Daguerreotype was a mirror with a memory, since the
images were not reversed.
D) It is difficult to understand why the French public preferred Daguerreotypes to painted
portraiture.
E) It was difficult for portrait artists to acquire the skills necessary to become
Daguerreotypists.

Answers and Explanations


1. The correct answer is B.
I) Incorrect. Photographic images were not shown in de la Roches book Giphantie.
According to lines 1-3, In an imaginary story called Giphantie, mirror images of
scenes from nature could be captured permanently on a canvas Since the story
was an imaginary story about images, the images were not shown. Further, according
to line 1, the story was a prediction, which tells about something that does not yet
exist.
II) Correct. According to lines 6-7, Nicphore Nipce, who was a French inventor,
was interested in lithography, which is a printmaking technique. He was
experimenting with lithography Since he was experimenting with lithography,
and lithography is a type of printmaking, then he was experimenting with
printmaking.
III) Incorrect. It is not stated in the passage that Daguerre invented photography.
According to line 10, Daguerre improved the process which had been shared with
him by Nicphore Nipce.
A)
B)
C)
D)
E)

I only
II only
III only
I and II only
II and III only

2. The correct answer is C.


A) Incorrect. The prediction could not have been common. According to line 4, At the
time, the idea was unheard of. When something is unheard of, it is very uncommon or
completely nonexistent.
B) Incorrect. The story was imaginary (line 1), and the idea was unheard of (line 4), and
while some imaginary stories might be humorous, there is nothing which indicates that
the prediction was humorous.
C) Incorrect. Something which is obvious is easy to see. Since the prediction was made in an
imaginary story (line 1), the idea was unheard of (line 4), and photography was not
invented until the following century (lines 4-5), all stated in the first paragraph, the
prediction could not have been obvious.
D) Incorrect. The prediction could not have been popular. In order for something to be
popular, it must be well-known. Since this idea was unheard of, according to line 4, it
was not well-known, and therefore it was not popular.
E) Correct. The prediction was strange.
3. The correct answer is A.
25

A) Correct. The last paragraph shows the authors difficulty understanding how the

prediction was made. But how was the prediction of mirror-image pictures made in the
first place? Was de la Roches prediction of mirror-image photography just a lucky
guess? Or was the Daguerreotype a picture-perfect case of life imitating art?
B) Incorrect. In lines 14-15, the author writes, Others became Daguerreotypists, now
known as photographers. Here the author states that Daguerreotypists are the same as
photographers, so she has no difficulty comparing them.
C) Incorrect. Although may be true that the phrase is inaccurate, there was no mention that
the term mirror with a memory was inaccurate.
D) Incorrect. In lines 12-14, the author writes, It was so successful, in fact, that French
newspapers said the French public had an illness called Daguerreotypomania!
Daguerreotypes were inexpensive and were suitable for portraiture, which shows that
the French public liked Daguerretypes because they were inexpensive and suitable for
portraiture.
E) Incorrect. In lines 14-15, the author writes, Others (other portrait artists) became
Daguerreotypists, now known as photographers. There was no mention of the difficulty
of acquiring photography skills.

englishforeveryone.org

Name________________
Date________________

Intermediate Critical Reading The Robin

The North American Robin is usually called the robin. The robin is a bird. The male robin
has a bright red breast. The robin has a cheerful song.
Robins live throughout North America. They live in Canada. They live in Alaska. They live
in the 48 lower states. In the winter, robins go south. In the winter, some robins go to central
5 Mexico. Some robins go to the Pacific coast.
Robins eat during the day. Robins eat beetles. Robins eat berries. Robins eat fruits. Robins
eat about 40% invertebrates like beetles and grubs. Robins eat about 60% fruits and berries. Mother
robins find food for baby robins. Father robins find food for baby robins. Baby robins eat worms.
Baby robins eat other soft-bodied animals too.
10
Squirrels prey on robin eggs and baby robins. Snakes prey on baby robins and robin eggs.
Some other birds prey on baby robins and robin eggs. Blue jays prey on baby robins and robin eggs.
Crows prey on baby robins and robin eggs. Hawks prey on adult robins. Cats prey on adult robins.
Snakes prey on adult robins.
To stay safe, robins stay vigilant. Robins pay close attention to their surroundings. When
15 they gather in groups, they watch other robins for signs of danger. Robins give loud warning calls
when a dangerous predator approaches. Although robins are territorial, they band together for
protection from predators.
The robin is a sign a spring. Some people think that seeing the first robin of springtime brings
good luck.

Questions
1. According to the passage, which of the following statements is/are true?
I)
Some robins go to South America during the winter.
II) The female robin has a bright red breast.
III) Cats hunt adult robins.
A)
B)
C)
D)
E)

I only
II only
III only
I and II only
II and III only

2. What is the main idea of the fourth paragraph?


A)
B)
C)
D)
E)

Squirrels hunt baby robins.


Other animals prey on robins.
Adult robins do not care for their young.
Crows and hawks hunt adult robins.
Other birds steal robin eggs.

3. In line 14, vigilant most closely means


A)
B)
C)
D)
E)

attentive
dangerous
noisy
safe
territorial

Answers and Explanations


1. The correct answer is C.
I)

Incorrect. Lines 4-5 state that In the winter some robins go to central Mexico.
Some robins go to the Pacific coast. While some robins may go to Central
America during the winter, there is no information in the passage about Central
America.
II) Incorrect. Lines 1-2 state that The male robin has a bright red breast. There is no
mention of the coloring of the female robin.
III) Correct. Line 13 states that Cats prey on adult robins. To prey on means to hunt,
so it is true that cats hunt adult robins.
A)
B)
C)
D)
E)

I only
II only
III only
I and II only
II and III only

2. The correct answer is B.


A) Incorrect. While line 10 states that Squirrels prey on robin eggs and baby robins, the
paragraph goes on to list other animals which prey on, or hunt, robins and their eggs. So
the main idea does not center on squirrels, but rather on animals which prey on robins
and their eggs.
B) Correct. Lines 10-12 list the animals which prey on baby robins and robin eggs. Lines
12-13 list animals which prey on adult robins. Therefore, the entire paragraph is centered
on animals which prey on robins.
C) Incorrect. While lines 10-12 list animals which prey on baby robins and robin eggs, and
lines 12-13 list animals which prey on adult robins, there is no mention that the adult
robins do not care for their young.
D) Incorrect. Lines 12-13 state that Crows prey on baby robins and robin eggs. Hawks prey
on adult robins. However, there is no mention that crows prey on adult robins or that
hawks prey on baby robins and robin eggs.
E) Incorrect. While lines 10-12 state that some animals prey on baby robins and robin eggs,
the main idea does not center on the robin eggs, but rather on the animals which prey on
robins.
3. The correct answer is A.
A) Correct. According to line 14, To stay safe, robins are vigilant. Robins pay close
attention to their surroundings. Vigilant robins are robins which pay close attention to
their surroundings. To pay attention means to be attentive. Therefore, vigilant must
mean attentive.
B) Incorrect. According to lines 14-16, To stay safe, robins are vigilant. Robins pay close
attention to their surroundings. When they gather in groups, they watch other robins for
signs of danger. Since vigilant robins watch for signs of danger, it stands to reason that
the word vigilant does not mean dangerous.
C) Incorrect. According to lines 14-16, To stay safe, robins are vigilant. Robins pay close
attention to their surroundings. When they gather in groups, they watch other robins for

signs of danger. Robins give loud warning calls when a dangerous predator approaches.
Although vigilant robins do give loud or noisy warning calls in when there is a dangerous
predator in the area, the other sentences in the paragraph center on the attentiveness, the
watchfulness, and the awareness of danger that robins possess. So their noisiness seems
to stem from their vigilance, but noisy does not mean the same as vigilant.
D) Incorrect. According to line 14, To stay safe, robins are vigilant. While vigilance and
safety seem to go together, they do not have the same meaning. So the word safe does
not mean the same as vigilant.
E) Incorrect. The paragraph states, To stay safe, robins are vigilant. Although robins are
territorial, they band together for protection from predators. While banding together for
protection is a way of staying safe or to find protection from predators, robins are also
said to be territorial. Being territorial seems to mean something very different from
banding together. Therefore, if birds are vigilant for safety and if they also band together
for safety, vigilant cannot mean the same as territorial.

englishforeveryone.org

10

15

20

25

30

35

40

Name________________
Date________________

Advanced Critical Reading - Biomimetics

Biomimetics is the billiondollar industry which draws inspiration from nature to solve
problems in engineering, medicine, and other fields. While human beings have always had a
natural propensity to observe and try to copy the ingenuity of nature, it is only recently that
biomimetics has taken off as a field of its own.
The Renaissance genius Leonardo da Vinci dedicated himself to observing and recording
natural phenomena half a millennium ago. His superb graphic renditions of plausible flying
machines are based on his direct observations of birds in flight. His renowned Codex Atlanticus,
along with smaller codices, includes renderings of animal musculature, revealing a fascination
with movement. His erudite studies of flowing hair and water manifest his contemplation of the
movement of waves in nature. However, without modern technology, early inventors like da
Vinci were unable to implement their biomimetic insights.
On the other hand, contemporary biomimeticists have access to the technology and
nanotechnology essential to the industry. One tool of enormous utility to todays biomimetics is
the electron microscope. Being able to see and comprehend the nanoscale construction of natural
structures is crucial to synthesizing those miniscule formations.
One person who has taken on the challenge of biomimetics is Robert Fearing, a professor
of electrical engineering at the University of California, Berkeley. A modernday echo of da
Vincis work, Fearings current challenge is to create a biorobotic fly which is small, swift, and
maneuverable enough to deploy on surveillance or rescue missions. Fearing does not aspire to
replicate the fly. Rather, he hopes to isolate the enigmatic natural structures which give it flight,
and perhaps find a simpler solution than the 20 muscles which power a flys wing. Those 20
muscles allow it to make a 90degree turn from straightline flight in under 50 milliseconds,
something even the most advanced planes are not able to accomplish. Some things are just too
mysterious and complicated to be able to replicate, says Fearing.
Engineer Anthony Brennan, professor in the University of Floridas Materials Science
and Engineering Department, learned that the pattern of miniscule diamond shapes on sharks
skin inhibit the growth of algae and other organisms. He created a surface nanotechnology
comprised of billions of microscopic diamondshaped bumps which repel bacteria and other
microorganisms. His biomimetic plastic wrap is being used in hospitals on typically bacteria
laden surfaces such as light switches.
According to MIT chemical engineer Robert Cohen, The natural structure provides a
clue to what is useful in a mechanism. But maybe you can do it better. Cohen recently utilized
nanotechnology to mimic the scales of a desert lizard to produce a water collection device. In a
National Geographic interview, Cohen says, Looking at pretty structures in nature is not
sufficient. What I want to know is, can we actually transform these structures into an
embodiment with true utility in the real world?
Leonardo would have met todays technologies with a receptive mind. In his own time,
he was heralded primarily not as an artist, but as an engineer. His orientation as a biomimeticist
might be summed up in his own quotation concerning nature: Human subtlety will never devise
an invention more beautiful, more simple or more direct than does nature because in her
inventions nothing is lacking, and nothing is superfluous.

Questions
1. According to the passage, how do todays biomimeticists differ from Leonardo da Vinci?
I)

Leonardo did not have the technology to construct his machines when he
envisioned them.
II) Todays biomimeticists aspire to improve upon nature, whereas Leonardo
considered nature to be ultimately better than any manmade invention.
III) Todays biomimeticists have conceived of replacing many natural structures with
manmade structures.
A)
B)
C)
D)
E)

I only
II only
III only
I and II only
II and III only

2. How is Fearings current challenge an echo of da Vincis work?


A)
B)
C)
D)
E)

Fearing is attempting to create a flying machine based on a natural structure.


Fearing hopes to improve upon the flys movement mechanism.
Fearing aspires to create a miniscule technology.
Fearing hopes to simplify modern flight technology.
Fearing is working on electricity.

3. In line 20, enigmatic most closely means


A)
B)
C)
D)
E)

discrete
strategic
pragmatic
inscrutable
determinate

4. Based on the passage, which modern biomimeticist holds views most similar to those of
Leonardo as exemplified in lines 3941, and why?
A) Robert Fearing because he wants to simplify the natural structures that give the fly
flight
B) Robert Fearing because he believes that some things are too complicated to be able to
replicate
C) Anthony Brennan because he replicated a structure found in nature
D) Anthony Brennan because he created a surface technology
E) Robert Cohen because he says, maybe you can do it better.

Answers and Explanations


1. The correct answer is D.
I) Correct. Leonardo did not have the technology to construct his machines when he
envisioned them.
II) Correct. Todays biomimeticists aspire to improve upon nature, whereas Leonardo
considered nature to be ultimately better than any manmade invention.
III) Incorrect. Todays biomimeticists have not conceived of replacing many natural
structures with manmade structures.
2. The correct answer is A.
A) Correct. Fearing is attempting to create a flying machine based on a natural structure, just
as Leonardo wanted to build a flying machine based on the flight of birds.
B) Incorrect. Fearing hopes to improve upon the flys movement mechanism, however da
Vinci did not hope to improve on nature.
C) Incorrect. Fearing aspires to create a miniscule technology, but da Vinci did not aspire to
create a small technology.
D) Incorrect. Fearing hopes to simplify the flys natural flight mechanism.
E) Incorrect. There is no mention that Fearing is working on electricity. Fearing is an
electrical engineer.
3. The correct answer is D.
A) Incorrect. Discrete means separate. Since Fearing hopes to isolate the structures, or make
them separate, discrete cannot be correct.
B) Incorrect. Strategic means planned. Since fearing hopes to isolate these structures, he
does not yet know whether or not they are planned, so strategic cannot be correct.
C) Incorrect. Pragmatic means practical. Since Fearing hopes to simplify these structures, he
seems to believe they are not practical or pragmatic, so pragmatic cannot be correct.
D) Correct. Inscrutable means puzzling. Fearing is hoping to isolate the structures, so he
seems to believe they are puzzling, so enigmatic is correct.
E) Incorrect. Determinate means distinct. Since Fearing hopes to isolate the structures, or
make them determinate or distinct, determinate cannot be correct.
4. The correct answer is C.
A) Incorrect. Robert Fearing because he wants to simplify the natural structures that give
the fly flight, whereas da Vinci believed that nature had the simplest answers.
B) Incorrect. Robert Fearing because he believes that some things are too complicated to
be able to replicate, whereas da Vinci believed that nature had the simplest not
complicated answers.
C) Correct. Anthony Brennan because, like da Vinci, he replicated a structure found in
nature
D) Incorrect There is no mention that Leonardo created a surface technology.
E) Incorrect. Robert Cohen because he says, maybe you can do it better, whereas da
Vinci thought nature had the best answers.

englishforeveryone.org

10

15

20

Name________________
Date________________

Advanced Critical Reading The Great Debates

The first of the Great Debates, between Senator John F. Kennedy of Massachusetts and
the incumbent Vice President Richard Nixon on September 26, 1960, centered around domestic
issues. The topic of the next debate, on October 7, was a clash over U.S. policy regarding two
small islands off the Chinese coast, and on October 13, this controversy continued. On October
21, the final debate, the candidates focused on American/Cuban relations.
Few of the 70 million viewers could have fathomed what this firstever televised
presidential debate augured, not only for this specific series of debates, but more importantly for
the preeminent role the fledgling medium would play in the future of the political arena.
A pallid Nixon arrived at the Chicago CBS studios after a grueling day of campaigning.
The previous August a knee infection had sidelined him. He was still twenty pounds
underweight, and he perspired profusely in an illfitting shirt. Moreover, he declined makeup to
burnish his hospital pallor. The freshlypainted studio backdrop had dried to an ashen hue that
obscured his matching suit.
The Democratic contender by contrast exuded a robust glow after a month of
campaigning in California. He had spent his day rehearsing potential questions and relaxing. An
aide later admitted that he supplemented his natural glow with a smidge of makeup. He was fit,
trim, and confident.
Despite the remarkably similar agendas and arguments of the Republican and the
Democrat, TV viewers unequivocally believed Kennedy to be the victor whereas people who
had followed the debates on the radio held the opposite opinion. The age of TV had arrived, and
the subsequent party shuffle proved the undeniable potency of television.

Questions
1. The author is mainly concerned about
A)
B)
C)
D)
E)

the debating styles of John Kennedy and Richard Nixon during the 1960 Great Debates
the domestic issues which affected the result of 1960 Great Debates
the health of Richard Nixon at the time of the 1960 Great Debates
the number of television viewers who tuned in to the 1960 Great Debates
the effect of television on the results of the 1960 Great Debates

2. It can be inferred from the passage that


A)
B)
C)
D)
E)

Kennedy was a better debater than Nixon


Nixon was the unequivocal winner of the 1960 debates
The Democrat beat the Republican in the 1960 election
Nixon was more prepared for the first debate than Kennedy
Kennedy and Nixon disagreed strongly on issues on the home front.

3. According to the passage, which of the following was true of Richard Nixon?
A)
B)
C)
D)
E)

He had a five oclock shadow during the first debate.


He wore a brown suit during the first debate.
He warned of the impending Cuban crisis.
He limped onstage for the first debate.
He lost his job after the election.

Answers and Explanations


1. The correct answer is E.
A) The debating styles of John Kennedy and Richard Nixon during the 1960 Great Debates
were similar, since TV viewers and radio listeners ended up with different opinions of
who won the debates.
B) The candidates had similar agendas and arguments, so domestic issues were not pivotal.
C) Richard Nixon was not sick at the time of the 1960 Great Debates. He was thin and pale,
but there is no mention that he was sick.
D) The number of television viewers who tuned in to the 1960 Great Debates was not the
authors main concern.
E) Correct. The effect of television on the results of the 1960 Great Debates was the main
concern of the author
2. The correct answer is C.
A) Kennedy was not a better debater than Nixon: people who followed the debates on radio
thought Nixon had won the debates.
B) Nixon was not the unequivocal winner of the 1960 debates; people who watched the
debates on TV thought Kennedy was the winner.
C) Correct. The Democrat beat the Republican in the 1960 election; there was a party
shuffle. This is mentioned in the last line (21) of the passage. Since Nixon, the
Republican, was the incumbent, the shuffle resulted in Democrats taking office.
D) There is no mention of whether Nixon was more prepared for the first debate than
Kennedy. Kennedy rehearsed the day of the debate.
E) Kennedy and Nixon did not disagree strongly on issues on the home front. They had
similar agendas and arguments.
3. The correct answer is E.
A) He had a five oclock shadow during the first debate. While this fact is widely known, it
is not mentioned in the passage.
B) He did not wear a brown suit during the first debate. His suit was obscured by the ashen
colored gray paint.
C) Whether Nixon warned of the impending Cuban crisis was not mentioned in the passage.
D) Whether Nixon limped is not mentioned in the passage.
E) Correct. Nixon lost his job after the election. Since there was a party shuffle, the
incumbent lost his job. The incumbent Vice President was Richard Nixon.

englishforeveryone.org

10

15

Name________________
Date________________

Beginning Critical Reading Soaps and Detergents

Soaps and detergents are used for washing. Soaps and detergents are used for cleaning.
People usually use detergent to wash clothes. People usually use detergent to wash dishes.
People usually use soap to wash their bodies.
Soap has a long history. There was soap in Ancient Babylon. There was soap in Ancient
Egypt. Some people think there was soap before people wrote history.
Soap and detergent are similar, but soap and detergent are not exactly the same. Soaps are
made of natural products. Detergents are made of man-made products.
In some cases, soap is better than detergent. For example, soap is milder on the skin.
Soap is milder on the environment. Soap is biodegradable natures processes clean soap up.
Soap does not build up in rivers. Soap does not cause pollution in rivers. Soap does not build up
in streams. Soap does not cause pollution in streams.
In some cases, detergent is better than soap. For example, soap builds up in clothes after
many washings. Detergent does not build up in clothes after many washings. Soap loses its
cleaning power in clothes over time. Detergent does not lose its cleaning power in clothes over
time.

Questions
1. According to the passage, which of the following statements is/are true?
I)
Soap is made of natural or man-made products.
II) Detergents are less mild than soaps.
III) Detergent washes out of clothing.
IV)
A)
B)
C)
D)
E)

I only
II only
III only
I and II only
II and III only

2. What is the main idea of the fifth paragraph?


A)
B)
C)
D)
E)

How soaps and detergents are made


What soap and detergent are used for
Which product causes less pollution
How soap loses its cleaning power
When detergent is better than soap

3. In line 9, biodegradable most closely means


A)
B)
C)
D)
E)

man-made
dangerous
permanent
easy to clean up
hard to clean up

Answers and Explanations


1. The correct answer is E.
I)

Incorrect. Line 7 states that Soaps are made of natural products. Detergents are
made of man-made products.
II) Correct. Lines 8-9 state that For example, soap is milder on the skin. Soap is
milder on the environment. Since soap is milder than detergent, detergent is less
mild than soap.
III) Correct. Line 14 states that Detergent does not build up in clothes after many
washings. Since detergent does not build up, or stay, in clothing, it must wash out.
A)
B)
C)
D)
E)

I only
II only
III only
I and II only
II and III only

2. The correct answer is E.


A) Incorrect. Line 7 in the third paragraph explains how soap and detergent are made of
different products, when it states that Soaps are made of natural products. Detergents are
made of man-made products, the fifth paragraph gives examples of ways in which
detergent is sometimes better than soap. For example, the fifth paragraph states that
Soap loses its cleaning power in clothes over time. Detergent does not lose its cleaning
power in clothes over time.
B) Incorrect. While lines 1-2 explain the uses of soaps and detergents when they state that
Soaps and detergents are used for washing. Soaps and detergents are used for cleaning,
the fifth paragraph gives examples of ways in which detergent is sometimes better than
soap. For example, the fifth paragraph states that Soap loses its cleaning power in
clothes over time. Detergent does not lose its cleaning power in clothes over time.
C) Incorrect. While lines 9-11 explain that soap causes less pollution when they state that
Soap is milder on the environment. Soap is biodegradable natures processes clean
soap up. Soap does not build up in rivers. Soap does not cause pollution in rivers. Soap
does not build up in streams. Soap does not cause pollution in streams, the fifth
paragraph gives examples of ways in which detergent is sometimes better than soap. For
example, the fifth paragraph states that Soap loses its cleaning power in clothes over
time. Detergent does not lose its cleaning power in clothes over time.
D) Incorrect. While line 14 states that Soap loses its cleaning power in clothes over time, it
does not tell how soap loses its cleaning power.
E) Correct. The fifth paragraph gives examples of ways in which detergent is sometimes
better than soap. For example, the fifth paragraph states that Soap loses its cleaning
power in clothes over time. Detergent does not lose its cleaning power in clothes over
time.

3. The correct answer is D.

A) Incorrect. Since line 7 states that Soaps are made of natural products, and line 9 states
that Soap is biodegradable, the word biodegradable cannot mean man-made.
B) Incorrect. Since lines 8-9 state that soap is milder on the skin. Soap is milder on the
environment, and line 9 states that Soap is biodegradable, the word biodegradable
cannot mean dangerous.
C) Incorrect. Since lines 9-10 state that natures processes clean soap up, the word
biodegradable cannot mean permanent.
D) Correct. Since lines 9-10 state that natures processes clean soap up, the word
biodegradable probably means easy to clean up.
E) Incorrect. Since lines 9-10 state that natures processes clean soap up, the word
biodegradable probably doesnt mean hard to clean up.

READTHEORY
Name________________
Date________________

Reading Comprehension 1

Level 6

Directions: Read the passage. Then answer the questions below.


When we are young, we learn that tigers and sharks are dangerous animals. We might be scared
of them because they are big and powerful. As we get older, however, we learn that sometimes the most
dangerous animals are also the smallest animals. In fact, the animal that kills the most people every year
is one that you have probably killed yourself many times: the mosquito.
While it may seem that all mosquitoes are biters, this is not actually the case. Male mosquitoes
eat plant nectar. On the other hand, female mosquitoes feed on animal blood. They need this blood to live
and produce eggs. When a female mosquito bites a human being, it transmits a small amount of saliva
into the blood. This saliva may or may not contain a deadly disease. The result of the bite can be as
minor as an itchy bump or as serious as death.
Because a mosquito can bite many people in the course of its life, it can carry diseases from one
person to another very easily. Two of the most deadly diseases carried by mosquitoes are malaria and
yellow fever. More than 700 million people become sick from these diseases every year. At least 2 million
of these people will die from these diseases.
Many scientists are working on safer and better ways to kill mosquitoes, but so far, there is no
sure way to protect everyone in the world from their deadly bites. Mosquito nets can be placed over beds
to protect people against being bitten. These nets help people stay safe at night, but they do not kill any
mosquitoes. Mosquitoes have many natural enemies like bats, birds, dragonflies, and certain kinds of
fish. Bringing more of these animals into places where mosquitoes live might help to cut down the amount
of mosquitoes in that area. This is a natural solution, but it does not always work very well. Mosquitoes
can also be killed with poisons or sprays. Even though these sprays kill mosquitoes, they may also harm
other plants or animals.
Although mosquitoes may not seem as scary as larger, more powerful animals, they are far more
dangerous to human beings. But things are changing. It is highly likely that one day scientists will find a
way to keep everyone safe from mosquitoes and the diseases they carry.

Questions
1) According to the author, some people are more afraid of tigers and sharks than mosquitoes because
tigers and sharks
A.
B.
C.
D.

kill more people than mosquitoes


are big and powerful
are found all over the world
have no natural enemies

2) Based on the information in paragraph 2, we can understand that


I. male mosquitoes and female mosquitoes have different eating habits
II. male mosquitoes are harmless to humans
III. female mosquitoes are responsible for transmitting diseases to humans
A.
B.
C.
D.

I only
I and II only
II and III only
I, II, and III
Copyright Read Theory LLC, 2012. All rights reserved.

3) In paragraph 2 the author writes, "This saliva may or may not contain a deadly disease." The purpose
of this statement is to
A.
B.
C.
D.

oppose a previous argument


question an upcoming conclusion
confirm a hypothesis
support a later statement

4) As used in paragraph 2, minor most nearly means


A.
B.
C.
D.

insignificant
deadly
frustrating
dangerous

5) Based on information in paragraph 3, it can be understood that if you get sick with malaria or yellow
fever, your chances of survival are
A.
B.
C.
D.

terrible
mediocre
good
excellent

6) It can be understood that the introduction of dragonflies might reduce the number of flies in a given
area because dragonflies
A.
B.
C.
D.

work together with mosquitoes


kill mosquitoes
cannot be killed by poisons or sprays
attract bats

7) Which of the following best summarizes the information in paragraph 4?


A. Mosquito nets provide adequate protection from deadly mosquitoes.
B. Poisons and sprays provide adequate protection from deadly mosquitoes.
C. The introduction of the mosquito's natural enemies provides adequate protection from deadly
mosquitoes.
D. There is no perfect solution to the mosquito problem.
8) Which of the following words best describes the author's overall attitude towards the prospect of
solving the mosquito problem?
A.
B.
C.
D.

despondent, meaning hopeless or dejected


exasperated, meaning extremely irritated or annoyed
equivocal, meaning doubtful or uncertain
optimistic, meaning hopeful or taking a favorable view

Copyright Read Theory LLC, 2012. All rights reserved.

Answers and Explanations


1) B
In the first paragraph the author says, "We might be scared of them because they are big and powerful." In the last paragraph, the
author says that we might be more afraid of tigers and sharks than we are of mosquitoes because tigers and sharks are "larger, and
more powerful." This lets us know that (B) is correct. In paragraph 1, the author tells us that of all animals, the mosquito "kills the
most people every year." This eliminates (A). The passage does not contain information to support choices (C) and (D). Therefore
they are incorrect.
2) D
In paragraph 2 the author writes, "Male mosquitoes eat plant nectar. On the other hand, female mosquitoes feed on animal blood."
Using this information, we can understand that male mosquitoes and female mosquitoes have different eating habits. This supports
option (I). In paragraph 2 the author writes, "Male mosquitoes eat plant nectar. On the other hand, female mosquitoes feed on
animal blood. They need this blood to live and produce eggs. When a female mosquito bites a human being, it transmits a small
amount of saliva into the blood. This saliva may or may not contain a deadly disease." Since male mosquitoes eat plant nectar (and
not animal blood), and it is the bite that causes the transmission of diseases, we can understand that male mosquitoes do not bite
humans. Therefore they are harmless to humans. This supports option (II). In paragraph 2 the author writes, "Male mosquitoes eat
plant nectar. On the other hand, female mosquitoes feed on animal blood. They need this blood to live and produce eggs. When a
female mosquito bites a human being, it transmits a small amount of saliva into the blood. This saliva may or may not contain a
deadly disease." Since male mosquitoes eat plant nectar (and not animal blood), and it is the bite that causes the transmission of
diseases, we can understand that female mosquitoes are responsible for transmitting diseases to humans. This supports option
(III). Therefore (D) is correct.
3) D
To answer this question correctly, it helps to use context. At the end of paragraph 2 the author writes, "This saliva may or may not
contain a deadly disease." In the next sentence the author writes, "The result of the bite can be as minor as an itchy bump or as
serious as death." Using this information, we can understand that the reason why the result of the bite can be as minor as an itchy
bump or as serious as death is because the mosquito's saliva may or may not contain a deadly disease. This lets us know that the
purpose of the statement in question is to support a later statement, which, in this case, is the next sentence. Choice (D) is correct.
The passage does not provide information to support choices (A), (B). and (C). Therefore they are incorrect.
4) A
minor (adjective): relatively small in size, quantity, or degree.
In paragraph 2, the author says that when a mosquito bites a person, The result of the bite can be as minor as an itchy bump or as
serious as death. Here, the author contrasts an itchy bump with death. An itchy bump is not very serious when compared to death.
An itchy bump is a small, insignificant problem, while death is aptly described as "serious." This makes (A) the correct choice.
Although itchy bumps can be frustrating, this is not as good a contrast for "serious" as insignificant. Therefore (C) is incorrect. (B)
and (D) are incorrect because the author contrasts the itchy bumps with deadly ones. The author means to show that an itchy bump
is harmless, not deadly or dangerous.
5) D
At the end of paragraph 3 the author writes, "More than 700 million people become sick from these diseases every year. At least 2
million of these people will die from these diseases." This lets us know that of the 700 million people who become sick, only around
2 million do not survive. Because 2 million is an extremely small percentage of 700 million (2/700 or .002%), we can understand that
if you get sick with malaria or yellow fever, your chances of survival are excellent. This means (D) is correct. The passage does not
provide information to support choices (A), (B). and (C). Therefore they are incorrect.
6) B
In paragraph 4, the author says that, Mosquitoes have many natural enemies like bats, birds, dragonflies, and certain kinds of fish.
Bringing more of these animals into places where mosquitoes live might help to cut down the amount of mosquitoes in that area.
(B) is the only choice that uses information supported by the passage to provide a reason why dragonflies would reduce (or make
less) the number of mosquitoes in a given area. Therefore it is correct. The passage does not provide information to support choices
(A), (C), and (D). Therefore they are incorrect.
7) D
In paragraph 4, the author writes, "Many scientists are working on safer and better ways to kill mosquitoes, but so far, there is no
sure way to protect everyone in the world from their deadly bites." The author continues to discuss possible solutions and their
respective drawbacks. This lets us know there is not perfect solution to the mosquito problem. Choice (D) is correct. The passage
does not provide information to support choices (A), (B), and (C). Therefore they are incorrect.
8) D
In the final paragraph the author concludes the passage by saying, "But things are changing. It is highly likely that one day scientists
will find a way to keep everyone safe from mosquitoes and the diseases they carry." This lets us know that although the problem
may look grim now, there is a good chance that scientists will be able to solve it in the future. Thus, the author's attitude towards the
prospect of solving the mosquito problem can best be described as optimistic. Therefore (D) is correct. Despondent and
exasperated are too negative to accurately describe the author's attitude. Therefore choices (A) and (B) are incorrect. The author
does not express uncertainty about the prospect of solving the mosquito problem. Therefore the author is not equivocal. Choice (C)
is incorrect.

Copyright Read Theory LLC, 2012. All rights reserved.

READTHEORY
Name________________
Date________________

Reading Comprehension 2

Level 6

Directions: Read the passage. Then answer the questions below.


It is bad to have food stuck between your teeth for long periods of time. This is because food
attracts germs, germs produce acid, and acid hurts your teeth and gums. Flossing helps to remove the
food that gets stuck between your teeth. This explains why flossing helps to keep your mouth healthy, but
some doctors say that flossing can be also good for your heart.
It may seem strange that something you do for your teeth can have any effect on your heart.
Doctors have come up with a few ideas about how flossing works to keep your heart healthy. One idea is
that the germs that hurt your teeth can leave the mouth and travel into your blood. Germs that get into the
blood can then attack your heart. Another idea is based on the fact that when there are too many germs
in your mouth, the body tries to fight against these germs. For some reason, the way the body fights
these mouth germs may end up weakening the heart over time.
Not every doctor agrees about these ideas. Some doctors think that the link between good
flossing habits and good heart health is only a coincidence. A coincidence is the occurrence of two or
more events at one time apparently by mere chance. The incidence of these events is completely
random, as they do not admit of any reliable cause and effect relationship between them. For example,
every time I wash my car, it rains. This does not mean that when I wash my car, I somehow change the
weather. This is only a coincidence. Likewise, some doctors think that people who have bad flossing
habits just happen to also have heart problems, and people who have good flossing habits just happen to
have healthy hearts.
The theory that flossing your teeth helps to keep your heart healthy might not be true. But every
doctor agrees that flossing is a great way to keep your teeth healthy. So even if flossing does not help
your heart, it is sure to help your teeth. This is enough of a reason for everyone to floss their teeth every
day.

Questions
1) Which of the following would be the best title for this passage?
A.
B.
C.
D.

Why Doctors Disagree about Flossing


How to Keep Your Teeth Healthy
Flossing Your Way to a Healthy Heart
Flossing by Coincidence

2) Flossing effectively helps to keep your mouth healthy by preventing


A.
B.
C.
D.

germs from producing acid


food from entering your body
germs from entering into your blood
acid from contacting your teeth and gums

3) In paragraph 2, the author introduces ideas about how flossing works to keep your heart healthy.
Exactly how many of these ideas does the author put forth in this paragraph?
A.
B.
C.
D.

1
2
3
4
Copyright Read Theory LLC, 2012. All rights reserved.

4) Based on information in paragraph 2, it can be understood that germs in the mouth may harm your
heart by
I. getting into the blood that flows to the heart
II. forcing the body to fight against too many of them
III. causing food to get stuck in the arteries
A.
B.
C.
D.

l only
I and II only
II and III only
I, II, and III

5) In paragraph 2, the author explains how having too many germs in your mouth can "end up
weakening the heart." Using the passage as a guide, it can be understood that with respect to the
actual way in which this occurs, doctors are
A.
B.
C.
D.

reluctant to hypothesize
confident in their estimations
extremely knowledgeable
uncertain but speculative

6) In paragraph 3 the author writes, "Not every doctor agrees about these ideas." The author's purpose
in writing this sentence is to
A.
B.
C.
D.

provide an example
introduce a new topic
change a previous statement
clarify an earlier assertion

7) Using information in paragraph 3 as a guide, which of the following is the best example of a
coincidence?
A. Jim wakes up with a sore throat. He eats a piece of bacon for breakfast. By noon, he feels much
better. Jim decides that the bacon has cured his sore throat.
B. Laura remembers to brush her teeth every day, but she only remembers to floss once a week.
She writes a note to herself, reminding herself to floss and sticks it to her bathroom mirror.
C. Mario is not very good at baseball. He practices playing every day. After a several months of
practice, he is a much better baseball player.
D. Jai has a bad heart. Her doctor tells her to eat more vegetables and less junk food. After nearly a
year of doing this, the doctor tells Jai that her heart is doing much better.
8) Based on its use in the final paragraph, it can be inferred that theory belongs to which of the
following word groups?
A.
B.
C.
D.

query, question, interrogation


assertion, declaration, affirmation
hypothesis, supposition, belief
idea, thought, notion

9) Which of the following best states the main idea of the final paragraph?
A.
B.
C.
D.

Because doctors do not agree that flossing will help your heart, it is useless to floss.
It is a fact that flossing can help your heart as well as your teeth.
Even if flossing is only good for your teeth, you should still do it every day.
There is no good reason to believe that flossing will help your heart, but it is still a good idea to do
it every day.
Copyright Read Theory LLC, 2012. All rights reserved.

Answers and Explanations


1) C
A good title summarizes the main idea of the passage, letting the reader know what to expect should he or she continue reading.
The main idea of this passage is that flossing may be linked to having a healthy heart. This idea is introduced at the end of the first
paragraph, when the author states, "This explains why flossing helps to keep your mouth healthy, but some doctors say that flossing
can be also good for your heart." In paragraph 2, the author gives possible reasons for this idea. In paragraph 3, the author talks
about how flossing and heart health may be a coincidence. In the final paragraph, the author gives concluding remarks about the
benefits of flossing. Using this information, we can understand that the best title for the passage would be Flossing Your Way to a
Healthy Heart, as it best summarizes the main idea of the passage. Choice (C) is correct. Even though the passage does mention
that not all doctors agree about whether or not flossing is good for your heart, and that flossing is good for your teeth, these are not
the main ideas. They are merely details the author uses to support a larger idea: that flossing is good for your teeth and may also be
good for your heart. Therefore (A) and (B) are incorrect. The author does not state the idea that flossing occurs by coincidence.
Rather, the author uses the idea of coincidence to illustrate why some "doctors think that the link between good flossing habits and
good heart health,"an idea explained in paragraph 3. This lets us know that choice (D) is incorrect.
2) D
In paragraph 1, the author tells us that "food attracts germs, germs produce acid, and acid hurts your teeth and gums." The author
continues to tell us that "Flossing helps to remove food that gets stuck between your teeth." Using this information, we can
understand that flossing helps remove food from between your teeth. This, in turn, removes germs from your mouth. If there are no
germs in your mouth, the acid they produce cannot get on your teeth and gums. If the acid cannot get on your teeth and gums, it
cannot harm them. This is how flossing effectively helps to keep your mouth healthy. This idea is reflected in (D), making it the
correct choice. Although flossing helps prevent food from attracting germs, it does not prevent germs from producing acid. Germs
will produce acid regardless of flossing. Flossing simply makes sure that germs don't produce acid in your mouth. This lets us know
that flossing does not help to keep your mouth healthy by preventing germs from producing acid. This means (A) is incorrect.
Technically speaking, flossing may indeed prevent miniscule pieces of food from entering your body. However, this is unrelated to
the way in which it helps you teeth. This lets us know that flossing does not help to keep your mouth healthy by preventing food from
entering your body. Choice (B) is incorrect. The author does not discuss the possibility that germs will enter the blood. Rather, the
author says that acid may enter the blood. However, this is unrelated to the way in which flossing helps your teeth. This lets us know
that flossing does not help to keep your mouth healthy by preventing germs from entering your blood. Choice (C) is incorrect.
3) B
In paragraph 2, the author introduces two ideas about how flossing works to keep your heart healthy. The first idea is introduced as
the author writes, "One idea is that the germs that hurt your teeth can leave the mouth and travel into your blood." The second idea
is introduced as the author writes, "Another idea is based on the fact that when there are too many germs in your mouth, the body
tries to fight against these germs." This means (B) is correct. The passage does not provide information to support choices (A), (C),
and (D). Therefore they are incorrect.
4) B
In paragraph 2, the author introduces two ideas about how flossing works to keep your heart healthy. First, the author says, "One
idea is that the germs that hurt your teeth can leave the mouth and travel into your blood. Germs that get into the blood can then
attack your heart." This supports option (I). Second, the author says, "Another idea is based on the fact that when there are too
many germs in your mouth, the body tries to fight against these germs. For some reason, the way the body fights these mouth
germs may end up weakening the heart over time." This supports option (II). The author does not provide information to suggest
that germs may harm your heart by causing food to get stuck in the arteries. This eliminates option (III). Therefore (B) is correct.
5) D
At the end of paragraph 2, the author writes, "Another idea is based on the fact that when there are too many germs in your mouth,
the body tries to fight against these germs. For some reason, the way the body fights these mouth germs may end up weakening
the heart over time." After explaining how having too many germs in your mouth causes the body to fight against them, the author
introduces the explanation behind this using the phrase, "For some reason." This phrase is used to signify that doctors are uncertain
about the actual cause of this. Nonetheless, they venture to speculate that it has something to do with the way the body fights these
mouth germs and how that may end up weakening the heart over time. Using this information, we can understand that doctors are
uncertain but speculative with respect to the actual way in which this occurs. Choice (D) is correct. Although doctors may be
reluctant, they still venture to speculate, or hypothesize, about the actual way in which this occurs by positing that "the way the body
fights these mouth germs may end up weakening the heart over time." This lets us know that (A) is incorrect. Choices (B) and (C)
depict the doctors as more confident in their suppositions than the passage leads us to believe. Therefore they are incorrect.
6) D
To answer this question correctly, it helps to use context. In paragraph 2 the author writes, "Doctors have come up with a few ideas
about how flossing works to keep your heart healthy." The author continues to describe these ideas in the remainder of paragraph 2.
At the beginning of paragraph 3, the author tells us that "Not every doctor agrees about these ideas." This lets us know that the
author plans to use paragraph 3 to clarify his or her earlier assertion presented in paragraph 2. This means (D) is correct. The
passage does not provide information to support choices (A), (B), and (C). Therefore they are incorrect.
7) A
coincidence (noun): a striking occurrence of two or more events at one time apparently by mere chance.
In paragraph 3, the author tells us that A coincidence is the occurrence of two or more events at one time apparently by mere
chance. The incidence of these events is completely random, as they do not admit of any reliable cause and effect relationship

Copyright Read Theory LLC, 2012. All rights reserved.

between them. In Jims case, Jim eats bacon and his sore throat happens to go away. These two events do not admit of any
reliable cause and effect relationship. Jim's eating of the bacon (cause) does not mean that bacon cured his sore throat (effect).This
is a single occurrence that intuitively seems very unlikely to happen again. This fits the author's description of a coincidence.
Therefore choice (A) is correct. (B) is unrelated to the idea of a coincidence and is therefore incorrect. In choice (C), Marios
practice makes him a better player. A reliable cause (several months of practice) and effect (getting better at baseball) relationship
is readily identifiable. This is not a coincidence. In choice (D), Jais healthier diet makes her heart healthier. A reliable cause (nearly
a year of eating more vegetables and less junk food) and effect (her heart is doing much better) relationship is readily identifiable.
These effects are directly linked to their causes.
8) C
theory (noun): a proposed explanation whose status is still conjectural and subject to experimentation, in contrast to wellestablished propositions that are regarded as reporting matters of actual fact.
To answer this question correctly, it helps to use context. In the final paragraph, the author writes, "The theory that flossing your
teeth helps to keep your heart healthy might not be correct. Nonetheless, every doctor agrees that flossing is a great way to keep
your teeth healthy. So even if flossing does not help your heart, it is sure to help your teeth." Using this text, we can understand that
the idea that flossing your teeth helps to keep your heart healthy is a theory. We also learn that this idea may or may not be correct.
So, this idea is a speculative, or uncertain, attempt to state a fact about the world or how it works. A theory is not merely an idea, it
is a speculative assertion. The words hypothesis, supposition, belief capture this definition nicely. This lets us know that choice (C)
is correct. In his or her use of the word theory, the author does not pose a query, question, or interrogation. Rather, the author
makes a speculative assertion, as explained above. This means (A) is incorrect. In his or her use of the word theory, the author
does not make an assertion, declaration, or affirmation. Rather, the author makes a speculative assertion, as explained above. This
is different than a straightforward assertion, which is a simple claim that is not intended to be speculative. This means (B) is
incorrect. In his or her use of the word theory, the author does not illustrate an idea, thought, or notion. Given the above explanation,
we can understand that this idea is a speculative, or uncertain, attempt to state a fact about the world or how it works. A theory is
not merely an idea, it is a speculative assertion. This means (D) is incorrect.
9) C
In the final paragraph, the author says that even if flossing does not help your heart, it is sure to help your teeth. This is enough of a
reason for everyone to floss their teeth every day. This statement is intended to mean that while it may be true that flossing might
not really help your heart, it definitely is good for your teeth. Therefore, the author recommends that you should do it every day.
Choice (C) is correct. The passage does not provide information to support choices (A) or (B). Therefore they are incorrect. (D) is
incorrect because the author does not suggest that you should floss merely because it may be good for your heart. Rather, the
author says that flossing is good for your teeth, and that the fact that it may also be good for your heart is a potential added benefit.

Copyright Read Theory LLC, 2012. All rights reserved.

READTHEORY
Name________________
Date________________

Reading Comprehension 3

Level 6

Directions: Read the passage. Then answer the questions below.

When you imagine the desert, you probably think of a very hot place covered with sand. Although
this is a good description for many deserts, Earths largest desert is actually a very cold place covered
with ice: Antarctica.
In order for an area to be considered a desert, it must receive very little rainfall. More specifically,
it must receive an average of less than ten inches of precipitationwhich can be rain, sleet, hail, or
snowon the ground every year. Antarctica, the coldest place on earth, has an average temperature that
usually falls below the freezing point. And because cold air holds less moisture than warm air, the air in
Antarctica does not hold much moisture at all. This is evident in the low precipitation statistics recorded
for Antarctica. For example, the central part of Antarctica receives an average of less than 2 inches of
snow every year. The coastline of Antarctica receives a little bit morebetween seven and eight inches a
year. Because Antarctica gets so little precipitation every year, it is considered a desert.
When precipitation falls in hot deserts, it quickly evaporates back into the atmosphere. The air
over Antarctica is too cold to hold water vapor, so there is very little evaporation. Due to this low rate of
evaporation, most of the snow that falls to the ground remains there permanently, eventually building up
into thick ice sheets. Any snow that does not freeze into ice sheets becomes caught up in the strong
winds that constantly blow over Antarctica. These snow-filled winds can make it look as if it is snowing.
Even though snowfall is very rare there, blizzards are actually very common on Antarctica.

Questions

1) The main purpose of paragraph 1 is to


A.
B.
C.
D.

accept a conclusion
introduce an argument
provide a brief history
deny a common belief

2) The best title for this passage would be


A.
B.
C.
D.

Earths Many Deserts


Antarctica: The Coldest Place on Earth
A Desert of Ice
Unusual Blizzards

3) Africas Sahara Desert is the second-largest desert on earth. Based on the information in the
passage, what characteristic must the Sahara share with Antarctica?
A.
B.
C.
D.

low temperatures
high temperatures
frequent blizzards
low precipitation

Copyright Read Theory LLC, 2012. All rights reserved.

4) As used in paragraph 2, which is the best definition for precipitation?


A.
B.
C.
D.

moisture in the air that falls to the ground


any type of weather event
weather events that only happen in very cold areas
a blizzard that occurs in areas with limited snowfall

5) In paragraph 2 the author writes, "And because cold air holds less moisture than warm air, the air in
Antarctica does not hold much moisture at all." Using this information, it can be understood that
A.
B.
C.
D.

air in Africa holds more moisture than the air in Antarctica


air surrounding a tropical island holds less moisture than the air in Antarctica
air in the second floor of a house is typically warmer than air on the first floor
air at the mountains is typically colder than the air at the beach

6) According to the final paragraph, any snow that falls over Antarctica
I. becomes part of the Antarctic ice sheet
II. is blown around by strong winds
III. evaporates back into the atmosphere
A.
B.
C.
D.

l only
I and II only
II and III only
I, II, and III

7) Based on the information in the final paragraph, it can be understood that blizzards in Antarctica are
mainly the result of
A.
B.
C.
D.

freezing cold temperatures


large amounts of snowfall
low amounts of precipitation
strong winds

Copyright Read Theory LLC, 2012. All rights reserved.

Answers and Explanations


1) D
In paragraph 1 the author writes, "When you imagine the desert, you probably think of a very hot place covered with sand." This lets
us know that the author thinks that deserts are commonly believed to be very hot places covered with sand. However, as the author
continues to explain, this is not actually the case, stating "Many other deserts, like Antarctica, are just the opposite." Using this
information, we can understand that the main purpose of paragraph 1 is to deny a common belief. This means (D) is correct. The
author does not mention any conclusion. Therefore (A) is incorrect. The author does not argue the idea that Antarctica is a desert.
This is factual information that is documented to be true. Choice (B) is incorrect. The author does not provide any history. This
means (C) incorrect.
2) C
A good title summarizes the main idea of a passage and lets the reader know what to expect should he or she read further. This
passage is mostly devoted to explaining why Antarctica is a desert of ice, which most people may find surprising, because deserts
are stereotypically hot and sandy. This idea is introduced in the first paragraph. The subsequent paragraphs are devoted to
explaining this idea. Therefore (C) is correct. In this passage, the author presents information about Antarctica only. It does not
discuss any other of earth's deserts. The title in (A) is too general for a passage that is just about Antarctica, making this choice
incorrect. Although the passage does mention the fact that Antarctica is the coldest place on earth, and discusses Antarcticas
blizzards, these are details. The main idea of the passage involves the fact that Antarctica is a desert. Therefore (B) and (D) are
incorrect.
3) D
In paragraph 2, the author writes: In order for an area to be considered a desert, it must receive very little precipitation. More
specifically, it must receive an average of less than ten inches of precipitationwhich can be rain, sleet, hail, or snowon the
ground every year." Like Antarctica, the Sahara is a desert. Therefore, it must also have low precipitation. This means (D) is correct.
The passage does not provide information to support choices (A), (B), and (C). Therefore they are incorrect.
4) A
precipitation (noun): moisture in the air that falls to the ground.
In paragraph 2, the author states that precipitation can be rain, sleet, hail, or snow. All of these are forms of air moisture that falls
to the ground. Using this information, we can understand that (A) is the correct choice. Although rain, sleet, hail, and snow are all
types of weather events, the phrase any type of weather is too general, as it may include other types of weather such as high wind,
clouds, heat, humidity, etc. Therefore (B) is incorrect. Even though it is very cold in Antarctica, rain can fall in any climate. This
means precipitation isnt specific to very cold areas. This eliminates (C). (D) is incorrect because a blizzard that occurs in areas with
limited snowfall is only something that happens in extremely cold, windy places like Antarctica, and is not representative of
precipitation in general.
5) A
In paragraph 2 the author writes, "And because cold air holds less moisture than warm air, the air in Antarctica does not hold much
moisture at all." Because Africa is widely known to have a warm, if not hot, climate, we can understand that the air in Africa holds
more moisture than the freezing cold air in Antarctica. Therefore (A) is correct. (B) is incorrect because a tropical island has hot
climate. This means it should hold more moisture than the air in Antarctica. Although choices (C) and (D) may provide accurate
information, they are incorrect because the quote from paragraph 2 does not discuss how air temperature is related to geographical
location.
6) B
In the final paragraph, the authors tells us that any snow that falls over Antarctica either remains there permanently, eventually
building up into thick ice sheets. or becomes caught up in the strong winds." This supports options (I) and (II). In the beginning of
this paragraph, the author says the air over Antarctica is too cold to hold water vapor, so there is very little evaporation. This
eliminates option (III). Therefore (B) is correct.
7) D
In the final paragraph, the author informs us that the blizzards in Antarctica are caused when strong winds blow snow around.
Although there is a very low amount of precipitation that falls over Antarctica, some of the snow that does fall becomes caught up in
the strong winds that constantly blow over Antarctica. Because of these high winds, even though snowfall is very rare there,
blizzards are actually very common on Antarctica. Using this information, we can understand that (D) is correct The passage does
not provide information to support choices (A), (B), and (C). Therefore they are incorrect.

Copyright Read Theory LLC, 2012. All rights reserved.

READTHEORY
Name________________
Date________________

Reading Comprehension 4

Level 6

Directions: Read the passage. Then answer the questions below.


Mona doesnt like to ask people for help. But it is hard for her to perform daily activities
on her own. She is almost 13, yet she is no larger than a 5-year-old. Mona has trouble keeping
her balance and cant walk very far. When she uses a wheelchair, she cant push it herself.
Fortunately, Mona has a wonderful service dog named Sam. A service dog is a dog that
has been trained to assist someone who has a physical problem. Sam lets Mona lean on him
when she walks. He also pulls her wheelchair and turns lights on and off. When Mona drops
something, Sam picks it up. He even pulls her socks off at night.
Sam also helps Mona with everyday tasks at school. He carries her books from class to class in
a special backpack. He puts Monas completed assignments in her teachers homework trays.
In the lunchroom he throws away her trash.
Besides making Mona less dependent on other people, Sam helps her lead a fuller life.
Monas classmates flock around Sam like geese. This has helped her make friends. Sam also
helps Mona be more active. With his aid, she raised over $500 in a walk-a-thon for her local
humane society.
Because of Sam, Mona doesnt have to ask people for help. Sam brings her closer to
other kids. And he even helps her contribute to her community.

Copyright Read Theory LLC, 2012. All rights reserved.

Questions

1) Which of the following would be the best title for this passage?
A.
B.
C.
D.

Why Mona Loses Her Balance


How Monas Service Dog Helps Her
Sam Helps Mona at School
Raising Money for the Humane Society

2) Using the passage as a guide, which of the following dogs is most likely a service dog?
A.
B.
C.
D.

Frank's dog, who turns on the lights when Frank enters the room.
Raul's dog, who fetches the newspaper for Raul while he is busy getting dressed.
Mei's dog, who licks Meis face when she cries.
Teddy's dog, who loves to play catch, go on walks, and watch movies with the family.

3) According to the passage, Sam helps Mona by


I. helping her to walk
II. performing everyday tasks for her
III. bringing her closer to her classmates
A.
B.
C.
D.

I only
I and II only
II and III only
I, II, and III

4) In paragraph 4, the author writes, Monas classmates flock around Sam like geese. Which
of the following literary techniques is used in this sentence?
A. allusion, characterized by a reference to, or representation of, people, places, events,
literary work, myths, or works of art, either directly or by implication
B. personification, characterized by giving human traits to nonhuman things, such as
animals or objects
C. irony, characterized by the use of words to express the opposite of their usual meaning
D. simile, characterized by the making of a comparison using the words like or as

Copyright Read Theory LLC, 2012. All rights reserved.

Answers and Explanations


1) B
A good title summarizes the main idea of the passage and lets the reader know what to expect should he or she read further. In
paragraph 1, the author illustrates Mona's need for assistance with routine activities. In the beginning of paragraph 2, the author
tells us that Mona has a service dog and explains that a service dog helps someone who has a physical problem. In paragraph 3,
the author shows how Monas service dog helps her lead a fuller life. In the final paragraph, the author sums up the main ways that
Monas service dog helps her. Using this information, we can understand that the main idea of this passage is to explain how
Monas service dog, Sam, helps her. Therefore, the best title for the passage would be How Monas Service Dog Helps Her, since it
best summarizes the main idea. Choice (B) is correct. The author never explains why Mona has trouble keeping her balance. So (A)
is incorrect. The passage does show that Sam helps Mona at school (C), and that Mona raised money for her local humane society
(D), but these are not main ideas. They are merely details the author uses to support a larger idea. Therefore they are incorrect.
2) A
In paragraph 2, the author tells us a service dog is a dog that has been trained to assist someone who has a physical problem.
Later in the paragraph, the author tells us that Mona's service dog "pulls her wheelchair and turns lights on and off." This lets us
know that a service dog does things that help someone who has a physical problem. Since Frank's dog turns on the lights when
Frank enters the room, we can understand that this is something that a dog would do to help someone who has a physical problem.
Therefore, it is most likely that Frank's dog is a service dog. This means choice (A) is correct. Raul's dog helps Raul by fetching the
newspaper while he gets dressed. Although this is helpful, there is no indication that Raul is disabled, or that he needs a dog to do
this work for him. This eliminates choice (B). Although it seems sweet that Mei's dog licks Meis face when she cries, this is not an
act of helping her do something that she cannot do on her own. This means (C) is incorrect. (D) is incorrect because Teddy's dog
does not do anything beyond what an ordinary dog would do.
3) C
In paragraph 2, the author tells us that "Sam lets Mona lean on him when she walks." This supports option (I).In paragraph 2, the
passage shows us that Sam performs everyday tasks for Mona, such as pulling off her socks and carrying her schoolbooks. This
supports option (II). In paragraph 3, the author writes, Monas classmates flock around Sam like geese. This has helped her make
friends. This supports option (III). Therefore (C) is correct.
4) D
The sentence in question uses the word like to compare Monas classmates to geese. Therefore the above sentence is a simile,
making (D) the correct choice. The above sentence does not reference to, or representation of, people, places, events, literary
work, myths, or works of art, either directly or by implication. Therefore, the above sentence is not an allusion. (A) is incorrect.
Saying that Monas classmates flock around Sam like geese gives animal traits to human beings, not the reverse. Therefore, the
above sentence does not use personification, and (B) is incorrect. Finally the phrase flock around Sam like geese uses words to
express their ordinary meaning. Therefore, the above sentence does not use irony. So (C) is incorrect.

Copyright Read Theory LLC, 2012. All rights reserved.

READTHEORY
Name________________
Date________________

Reading Comprehension 5

Level 6

Directions: Read the passage. Then answer the questions below.


To Whom it May Concern:
On March 5, 2010, I bought a Perfect Muffin Kit from your store at Midfield Mall. The cashier who
assisted me was George. George was very friendly and assured me that the Perfect Muffin Kit would live
up to the guarantee on the box: Perfect Muffins Every Time!
Unfortunately, this product did not live up to its claim. Although the box promised, as I stated
above, to provide Perfect Muffins Every Time, the muffins I made were far from perfect. I followed the
directions included in the package very carefully. First, I removed the bag of mix from the box. Then, I
poured it into a bowl. Next, I added the correct amount of water to the mix and stirred it. The directions
said that after stirring the mixture, I could, and I quote, add half a cup of raisins, nuts, berries, or another
favorite ingredient. My favorite ingredient happens to be hot sauce. I find that a dash of hot sauce makes
pizza, pasta, and soup taste very delicious. So, continuing to follow the directions, I added half a cup of
hot sauce to the mix and stirred it. Finally, I poured the mix into muffin tins and baked it in the oven at 350
degrees for exactly 20 minutes.
When the muffins finished baking, I was very excited to eat the Perfect Muffins as promised on
the box. You can imagine my disappointment when, upon tasting the muffins, I discovered that they were
not perfect. These muffins were, in fact, absolutely terrible. Not even my dog was interested in eating
these supposedly perfect muffins.
I would appreciate a full refund ($3.99) for this product as soon as possible. Enclosed are the
receipt, the empty box, and one of the un-perfect muffins so that you can experience it for yourself. Thank
you for your prompt attention to this matter.
Sincerely,
Michelle Bauer

Questions
1) This letter is most likely addressed to
A.
B.
C.
D.

the owner of the muffin mix company


a local storeowner
the clerk at a local muffin bakery
George, the cashier who sold Michelle the muffin mix

2) The tone of the author can best be described as


A.
B.
C.
D.

furious
disgusted
embarrassed
frustrated

3) As used in paragraph 1, which is the best synonym for guarantee?


A.
B.
C.
D.

lie
warning
promise
sentence
Copyright Read Theory LLC, 2012. All rights reserved.

4) Which of the following best describes the organization of paragraph 2?


A. The paragraph follows chronological order, in which events are presented in the sequence that
they occur.
B. The author explains a problem and then proposes a solution to that problem.
C. The author makes an argument and then provides evidence to support that argument.
D. The paragraph follows a cause-and-effect order, in which a series of causes are given, followed
by their effects.
5) It can be understood that the author is disappointed by the product because
A.
B.
C.
D.

it was worth less money than she paid for it


it did not fulfill the promise made on the box
the directions included with the product contained a mistake
the directions included with the product were too difficult to follow

6) It can be understood that the author put hot sauce in the muffins because she
A.
B.
C.
D.

wanted to get a full refund


did not read the directions carefully
was out of raisins, nuts, and berries
considered it one of her favorite ingredients

7) The authors main purpose in writing this letter is to


A.
B.
C.
D.

complain about how bad the muffins tasted


obtain a full refund for her money
prevent others from making the same mistake she did
persuade the company to change the wording on their box

8) Which of the following words best describes the author?


A.
B.
C.
D.

shrewd
senseless
unreasonable
careful

9) Instead of hot sauce, which of the following ingredients might the author have reasonably added to
enhance the flavor of the muffins?
I. raspberries
II. banana nuts
III. chocolate chips
A.
B.
C.
D.

I only
I and II only
II and III only
I, II, and III

10) According to the author, the muffins tasted "absolutely terrible." This is the result of
A.
B.
C.
D.

misinterpretation
negligence
forgetfulness
recklessness

Copyright Read Theory LLC, 2012. All rights reserved.

Answers and Explanations


1) B
In the first paragraph, the author writes that she bought a Perfect Muffin Kit from your store at Midfield Mall. This lets us know that
the author is most likely addressing the owner of the store from which she bought the muffin mix. Therefore (B) is correct. The
passage does not provide information to support choices (A), (C), and (D). Therefore they are incorrect.
2) D
In this passage, the author requests a refund for her product. In writing this request, she seems annoyed, or irritated, that the
muffins did not live up to their expectations, even though she took extra care to follow the directions included in the package "very
carefully." In paragraph 3, the author expresses excitement about eating the "Perfect Muffins" as promised on the box. However,
upon trying them, she notices that they taste terrible. Given that she took special care to ensure that she made them correctly, yet
still found them to taste terrible, we can understand that she feels disappointed or thwarted. The word frustrated accurately captures
this sentiment. Therefore (D) is correct. Although she is upset, it is wrong to think the author is furious. This is too strongly negative.
Therefore (A) is incorrect. In paragraph 3 the author says that the muffins tasted "absolutely terrible." Although the muffins may
have tasted disgusting, this is different than saying that the author was disgusted. If someone is disgusted, he or she is offended or
revolted. This is too strongly negative. What is more, this does not capture the idea that the author feels thwarted, as her careful
efforts were not properly rewarded. This means (B) is incorrect. (C) is incorrect because the author is very explicit, or clear and
forthright, in her explanation of why she deserves a refund. Since she holds no reservations in telling the letter's recipient her exact
thoughts, we can understand that her tone is not embarrassed.
3) C
guarantee (noun): a promise of quality; something that assures a specific outcome.
The question asks us to find the best synonym. Synonyms are words that have nearly the same meanings. In the first paragraph,
the author states that the Perfect Muffin Kit had a guarantee on the box: Perfect Muffins Every Time! Using the context in
paragraph 2, we can figure out the meaning of this word. In paragraph 2, the author writes: Unfortunately, this product did not live
up to this claim. Although the box promised, as I stated above, to provide Perfect Muffins Every Time, the muffins I produced were
far from perfect. The words claim and promise should help readers understand that a guarantee is a promise. This lets us
know that (C) is correct. The passage does not provide information to support choices (A), (B), and (D). Therefore they are
incorrect.
4) A
In paragraph 2, the author explains the steps she followed in making the muffin using the words First, Then, Next, and Finally.
Since these words all are used to describe the order of events in time, this lets us know that the author is using a chronological
order. Therefore (A) is correct. The passage does not provide information to support choices (B), (C), and (D). Therefore they are
incorrect.
5) B
According to the author, the Perfect Muffin Kit box guarantees, or promises, that the product will produce Perfect Muffins Every
Time! The authors muffins are not perfect, as they turn out to be absolutely terrible. Since this qualifies (in the author's opinion) a
violation of the product's guarantee, we can understand that (B) is correct. (A) is incorrect because although the author requests a
refund in her letter, this does not mean that she thinks the product was worth less money than she paid for it. Rather, she requests a
refund for the product. This means that she would like the full amount paid for the product to be returned to her. Her disappointment
does not result from anything involving the actual price of the product. It may seem like the directions that came with the product
contained a mistake. After all, the author did follow them perfectly as they were written. However, this is not actually the case. Using
the passage as a guide, it can be understood that directions were written with the understanding that their reader is able to
reasonably conclude that "one of your favorite ingredients" does not include something as extreme as hot sauce. Rather, as the
directions suggests, something like raisins, nuts, or berries would be more suitable. Although hot sauce may indeed by one of the
author's favorite ingredients, it is unreasonable to believe that it could be a tasty ingredient in muffins. In any case, one does not
have to be a chef to understand that hot sauce does not go well with muffins. Therefore (C) is incorrect. Although the author did fail
to follow the directions properly, she is unaware of this fact. Therefore, she could not be disappointed by this. This means (D) is
incorrect.
6) D
In paragraph 2, the author states that the directions for the muffin mix tell her to add half a cup of raisins, nuts, berries, or another
favorite ingredient. Because the authors favorite ingredient is hot sauce (and she wants to follow the instructions "very carefully"),
she puts half a cup of hot sauce into the mix. This lets us know that (D) is correct. The passage does not provide information to
support choices (A), (B), and (C). Therefore they are incorrect.
7) B
Paragraphs 1-3 in the author's letter are all intended to provide reasoning behind why the author should be eligible for a "full refund,"
which she asks for in the final paragraph. Given that the majority of the letter is reserved for this purpose, we can understand that it
is the main purpose. This lets us know that (B) is correct. Although the author complains about how bad the muffins tasted, probably
does not want others to make the same mistake as she did, and suggests that the wording on the box was misleading, her main
purpose for writing this letter is to get her money back. Therefore (A), (C), and (D) are incorrect.
8) C
The author lacks the common sense necessary to realize that hot sauce would not be a good ingredient in muffins. Given that the
vast majority of people know that these two foods do not mix well, it is unreasonable of her to request a "full refund" simply because

Copyright Read Theory LLC, 2012. All rights reserved.

she fails to understand this. Choice (C) is correct. Although the author may seem shrewd in her willingness to write a letter
requesting monetary compensation for her own mistake, this is not as defining an aspect of her character as her inability to reason
properly. Therefore (A) is incorrect. (B) is incorrect because senseless is too strongly negative. We know that the author has some
sense (albeit very little), otherwise she wouldn't have come to the conclusion that hot sauce and muffins don't mix well. While the
author follows the instructions on the box "very carefully," she misses the larger idea that some ingredients simply do not mix well
with others. The failure of her to see this larger idea is a more defining aspect of her character than her desire to follow the
instructions carefully, as it results in her writing of the letter. Therefore (D) is incorrect.
9) D
In paragraph 2, we learn that the directions on the box say to "add half a cup of raisins, nuts, berries, or another favorite ingredient."
Since raspberries are a kind of berry, it is reasonable to think they would enhance the flavor of the muffins. This supports option (I).
Since banana nuts are a kind of nut, it is reasonable to think they would enhance the flavor of the muffins. This supports option (II).
Since chocolate chips are a common favorite, and they are a reasonable ingredient to add to muffins, we can understand that they
would enhance their flavor. This supports option (III). Therefore (D) is correct.
10) A
In paragraph 2, we learn that the directions say to "add half a cup of raisins, nuts, berries, or another favorite ingredient." The author
reads these directions literally, meaning she reads them following the strict meaning of the words. She does not think for herself
about what they may actually mean. So, she does not reason that, although it is one of her favorite ingredients, hot sauce would be
a bad ingredient to add to muffins. Since the author fails to interpret these directions properly, we can understand that they muffins
tasted terrible as a result of a misinterpretation. Choice (A) is correct. The author does not make any careless errors. Actually, her
error comes from following the directions too carefully. Since negligence is carelessness, we can understand that (B) is incorrect.
(C) is incorrect because the author does not forget anything while making the muffins. (D) is incorrect because the author is never
reckless, or wild and irresponsible.

Copyright Read Theory LLC, 2012. All rights reserved.

ReadTheory.Org 2010
EnglishForEveryone.Org 2008

Name________________
Date________________

A Day Like No Other


Reading Comprehension Short Stories

Directions: Read the story. Then answer the questions below.

Frank Wilcox has been Chief of Police in Lansett County for 25 years. He took
the job when he had just turned 30.
He has seen murders. He has seen robberies. He has seen cats stuck in trees.
He has found missing children.
But today would be like no other day on the job.
It is 11:00 at night. Chief Wilcox begins putting together his things. He is tired. He
wants to go home.
Chief Wilcox, calls an officer walking quickly into his office. It is Officer
Simpson. He looks nervous. He looks like he would like to be anywhere else but
there.
What is it, Simpson? asks the Chief.
Holmans Grocery was just held up at gunpoint, Simpson says. His voice
is shaky. He coughs to clear his throat.
Was anyone hurt? asks Chief.
Lansett is a very small county. The Chief knows just about everyone who lives
there. If anyone was hurt, there is a good chance he knows the person. Maybe
that's why Officer Simpson looks nervous.
No one was hurt, says Simpson. But we caught the suspect.
Ah, well, Simpson. You guys can take care of that. Im Chief Wilcox stops in
mid-sentence.
He understands what is wrong. From behind Officer Simpson, the Chief sees his
youngest daughter, Devon. She is in handcuffs.
Chief Wilcox gets a lump in his throat. He sits in his chair, stunned. How could it
be? he thinks.
Devon, will you please tell me what is going on? the Chief demands.
Devon does not look at him. The Chief can feel anger growing inside of him.
He refuses to let that anger show.
Take her back for questioning, the Chief says to Officer Simpson in a calm
voice.
Devon, whatever you do, tell the truth, the Chief says. Im your dad. I love you.
We will figure this out.

Questions:
1) What is the main problem in this
story?
A. Devon won't look at her dad.
B. Officer Simpson is nervous.
C. The Chief's daughter has been
arrested.
D. The Chief is tired and wants to
go home.

3) What can the reader tell about Chief


Wilcox?
I. He wants the truth.
II. He has seen a lot in his job.
III. He is 30 years old.
A.
B.
C.
D.

I only
I and II
II and II
I, II, and III

5) Why is Officer Simpson nervous?


A. He is scared of Chief Wilcox.
B. He has just been held up at
gunpoint.
C. He has just arrested the Chief's
daughter.
D. He has just found an important
piece of evidence in a new case.

7) When does the Chief discover what


is wrong with Officer Simpson?
A.
B.
C.
D.

when he sees the gun


when he stands up
when Officer Simpson tells him
when he sees his daughter

2) For how long has Wilcox been Chief


of police?
A.
B.
C.
D.

15 years
25 years
30 years
35 years

4) Why does the Chief want to go


home?
A.
B.
C.
D.

He is hungry.
He is tired.
He does not want to see Devon.
He is worried about his family.

6) If someones voice is shaky, it is


I. loud
II. unsure
III. unsteady
A.
B.
C.
D.

I only
I and II
II and III
I, II, and III

8) "He refuses to let that anger show."


Which sentence below means the
same thing?
A. He is forced to let that anger
show.
B. He is about to let that anger
show.
C. He wants to let that anger show.
D. He decides not to let that anger
show.

Questions (continued):
9) When Chief Wilcox asks Devon what 10) How do the Chief's feelings
is going on, Devon
change during the story?
A.
B.
C.
D.

does not look at him


starts crying
takes off the handcuffs
runs away

11) How old is Chief Wilcox?


A.
B.
C.
D.

30
45
55
60

A. from tired, to surprised, to angry,


to loving
B. from tired, to loving, to
surprised, to angry
C. from tired, to loving, to angry, to
surprised
D. from tired, to angry, to surprised,
to loving

12) What crime is Devon suspected


of?
A.
B.
C.
D.

being missing
holding up a grocery.
murder
having a gun

How do you think Chief should handle this case? Explain.


________________________________________________________________________
________________________________________________________________________
________________________________________________________________________
________________________________________________________________________
________________________________________________________________________
________________________________________________________________________
________________________________________________________________________
________________________________________________________________________
________________________________________________________________________
________________________________________________________________________

Answers and Explanations


1) C
The story first tells us that Frank has been the police chief for 25 years. In that time,
the Chief has seen many things, but today is different because today Chief Wilcox
own daughter is the suspect. The Chiefs daughters arrest is the problem at the
heart of the story. Therefore (C) is correct.
Devon wont look at her dad because she has been arrested for robbing a grocery
store, which means the main problem is the arrest for robbery. Therefore (A) is
incorrect. Officer Simpson is nervous because he has just arrested the Chiefs
daughter. The main problem, the arrest, has caused his nervousness. Therefore (B)
is incorrect. Although the story does tell us that Chief Wilcox is tired, that is a less
serious problem than his daughters arrest for robbery. Therefore (D) is incorrect.
2) B
At the beginning, the story says, Frank Wilcox has been Chief of Police in Lansett
County for 25 years. Therefore (B) is correct. The story does not provide
information to support answer choices (A), (C), and (D). Therefore they are
incorrect.
3) B
At the end of the story, Wilcox tells his daughter, Whatever you do, tell the truth.
This indicates that the truth is important to Wilcox. This supports option (I). At the
beginning of the story, the reader learns that over his 25 years as chief of police,
Wilcox has seen murders, robberies, cats stuck in trees, and has found missing
children. We can infer that Wilcox has seen a lot in his job. This supports option
(II). At the beginning of the story, the reader learns that Wilcox was 30 years old
when Wilcox became the police chief and Wilcox has been the police chief for 25
years. Therefore, Wilcox is 55 years old. This eliminates option (III). Therefore (B) is
correct.
4) B
At the beginning, the story says Wilcox is tired. He wants to go home. We can
understand from this that Wilcox wants to go home because he is tired. Therefore
(B) is correct. The passage does not provide information to support answer choices
(A), (C), and (D). Therefore they are incorrect.
5) C
In the middle of the story, Officer Simpson walks in and looks nervous. When
Wilcox sees his youngest daughter in handcuffs, Wilcox understands what is
wrong. We can understand from this that Simpson was nervous because he had
just arrested Wilcox daughter. Therefore (C) is correct.
According to the story, Holmans Grocery was held up at gunpoint. Simpson was
not held up at gunpoint. Therefore (B) is incorrect. The passage does not provide
information to support answer choices (A) and (D). Therefore they are incorrect.

6) C
shaky (adjective): trembling or quivering; uncertain or questionable.
Simpson tells the Chief, Holmans Grocery was just held up at gunpoint. Simpsons
voice is shaky. He coughs to clear his throat. From this, we can understand that
Simpson is uncomfortable and that his voice is not clear. A loud voice is clear rather
than unclear. This eliminates option (I). When Officer Simpson first appears at the
station, he looks nervous. A nervous voice is unsure. This supports option (II). An
uncomfortable, unclear voice is unsteady .This supports (III). Therefore (C) is
correct.
7) D
In the middle of the story, we learn that Officer Simpson seems nervous and shaky.
Wilcox wonders if Simpson is nervous and shaky because someone that Wilcox
knew got hurt. A little later, the story says Wilcox understands what is wrong. From
behind Officer Simpson, the Chief sees his youngest daughter, Devon. She is in
handcuffs. From this, we can understand that Wilcox figured out why Officer
Simpson was nervous when Wilcox saw his daughter in handcuffs. Therefore (D) is
correct.
The passage does not provide information to support answer choices (A), (B), and
(C). Therefore they are incorrect.
8) D
refuses (verb): declines to do something.
Near the end, the story says, The Chief can feel anger growing inside of him.
He refuses to let that anger show. We can understand from this that even though
the Chief feels angry, he chooses not to show this anger. Therefore (D) is correct.
Something that is forced is made necessary. The Chief does not feel it is necessary
to show his anger. Therefore (A) is incorrect. If you are about to do something, you
are going to do it presently. That is different from choosing to do something or not.
Therefore (B) is incorrect. To want is to desire. That is different from deciding not do
something. Therefore (C) is incorrect.
9) A
When Chief Wilcox asks Devon what is going on, Devon does not look at him.
Therefore (A) is correct. Although the story tells us that Devon is wearing handcuffs,
the story does not say that Devon takes them off. Therefore (C) is incorrect. The
story does not provide information to support answer choices (B) and (D). Therefore
they are incorrect.
10) A
Early in the story, we learn that Chief Wilcox is tired. Then when Wilcox first
realizes that the suspect is Devon, Wilcox sits in his chair, stunned. From this, we
can understand that Wilcox is surprised. Then, when Devon does not answer Wilcox
question, Wilcox can feel the anger growing inside of him. We can understand from
this that Wilcox is angry. Finally, Wilcox tells Devon, Im your dad. I love you. We

will figure this out. We can understand from this that Wilcox feels loving toward his
daughter. Chief Wilcox feelings in the story went from tired, to surprised, to angry, to
loving. Therefore (A) is correct.
Answer choices (B), (C) and (D) list the feelings in improper order. Therefore they
are incorrect.
11) C
At the beginning of the story, the reader learns that Wilcox was 30 years old when
Wilcox became the police chief and Wilcox has been the police chief for 25 years. 30
plus 25 is 55, so Wilcox is 55 years old. Therefore (C) is correct. The story does not
contain information to support answer choices (A), (B), and (D). Therefore they are
incorrect.
12) B
In the middle of the story, Officer Simpson says, Holmans Grocery was just held up
at gunpoint. Then Simpson says, We caught the suspect. Finally, the Chief sees
his youngest daughter, Devon. She is in handcuffs. We can understand from this
that Devon was caught holding up the grocery store at gunpoint. Therefore (B) is
correct.
Although Chief Wilcox has found missing children, the story does not say that
Devon was missing. Therefore (A) is incorrect. While Officer Simpson is describing
the crime, he states that no one was hurt. Therefore (C) is incorrect. Devons crime
was not merely having a gun, but holding up the grocery store at gunpoint. Therefore
(D) is incorrect.

ReadTheory.Org 2010

Name________________
Date________________

EnglishForEveryone.Org 2008

A Mystery
Reading Comprehension Short Stories

Directions: Read the story. Then answer the questions below.

Something is very wrong, says the detective.


I know! says Ms. Gervis. It is wrong that someone has stolen from me!
The detective looks around Ms. Gervis apartment. That is not what I am talking
about, maam. What is wrong is that I do not understand how the robber got in
and out.
Ms. Gervis and the detective stand in silence. Ms. Gervis eyes are full of tears.
Her hands are shaking.
The robber did not come through the window, says the detective. These
windows have not been opened or shut in months.
The detective looks at the fireplace. The robber did not squeeze down here.
The detective walks to the front door. He examines the latch. And since there
are no marks or scratches, the robber definitely did not try to break the lock.
I have no idea how he did it, says a bothered Ms. Gervis. It is a big mystery.
And you say the robber stole nothing else? asks the detective. No money, no
jewelry, no crystal?
Thats right, detective. He took only what was important to me, Ms. Gervis says
with a sigh. There is only one thing I can do now.
And what is that? the detective asks with surprise.
I will stop baking cakes, Ms. Gervis says. They are mine to give away. They
are not for someone to steal.
You cant do that! says the detective with alarm. Who will bake those delicious
cakes?
I am sorry. I do not know, says Ms. Gervis.
I must solve this case immediately! says the detective.

Questions:
1) Where does this story take place?
A.
B.
C.
D.

in a bakery
at the police station
in Ms. Gervis' house
in Ms. Gervis' apartment

3) What makes the detective sure that


the robber did not come through the
windows?
A. The windows are locked.
B. The windows face the police
station.
C. The windows have not been
used in months.
D. The windows are too small for a
person to fit through.

5) What else could the detective have


asked Ms. Gervis in order to solve
the mystery?
I. Which types of cakes does Ms.
Gervis know how to bake?
II. Does someone else have a key
to the apartment?
III. Does Ms. Gervis ever leave the
door unlocked?
A.
B.
C.
D.

I only
I and II
II and III
I, II, and III

2) Near the beginning of the story,


Ms. Gervis eyes are full of tears.
Her hands are shaking. How does
Ms. Gervis probably feel?
A.
B.
C.
D.

She is upset.
She is tired.
She is hungry.
She is confused.

4) "And the robber definitely did not


use the front door." Which is the
best way to rewrite this sentence?
A. "And the robber may not have
used the front door."
B. "And the robber probably did not
use the front door."
C. "And the robber was not able to
use the front door."
D. "And the robber certainly did not
use the front door."

6) What does Ms. Gervis do with her


cakes?
A.
B.
C.
D.

She eats them.


She sells them.
She hides them.
She gives them away.

Questions (continued):
7) What does the detective seem to
think will happen if he solves the
mystery?
A. Ms. Gervis will start baking
cakes again
B. Ms. Gervis will bake him extra
cakes
C. Ms. Gervis will give him her
secret recipe
D. Ms. Gervis will give him money
and jewels

9) What else was stolen from the


apartment?
A.
B.
C.
D.

crystal
jewelry
money
nothing

8) What is a mystery?
A. something that is wrong
B. something that happens at night
C. something a robber leaves
behind
D. something that cannot be
explained

10) If something is said with alarm,


how is it said?
A.
B.
C.
D.

with fear and panic


with bells and whistles
with smiles and laughter
with sadness and tears

Do you like mysteries? What is your favorite kind of story? Explain.


________________________________________________________________________
________________________________________________________________________
________________________________________________________________________
________________________________________________________________________
________________________________________________________________________
________________________________________________________________________
________________________________________________________________________
________________________________________________________________________
________________________________________________________________________
________________________________________________________________________

Answers and Explanations


1) D
Near the beginning of the story, the detective looks around Ms. Gervis
apartment. We can understand from this that the story takes place in Ms.
Gervis apartment. Therefore (D) is correct.
Ms. Gervis cake was stolen, but it was stolen from her apartment, not a
bakery. Therefore (A) is incorrect. Detectives work at the police station, but
the detective was investigating at Ms. Gervis apartment. Therefore (B) is
incorrect. Ms. Gervis has an apartment, not a house. Therefore (C) is
incorrect.
2) A
The detective is at Ms. Gervis apartment because Ms. Gervis has been
robbed. In fact, the robber took only what was important to Ms. Gervis.
Since Ms. Gervis apartment has been robbed and the robber took something
important to her, we can understand that Ms. Gervis is upset. Therefore (A) is
correct.
The story does not provide information to support answer choices (B) and
(C). Therefore they are incorrect. Ms. Gervis does not know how someone
got into her apartment, but she is shaking and crying because she has been
robbed, not because she is confused. Therefore (D) is incorrect.
3) C
In the middle of the story, the detective says, The robber did not come
through the windows. Then the detective says, These windows have not
been opened or shut in months. We can understand from these statements
that the detective believes that the robber didnt come through the windows
because the windows had not been used in months. Therefore (C) is correct.
The story does not provide information to support answer choices (A), (B),
and (D). Therefore they are incorrect.
4) D
definitely (adjective): certainly; without doubt.
In the middle of the story, the detective examines the latch on the door. The
detective says, Since there are no marks or scratches, the robber definitely
did not try to break the lock. The reader can understand from this that since
there is no evidence that someone tried to break the lock, someone certainly
did not try to break the lock. Certainly is another way to say definitely.
Therefore (D) is correct.
Definitely means certainly, but something that may not happen is uncertain.
Therefore (A) is incorrect. Since probably means that something could
happen but might not, it is not certain. Therefore (B) is incorrect. When

someone is not able to do something, they cannot do that thing. But when
someone definitely can do something, they are certain to do it. Therefore (C)
is incorrect.
5) C
The detective determined that the robber did not come in through the
windows or the fireplace. Also, the robber did not try to break the lock on the
door. Therefore, the mystery here is how someone got into the apartment to
steal the cake. Knowing what kinds of cakes Ms. Gervis can bake will not help
the detective determine how someone got in the apartment. This eliminates
option (I). Knowing whether someone else had a key could help the detective
find possible suspects, because someone with a key could get in the
apartment without breaking the lock. This supports option (II). If Ms. Gervis
left the door unlocked, someone could also have come in without breaking the
lock. This supports option (III). Therefore (C) is correct.
6) D
Near the end of the story, Ms. Gervis says that she will stop baking cakes:
They are mine to give away. They are not for someone to steal. We can
understand from this that Ms. Gervis gives her cakes away. Therefore (D) is
correct. The story does not provide information to support answer choices
(A), (B), and (C). Therefore they are incorrect.
7) A
Near the end of the story, Ms. Gervis says that she will stop baking cakes.
This makes the detective alarmed. The detective wonders, Who will bake
those delicious cakes? Then, the detective decides to solve this case
immediately! We can understand from this that the detective wants to solve
the case immediately because he thinks that Ms. Gervis will bake cakes again
if the case is solved. Therefore (A) is correct.
Ms. Gervis has simply said she will no longer bake cakes because they are
not made to be stolen. She does not offer extra cakes for solving the mystery.
Therefore (B) is incorrect. The story does not provide information to support
answer choices (C) and (D). Therefore they are incorrect.
8) D
mystery (noun): an unexplained event; something that is not fully understood.
In this story, the detective is trying to figure out how a robber got into an
apartment. The detective notices that the robber did not come in through a
window, through the fireplace, or break the lock to the door. Ms. Gervis says
that how the robber got in is a big mystery. Since the detective has looked at
all the normal ways that someone may have gotten into the apartment and
ruled them out, a mystery must be something that is hard to figure out or
explain. Therefore (D) is correct.

Something that is wrong is incorrect. A mystery is hard to figure out, but it is


not incorrect. Therefore (A) is incorrect. Robbers often break in at night, but
this story does not give that information. More importantly, a mystery is
something hard to explain. This could happen at any time. Therefore (B) is
incorrect. The story does not provide information about something the robber
left behind. Again, a mystery is something hard to explain. Therefore (C) is
incorrect.
9) D
In the middle of the story, the detective asks Ms. Gervis if the robber stole
anything else: No money, no jewelry, no crystal? Ms. Gervis says, Thats
right, detective. Ms. Gervis goes on to say that the robber has taken the only
thing that is important to her, and that now she will stop baking cakes. We can
understand from this that the only thing that the robber stole was cake.
Therefore (D) is correct.
Ms. Gervis agreed that the robber did not take crystal (A), jewelry (B), or
money (C). Therefore these answer choices are incorrect.
10) A
alarm (noun): fear or terror aroused by awareness of danger.
When Ms. Gervis says that she will no longer bake cakes, the detective
reacts with alarm. The detective says he will solve the case immediately!
The reader can understand from this that the detective is very afraid that Ms.
Gervis will not bake her delicious cakes if the he does not solve the case.
Alarm is fear or terror. Therefore (A) is correct.
Bells and whistles is a phrase that means with all the little extras, such as a
new car that comes with all the special features. Therefore (B) is incorrect.
The detective is afraid that there will be no more delicious cakes. This does
not make the detective happy, so alarm is not smiles and laughter. Therefore
(C) is incorrect. The detective is worried that there will be no more cake, but
he is not sad about ithe is ready to solve the case. Therefore (D) is
incorrect.

ReadTheory.Org 2010
EnglishForEveryone.Org 2008

Name________________
Date________________

Ana Finds an Apartment


Reading Comprehension Short Stories

Directions: Read the story. Then answer the questions below.

Ana, her husband Mario, and their four-year-old son, Antonio, just moved to
North Carolina.
They need a temporary place to call home until they get settled into their new
surroundings.
Right now, they are staying in a hotel not far from Marios job.
Ana begins a search for an apartment for the family to live in.
First, Ana picks up an Apartment Book at the local newspaper stand. The
Apartment Book contains listings of all the major apartment complexes in her
area.
Ana starts by looking at the prices for apartments in the Apartment Book. Then,
she reads about the amenities that each apartment complex offers.
For example, some apartments have a clubhouse. Some have a gym, which is
also called a fitness center. Some have a pool. Some have all of these!
Ana notices that the more amenities an apartment complex has, the more it costs
each month. She wants the familys new apartment to be nice, but she does not
want to spend too much money on it. Ana and Mario are trying to save money to
buy a house.
After considering prices, amenities, and locations, Ana finds several apartment
complexes that she thinks the family will like.
Ana calls the apartment complexes. She sets up appointments with the
apartment managers to see the apartments she thinks are interesting.
Ana makes five phone calls in total.
When Mario gets back to the hotel from work, Ana shows him the list of
apartments. These look good, he says.
The next day, while Mario is at work and Antonio is at daycare, Ana visits the
apartment complexes.
She likes the fifth one the best. It is in a good school district. It has a pool, but no
fitness center or clubhouse. It is near Marios job. Ana hopes to find a job nearby
as well.
When Ana gets back to the hotel, she discusses all that she has seen with
Mario. They decide to rent the last apartment Ana saw.

The next day, Ana calls the manager of the apartment complex with the news.
The manager asks Ana and Mario to sign a lease and pay a security deposit.
If the family damages the apartment in any way while they are living there, the
security deposit will help to pay for the cost of repairs.
Ana and Mario sign a lease and pay the money. The manager asks them when
they plan to move in and Ana looks up at Mario. He looks back at her for a
moment and then tells the manager, In a few days.
Finally, they have a place to stay.

Questions:
1) What kind of book does Ana get?
A.
B.
C.
D.

a City Guide Book


a Map Book
a House Book
an Apartment Book

2) What did Ana consider while looking


through the Apartment Book?
I. school districts
II. amenities
III. locations
A.
B.
C.
D.

3) Where is Ana's family staying while


they look for a place to call home?
A. in an apartment close to Marios
work
B. at an apartment next to Antonios
school
C. in Anas mothers house
D. in a hotel near Marios work

5) Why doesn't Ana want to spend too


much money on an apartment?
A.
B.
C.
D.

Her husband will get upset.


She does not have the money.
She cannot find one she likes.
She wants to save money for a
house.

I only
I and II
II and III
I, II, and III

4) What does Ana realize as she looks


through the Apartment Book?
A. that the nicest apartments are far
away
B. that all the apartments are small
and cramped
C. that it is easy to find a nice
apartment in a good school district
D. that the number of amenities is
related to cost

6) As described in the beginning of the


story, which of the following is not
mentioned as an amenity?
A.
B.
C.
D.

carpet
clubhouse
fitness center
pool

Questions (continued):
7) What makes Ana like the fifth
apartment best?
I. It has many amenities.
II. It is near Marios job.
III. It is in a good school district.
A.
B.
C.
D.

I only
I and II
II and III
I, II, and III

8) Where is Antonio while Ana looks at


apartments?
A.
B.
C.
D.

at daycare
at the hotel
at school
at work

9) As described in the middle of the story, what does it mean to consider something?
A.
B.
C.
D.

to rent it
to think about it
to make it happen
to read a book about it

10) "When Ana gets back to the hotel, she discusses all that she has seen with Mario."
Which of the following is the best way to rewrite the above sentence while keeping
its meaning the same?
A.
B.
C.
D.

When Ana gets back to the hotel, she reads about all she has seen with Mario.
When Ana returns to the hotel, she writes about all she has seen with Mario.
When Ana returns to the hotel, she talks about all she has seen with Mario.
When Ana leaves the hotel, she talks about all she has seen with Mario.

11) When will the family move into their


new apartment?
A.
B.
C.
D.

the next day


at the first of the month
in three or four days
in just over a week

12) As used at the end of the story, which


is the best description of a security
deposit?
A. money given to landlord to pay for
amenities
B. money given to landlord to pay for
utilities and telephone
C. money given to landlord to prove
the tenant can pay rent on time
D. money given to the landlord to pay
for any damage to the apartment

Answers and Explanations


1) D
Paragraph 5 tells us that Ana picks up an Apartment Book at the local
newspaper stand. Therefore (D) is the correct answer. The story does not
contain information to support answer choices (A), (B), and (C). Therefore
they are incorrect.
2) C
In paragraph 9, we learn that Ana chooses some apartments to visit after
considering prices, amenities, and locations. Although school districts are
related to locations, the two are not the same thing. Later, we learn that her
favorite apartment complex is in a good school district. But the question
asks what she considers while looking through the Apartment Book, and the
story does not contain information that she considers school districts at that
time. This eliminates option (I). In paragraph 9, we learn she does consider
amenities. This supports option (II). In paragraph 9, we also learn that she
considers locations. This supports option (III). Therefore (C) is correct.
3) D
In paragraph 3 we learn that they are staying in a hotel not far from Marios
job. Therefore (D) is correct. The story does not contain information to
support answer choices (A), (B), and (C). Therefore they are incorrect.
4) D
In paragraph 8 we learn that Ana notices that the more amenities an
apartment complex has, the more it costs each month. Since the price of the
apartment goes up in relation to the number of amenities offered, the number
of amenities is related to cost. Therefore (D) is correct.
The story does not contain information to support answer choices (A) and (B).
Therefore they are incorrect. Although it may be easy to find a nice apartment
in a good school district, the story does not provide enough information to
determine whether it was easy or hard for Ana to do. Also, the story does not
tell us this is a realization Ana has while reading the Apartment Book.
Therefore (C) is incorrect.
5) D
In paragraph 8 we learn that Ana wants the familys new apartment to be
nice, but she does not want to spend too much money on it. Ana and Mario
are trying to save money to buy a house. Therefore (D) is correct. The story
does not contain information to support answer choices (A), (B), and (C).
Therefore they are incorrect.
6) A
In paragraphs 6 and 7 it says that Ana reads about the amenities that each
apartment complex offers. For example, some apartments have a clubhouse.
Some have a gym, which is also called a fitness center. Some have a pool.
Clubhouse is mentioned. This eliminates (B). Fitness center is mentioned.
This eliminates (C). Pool is mentioned. This eliminates (D). Carpet is the only
thing that is not mentioned. Therefore (A) is correct.

7) C
In paragraph 14, we learn that Ana likes the fifth one best. The story tells us
it has a pool, but no fitness center or clubhouse. One amenity is not many,
so this eliminates option (I). We also learn it is near Marios job. This
supports option (II). In the next sentence we learn it is in a good school
district. This supports option (III). Therefore (C) is correct.
8) A
In paragraph 13, we learn that while Mario is at work and Antonio is at
daycare, Ana visits the apartment complexes. Therefore (A) is correct. The
story does not contain information to support answer choices (B), (C), and
(D). Therefore they are incorrect.
9) B
In paragraph 9, we learn that Ana chooses some apartments to visit after
considering prices, amenities, and locations. These are the things that Ana
thinks about as being important to finding an apartment her family will like. To
consider means to think about. Therefore (B) is correct.
Ana eventually rents an apartment, but she does not rent prices, amenities,
and locations. Therefore (A) is incorrect. Ana does not make prices,
amenities, and locations happen, because these facts are set. She is merely
thinking about them. Therefore (C) is incorrect. Although Ana does read the
Apartment Book, she does not read a book to find out what will work for her
family. She thinks about these things instead. Therefore (D) is incorrect.
10) C
Returns means the same thing as gets back to, and talks about means
the same things as discusses. Answer choice (C) uses these words to keep
the meaning of the original sentence. Therefore (C) is correct. Reads is not
the same thing as discusses. Therefore (A) is incorrect. Writes is not the
same thing as discusses. Therefore (B) is incorrect. Leaves is not the
same thing as gets back to. Therefore (C) is incorrect.
11) C
In paragraph 19, the story tells us the manager asks them when they plan to
move in and Ana looks up at Mario. He looks back at her for a moment and
then tells the manager, In a few days. A few means a small number. Three
or four days is a small number of days. Therefore (C) is correct. The story
does not contain information to support answer choices (A), (B), and (D).
Therefore they are incorrect.
12) D
In paragraphs 17 and 18, we learn that the manager asks Ana and Mario to
sign a lease and pay a security deposit. If the family damages the apartment
in any way while they are living there, the security deposit will help to pay for
the cost of repairs. We can infer from this that the security deposit will be
used to pay for any damage to the apartment. Therefore (D) is correct. The
story does not contain information to support answer choices (A), (B), and
(C). Therefore they are incorrect.

ReadTheory.Org 2010

Name________________
Date________________

EnglishForEveryone.Org 2008

Just One Touch


Reading Comprehension Short Stories

Directions: Read the story. Then answer the questions below.

I am sitting in a chair next to Mommas bed. I am watching her get ready for a
party.
She opens the beautiful jewelry box on her nightstand.
It is the size of a shoebox. It is wooden. It has colorful stones on top. They are
red, green, yellow, and blue. To me, the box looks magical. It looks like it has
special powers.
Now, you know you must never touch this box, right? Momma says. I feel like
she knows exactly what I am thinking.
I just want to touch it. I just want to open it.
I just want to try on all the jewelry inside and dance around the room!
Yes, Momma, I say. I know.
What do I know?
I know Momma has always told me not to touch the box. She has said it since I
was a little girl.
You are not old enough to wear my jewelry, Momma says.
I am almost eleven years old! What is the big deal?
I know that when Momma puts on the rings and bracelets from the box, she looks
different. She seems to glow.
There is one necklace with a yellow stone like a tigers eye. When Momma puts
this on, she seems to float instead of walk. Her feet do not seem to touch the
ground. She moves lightly and gracefully. She moves without effort.
Momma kisses me goodnight. She leaves for the party. I run to the window to
wave to her, but she is already gone.
Tonight I am very curious.
Just one touch.
I sit on the edge of Mommas bed. I place my hand on the nightstand. I pause. I
think.

My hand moves up and rests on the jewelry box. The box quickly flips open by
itself! Jewels fly into the air. They dance around my head. I feel strange. I fall
down to the floor.
I wake up in a place I have never seen before.

Questions:
1) Where does most of this story take
place?
A.
B.
C.
D.

at a party
in a pretend place
outside a window
in Momma's bedroom

3) At the beginning of the story, where


is the girl sitting?
A.
B.
C.
D.

in a chair
on the bed
on the floor
next to the window

5) What does Momma tell her


daughter about the box?
A.
B.
C.
D.

that it is magical
that she must never touch it
that the jewels are expensive
that she can touch it when she is
older

7) What does the jewelry box look


like?
I. It is brown.
II. It is wooden.
III. It is the size of a shoebox.
A.
B.
C.
D.

I only
I and II
II and III
I, II, and III

2) Who is telling the story?


A.
B.
C.
D.

Momma
a young girl
Momma's friend
a pretend person

4) The jewelry box looks magical.


What does this mean?
A. It looks simple.
B. It looks expensive.
C. It looks like it has special
powers.
D. It looks like it was made a long
time ago.

6) Where is Momma going?


A.
B.
C.
D.

to a play
to a party
to a concert
to a magic show

8) Which word best describes how the


girl feels about the jewelry box?
A.
B.
C.
D.

angry
curious
in love
scared

Questions (continued):
9) Momma seems to glow when she
puts on
A.
B.
C.
D.

makeup
a necklace
the rings and bracelets
nice shoes and a fancy dress

11) How old is the girl?


A.
B.
C.
D.

11
12
10
15

13) What does it mean if something is


a big deal?
A.
B.
C.
D.

It is a heavy thing.
It is very important.
It cannot be forgotten.
It can make a lot of money.

10) What does the girl seem to be


thinking about when she sits on
the edge of the bed?
A.
B.
C.
D.

going to a party
how to open the box
dancing around the room
if she should touch the box

12) What does the reader learn about


the box?
A.
B.
C.
D.

It is evil.
It can fly.
It is very heavy.
It has special powers.

14) At the end of the story, what does


the girl mean when she says that
Momma seems to float?
A. that Momma is swimming
B. that Momma looks like a boat
C. that it is difficult for Momma to
move
D. that Momma moves without
much effort

15) How might the end of the story make the reader feel?
I. unsure about what has happened
II. curious about the power of the jewelry box
III. wondering what will happen next
A.
B.
C.
D.

I only
I and II
II and III
I, II, and III

Answers and Explanations


1) D
At the beginning of the story, the speaker says, I am sitting in a chair next to
Mommas bed. From this we can understand that she is in Mommas
bedroom. She does not go anywhere else in the story until the very end,
when she says, I wake up in a place I have never seen before. We can
conclude that most of the story takes place in Mommas bedroom. This
means (D) is correct.
Momma goes to a party, but the story does not take place at the party, so (A)
is incorrect. At the end, the speaker says, I wake up in a place I have never
seen before. This seems like it could be a pretend place. But most of the
story takes place in Mommas bedroom. Therefore (B) is incorrect. Near the
end of the story, the speaker says that she runs to a window to watch her
mother leave, but the story does not take place outside a window. Therefore
(C) is incorrect.
2) B
In the middle of the story, the speaker says Momma tells her not to touch the
box, and that she has said it since I was a little girl. Then she says that she
is almost eleven years old. From these facts we can understand that the
speaker is a young girl who is almost eleven years old. This means (B) is
correct.
The story does not contain information to support answer choices (A), (C),
and (D). Therefore they are incorrect.
3) A
At the beginning, the speaker says, I am sitting in a chair next to Mommas
bed. From this we know that the speaker is in a chair. Therefore (A) is
correct. The story does not contain information to support answer choices (B),
(C), and (D). Therefore they are incorrect.
4) C
magical (adjective): possessing or using supernatural powers.
Near the beginning, the speaker describe Mommas jewelry box. She says,
To me, the box looks magical. It looks like it has special powers. From this
we can understand that magical means having special powers. Therefore (C)
is correct.
The speaker describes the box as decorated with different colored stones. It
does not sound simple, so (A) is incorrect. The story does not say whether or
not the box was expensive. This makes (B) incorrect. The story does not say
whether or not the box was made a long time ago. This makes (D) incorrect.
5) B
Near the beginning of the story, Momma says to the speaker, Now, you know
you must never touch this box, right? Therefore (B) is correct. The speaker

says the box looks magical, but Momma does not tell her that, so (A) is
incorrect. Momma does not say the jewels are expensive. This means (C) is
incorrect. Momma does say, You are not old enough to wear my jewelry, but
she does not say that the speaker will be able to touch the box when she is
older. This means (D) is incorrect.
6) B
At the beginning the speaker says, I am watching her get ready for a party.
She is talking about Momma. Therefore (B) is correct. The story does not
contain information to support answer choices (A), (C), and (D). Therefore
they are incorrect.
7) C
Near the beginning, the speaker describes the jewelry box. She does not say
it is brown. She does say it is wooden, but that does not necessarily mean it
is brown. It could be painted or stained. This eliminates option (I). The
speaker does say that the box is wooden. This supports option (II). The
speaker also says that the box is the size of a shoebox. This supports
option (III). Therefore (C) is correct.
8) B
Near the beginning the speaker says, I just want to touch it. I just want to
open it. Near the end, she says, Tonight I am very curious. She wants to
touch the jewelry box and to open it so she can look inside. She feels curious
about the box. Therefore (B) is correct.
Angry means mad. There is no point in the story when the speaker seems
mad at the box, so (A) is incorrect. In love means deeply or passionately
enamored with. There is no information in the story that tells us the speaker is
in love with the jewelry box. Instead, she is curious about it. This makes (C)
incorrect. Scared means afraid. The speaker is not afraid of the jewelry box,
so (D) is incorrect.
9) C
Near the end of the story, the speaker says, I know that when Momma puts
on the rings and bracelets from the box, she looks different. She seems to
glow. From this we can understand that Momma seems to glow when she
puts on the rings and bracelets. Therefore (C) is correct.
The story does not say that Momma puts on makeup, so (A) is incorrect. The
story says that Momma seems to float when she puts on a necklace, not
that she seems to glow. This makes (B) incorrect. The story does not say that
Momma puts on nice shoes and a fancy dress. This means (D) is incorrect.
10) D
Near the end of the story, the speaker says, Tonight I am very curious. Just
one touch. I sit on the edge of Mommas bed. I place my hand on the
nightstand. I pause. I think. From this we can understand that the speaker is
curious about touching the box. She puts her hand on the nightstand, where
the box is. She pauses to think, and then her hand moves up and rests on

the jewelry box. From this we can understand that she is thinking about if she
should touch the box or not. Therefore (D) is correct.
Momma is going to a party, but the speaker is not thinking about going to a
party, so (A) is incorrect. The speaker is not thinking about how to open the
box, but whether or not she should. This means (B) is incorrect. At the
beginning of the story, the speaker is thinking about dancing around the
room, but that is not when she is sitting on the bed. She is sitting on the bed
near the end. Therefore (C) is incorrect.
11) C
In the middle of the story, the speaker says, I am almost eleven years old!
Almost means close to, but not quite. From this we can understand that the
speaker is close to eleven years old, but she is not eleven yet. The age
before eleven is ten. Therefore she must be ten years old, so (C) is correct.
The speaker is almost eleven, but she is not eleven yet. This means (A) is
incorrect. Twelve is older than eleven, so (B) is incorrect. Fifteen is much
older than eleven, so (D) is incorrect.
12) D
At the beginning, the speaker says, The box looks magical. It looks like it has
special powers. At the end, after she finally touches it, the box quickly flips
open by itself! Jewels fly into the air. She says, They dance around my
head. I feel strange. I fall down to the floor. I wake up in a place I have never
seen before. From this we can understand that touching the box has caused
strange things to happen, including the speaker waking up in a new place.
We can conclude that the box has special powers. This means (D) is correct.
The story does not contain information to support answer choices (A), (B),
and (C). Therefore they are incorrect.
13) B
big deal (noun) (slang): something of great importance or consequence.
Near the middle of the story, Momma tells the speaker that she is not old
enough to wear her jewelry. Then the speaker says, I am almost eleven
years old! What is the big deal? From this, we can understand that the
speaker believes she is already old enough. She does not think it is very
important to wait until she is older. But Momma thinks it is a big deal. Momma
thinks it is important for her to wait. From this we can understand that big deal
means something very important. Therefore (B) is correct.
The big deal here refers to the speaker not being old enough to wear
Mommas jewelry. It does not refer to a physical thing, so it cannot be heavy.
This makes (A) incorrect. The speaker could one day forget the fact that
Momma thought she was no old enough to wear the jewelry. This means (C)
is incorrect. Not being old enough to wear Mommas jewelry is not a thing that
cannot make a lot of money. Therefore (D) is incorrect.

14) D
float (verb): to move easily or lightly.
Near the end of the story, the speaker says, There is one necklace with a
yellow stone like a tigers eye. When Momma puts this on, she seems
to float instead of walk. Her feet do not seem to touch the ground. She moves
lightly and gracefully. She moves without effort. From this we can understand
that in this context, to float is to move lightly and gracefully, without effort. She
does not mean that Mommas feet actually do not touch the ground. She says
that they do not seem to touch the ground. Here, to float means to seem to
move without much effort. Therefore (D) is correct.
People do float in water, but Momma here is not in water, so she cannot be
swimming. That is a different meaning of float. This means (A) is incorrect.
Boats do float, but nothing in the story suggests that Momma looks like a boat
here. That is also a different meaning of float. This makes (B) incorrect. If
Momma moves lightly and gracefully, without effort, it is easy for her to move.
It is not difficult. Therefore (C) is incorrect.
15) D
At the end of the story, the girl touches the jewelry box. Then the box quickly
flips open by itself! Jewels fly into the air. She says, They dance around my
head. I feel strange. I fall down to the floor. I wake up in a place I have never
seen before. The speaker does not tell us exactly what has happened or
where she is. This could make the reader feel unsure about what has
happened. This supports option (I). Near the end of the story, the speaker
says that tonight she is very curious about the jewelry box. Then, she
touches it and feels like she wakes up in a new place. She does not say what
the power of the jewelry box is. This could make the reader feel curious about
the power of the box, too. This supports option (II). The story ends in a place
that leaves the reader full of questions. It is hard to predict where the speaker
is or what will happen. It is reasonable to believe that this could leave the
reader wondering what will happen next. This supports option (III). Therefore
(D) is correct.

ReadTheory.Org 2010

Name________________
Date________________

EnglishForEveryone.Org 2008

The Bus Driver


Reading Comprehension Short Stories

Directions: Read the story. Then answer the questions below.

Dana Miller is a bus driver.


She drives a bus in the city of Philadelphia.
She works the night shift. That means she starts work at 10:00 at night and gets off at
6:00 in the morning. In the world of bus driving, this is sometimes called third shift.
Dana has been driving a bus for 15 years. She started when she was 23 years old.
She loves her job most of the time. She gets to see the beautiful city from her seat.
She gets to meet all kinds of people.
There is the guitar player. He lugs his heavy guitar on the bus every Friday night. He
plays at a nightspot downtown.
People call him Get Low. That is because he likes to play the guitar on his knees.
If the bus is not too crowded, Get Low plays in the back of the bus. The bus is seldom
crowded at night. He likes to warm up on the bus before shows.
There is the woman who works at the pretzel factory. Each day the Pretzel Lady
brings Dana a bag of fresh, soft pretzels. They fill the bus with a wonderful smell!
Sometimes Danas job is hard. It is hard when Dana sees people down on their luck.
One time, a young woman left her sleeping baby on the bus. There was a note on the
baby's blanket asking for help. Dana had to call the police.
Another time, a teenaged boy stole money from Dana on the bus. He had a fake gun.
Dana thought it was real. She was very scared.
Occasionally, driving the bus can be scary. But most of the time, driving the bus is
great. Dana does not want to work anywhere else.
How was your night? Danas husband likes to ask when she gets home.
Good music, good food, and a great view of the city, she says.

Questions:
1) Where does Dana drive the bus?
A.
B.
C.
D.

around Philadelphia
to the police station
to pick up guitar players
to pick up school children

3) How old is Dana?


A.
B.
C.
D.

15
23
38
39

5) What is one reason Dana loves her


job?
A. She finds sleeping babies.
B. She gets to call the police.
C. She gets to sleep during the
day.
D. She gets to meet all kinds of
people.

7) What does lug mean?


A.
B.
C.
D.

to throw
to push easily
to carry on one's head
to carry something that is hard
to move around

9) How often does Get Low play guitar


on the bus?
I. when the bus is not crowded
II. when he doesnt have a show
III. every Friday night
A.
B.
C.
D.

I only
I and II
II and III
I, II, and III

2) What shift does Dana work?


A.
B.
C.
D.

third shift
evening shift
afternoon shift
second shift

4) How long is Dana's shift?


A.
B.
C.
D.

6 hours
8 hours
10 hours
12 hours

6) Why is the guitar player called Get


Low?
A.
B.
C.
D.

He has a low voice.


He likes to play sitting down.
He likes to play on his knees.
It is the name of his favorite
song.

8) The main reason Get Low rides the


bus is because
A.
B.
C.
D.

he likes to see the city


he enjoys talking to Dana
he likes to play guitar there
he needs to get to a show

10) Who gives Dana pretzels?


A.
B.
C.
D.

the guitar player


the young mom
the teenager
the woman who works at the
factory

Questions:
11) Why might the young woman have
left her baby on the bus?
A.
B.
C.
D.

A.
B.
C.
D.

She forgot it there.


It was sleeping.
She meant to come right back.
She felt she could not take care
of it.

13) Why does Dana seem to think


people do bad things?
I. They are down on their luck.
II. They are bad by nature.
III. They know Dana is vulnerable.
A.
B.
C.
D.

12) Who steals money?


Get Low
the babys mother
a teenager
Dana

14) As used at the end of the story,


which is the best synonym
for occasionally?

I only
I and II
II and III
I, II, and III

A.
B.
C.
D.

seldom
one time
frequently
usually

15) What great view of the city does


Dana have?
A.
B.
C.
D.

the view from a tall building


the view from a mountain top
the view from the sidewalk
the view from her bus seat

Do you think Dana has a good job? Explain.


______________________________________________________________________________
______________________________________________________________________________
______________________________________________________________________________
______________________________________________________________________________
______________________________________________________________________________
______________________________________________________________________________
______________________________________________________________________________

Answers and Explanations


1) A
In paragraph 2, the author tells us, Dana drives a bus in the city of Philadelphia.
Therefore (A) is correct. The passage does not provide information to support
answer choices (B), (C), and (D). Therefore they are incorrect.
2) A
In paragraph 3, the author writes that Dana works the night shift. The author
explains, In the world of bus driving, this is sometimes called third shift. Therefore
(A) is correct. The passage does not provide information to support answer choices
(B), (C), and (D). Therefore they are incorrect.
3) C
In paragraph 4, we learn that Dana has been driving a bus for 15 years. The author
states, She started when she was 23 years old. To find out how old Dana is now,
we need to add the number of years Dana has been driving a bus to the age she
was when she started. Adding 15 to 23 gives us 38 (15 + 23 = 38). Therefore (C) is
correct.
The passage does not provide information to support answer choices (A) and (D).
Therefore they are incorrect. Dana started driving the bus when she was 23 years
old, but that was 15 years ago, so she is no longer 23. Therefore (B) is incorrect.
4) B
Paragraph 3 explains that Dana starts work at 10:00 at night and is done at 6:00 in
the morning. There are 2 hours between 10:00 and midnight. There are six hours
between midnight and 6:00. This means there are 8 hours between 10:00 at night
and 6:00 in the morning. Therefore (B) is correct.
The passage does not provide information to support answer choices (A), (C), and
(D). Therefore they are incorrect.
5) D
In paragraph 5, we learn that Dana loves her job most of the time. The next
paragraph states, She gets to meet all kinds of people. We can understand that
one reason Dana loves her job is that she is able to meet all kinds of people.
Therefore (D) is correct.
One time Dana found a sleeping baby on her bus, but that was an example of a time
when Danas job was hard. It is not what she loves about her job. Therefore (A) is
incorrect. When Dana found the baby, she had to call the police. But that is not what
she loves about her job. Therefore (B) is incorrect. If Dana works all night, it is
probably true that she gets to sleep during the day. But the passage does not say
that is what she loves about her job. Therefore (C) is incorrect.
6) C
In paragraph 8, we learn about the guitar player who rides the bus. The author
writes, People call him Get Low. That is because he likes to play the guitar on his
knees. Therefore (C) is correct.

The passage does not provide information to support answer choices (A), (B), and
(D). Therefore they are incorrect.
7) D
In paragraph 7, we learn that Get Low lugs his heavy guitar case on the bus every
Friday night. Because Get Lows guitar case is heavy, we can understand that it is
hard to move around. To lug something means to carry something that is hard to
move around. Therefore (D) is correct.
The passage does not provide information to support answer choices (A), (B), and
(C). Therefore they are incorrect.
8) D
Paragraph 7 tells us that Get Low plays at a nightspot downtown. We can
understand that Get Low is using the bus to get to a show. Therefore (D) is correct.
The passage does not provide information to support answer choices (A) and (B).
Therefore they are incorrect. Although Get Low does play guitar on the bus,
paragraph 9 tells us this is because he likes to warm up on the bus between
shows. Therefore (C) is incorrect.
9) A
In paragraph 9, we learn that if the bus is not too crowded, Get Low plays in the
back of the bus. Get Low only plays if the bus is not crowded. This supports option
(I). Paragraph 9 tells us that Get Low plays on the bus to warm up before a show.
We can understand that he would not be playing if he did not have a show. This
eliminates option (II). Paragraph 9 explains, The bus is seldom crowded at night.
If something happens seldom, this means it does not happen very often. So we can
understand from this sentence that some Friday nights the bus might be crowded,
although usually it is not. Get Low does not play if the bus is crowded. This
eliminates option (III). Therefore (A) is correct.
10) D
Paragraph 10 tells us about the woman who works at the pretzel factory. We learn
that each day the Pretzel Lady brings Dana a bag of fresh, soft pretzels. They fill
the bus with a wonderful smell. Therefore (D) is correct. The passage does not
provide information to support answer choices (A), (B), and (C). Therefore they are
incorrect.
11) D
In paragraph 12, the author tells us, There was a note on the babys blanket asking
for help. We can understand from the note that the babys mother felt she could not
take care of the baby by herself. Therefore (D) is correct. The passage does not
provide information to support answer choices (A), (B), and (C). Therefore they are
incorrect.

12) C
Paragraph 13 describes how a teenaged boy stole money from Dana on the bus.
Therefore (C) is correct. The passage does not provide information to support
answer choices (A), (B), and (D). Therefore they are incorrect.
13) A
In paragraph 11, the author writes, It is hard when Dana sees people down on their
luck. In the next 2 paragraphs, the author gives examples of people who do things
that might be thought of as bad. In paragraph 12, the author tells us about the
woman who left her baby on the bus. The paragraph after that, paragraph 13, tells
us about the teenager who stole money from Dana. Using this information, we can
understand that Dana believes people like the woman and the teenager do bad
things because they are down on their luck. This supports option (I). There is no
information in the passage to suggest that Dana thinks these people are bad by
nature. This eliminates option (II). There is no information in the passage to suggest
that Dana is vulnerable. This eliminates option (III). Therefore (A) is correct.
14) A
Paragraph 14 reads, Occasionally, driving the bus can be scary. But most of the
time, driving the bus is great. Since driving the bus is great most of the time,
occasionally must be the opposite of most of the time. Seldom, or infrequently, is the
only answer choice that means the opposite of most of the time. Therefore (A) is
correct.
The passage does not provide information to support answer choices (B), (C), and
(D). Therefore they are incorrect.
15) D
In paragraph 15, Danas husband asks her about her night when she has gotten
home from driving the bus. In paragraph 16, Dana tells him, Good music, good
food, and a great view of the city. Because Dana is answering her husbands
question about how her job went that night, we can understand that Dana gets the
great view of the city from the drivers seat of the bus. Therefore (D) is correct.
The passage does not provide information to support answer choices (A), (B), and
(C). Therefore they are incorrect.

Você também pode gostar